Anda di halaman 1dari 191

KANDUNGAN

Rekod Pentaksiran Murid iii 4.2 Muatan Haba Tentu 62


Objektif Pentaksiran vi Specific Heat Capacity
4.3 Haba Pendam Tentu 69
BAB Specific Latent Heat
1 Pengukuran
Measurement
1 4.4 Hukum Gas 76
Gas Laws
1.1 Kuantiti Fizik 2 Praktis Sumatif 84
Physical Quantities
1.2 Penyiasatan Saintifik 4 BAB
Scientific Investigation 5 Gelombang
Waves
90
Praktis Sumatif 8
5.1 Asas Gelombang 92
BAB Fundamentals of Waves
2 Daya dan Gerakan I
Force and Motion I
11 5.2 Pelembapan dan Resonans 96
Damping and Resonance
2.1 Gerakan Linear 13 5.3 Pantulan Gelombang 99
Linear Motion Reflection of Waves
2.2 Graf Gerakan Linear 18 5.4 Pembiasan Gelombang 101
Linear Motion Graphs Refraction of Waves
2.3 Gerakan Jatuh Bebas 20 5.5 Pembelauan Gelombang 105
Free Fall Motion Diffraction of Waves
2.4 Inersia 23 5.6 Interferens Gelombang 109
Inertia Interference of Waves
2.5 Momentum 27 5.7 Gelombang Elektromagnet 113
Momentum Electromagnetic Waves
2.6 Daya 30 Praktis Sumatif 116
Force
2.7 Impuls dan Daya Impuls 31 BAB
Impulse and Impulsive Force 6 Cahaya dan Optik
Light and Optics
122
2.8 Berat 34
Weight 6.1 Pembiasan Cahaya 124
Praktis Sumatif 36 Refraction of Light
6.2 Pantulan Dalam Penuh 131
BAB Total Internal Reflection
3 Kegravitian
Gravitation
42 6.3 Pembentukan Imej oleh Kanta 134
Image Formation by Lenses
3.1 Hukum Kegravitian Semesta Newton 43 6.4 Formula Kanta Nipis 137
Newton’s Universal Law of Gravitation Thin Lens Formula
3.2 Hukum Kepler 49 6.5 Peralatan Optik 141
Kepler’s Laws Optical Instruments
3.3 Satelit Buatan Manusia 52 6.6 Pembentukan Imej oleh Cermin Sfera 143
Man-made Satellites Image Formation by Spherical Mirrors
Praktis Sumatif 55 Praktis Sumatif 147
BAB Penilaian Akhir Tahun 152
4 Haba
Heat
59

4.1 Keseimbangan Terma 60 JAWAPAN


Thermal Equilibrium bit.ly/3X664Zg

Fizik Tg 4 (Prelims)2nd 9/2/23.indd 2 09/02/2023 5:55 PM


Rekod Pentaksiran Murid
Fizik Tingkatan 4

Nama:

Sekolah: Kelas:

Standard Prestasi Penguasaan

Tahap Menguasai (✓)


Tafsiran
Penguasaan Belum Menguasai (✗)
Bab 1 Pengukuran
Mengingat kembali pengetahuan dan kemahiran asas sains
1
mengenai Pengukuran.
2 Memahami Pengukuran serta dapat menjelaskan kefahaman tersebut.
Mengaplikasikan pengetahuan mengenai Pengukuran untuk
3 menerangkan kejadian atau fenomena alam dan melaksanakan
tugasan mudah.
Menganalisis pengetahuan mengenai Pengukuran dalam konteks
4
penyelesaian masalah mengenai kejadian atau fenomena alam.
Menilai pengetahuan mengenai Pengukuran dalam konteks penyelesaian
5
masalah dan membuat keputusan untuk melaksanakan satu tugasan.
Mereka cipta menggunakan pengetahuan dan kemahiran sains
mengenai Pengukuran dalam konteks penyelesaian masalah atau
6 membuat keputusan dalam melaksanakan aktiviti/ tugasan dalam
situasi baharu secara kreatif dan inovatif dengan mengambil kira nilai
sosial/ ekonomi/ budaya masyarakat.
Bab 2 Daya dan Gerakan I
Mengingat kembali pengetahuan dan kemahiran asas sains
1
mengenai Daya dan Gerakan I.
Memahami Daya dan Gerakan I serta dapat menjelaskan kefahaman
2
tersebut.
Mengaplikasikan pengetahuan mengenai Daya dan Gerakan I untuk
3 menerangkan kejadian atau fenomena alam dan melaksanakan
tugasan mudah.
Menganalisis pengetahuan mengenai Daya dan Gerakan I dalam konteks
4
penyelesaian masalah mengenai kejadian atau fenomena alam.
Menilai pengetahuan mengenai Daya dan Gerakan I dalam konteks
5 penyelesaian masalah dan membuat keputusan untuk melaksanakan
satu tugasan.
Mereka cipta menggunakan pengetahuan dan kemahiran sains
mengenai Daya dan Gerakan I dalam konteks penyelesaian masalah
6 atau membuat keputusan dalam melaksanakan aktiviti/ tugasan
dalam situasi baharu secara kreatif dan inovatif dengan mengambil
kira nilai sosial/ ekonomi/ budaya masyarakat.

iii

Fizik Tg 4 (Prelims)2nd 9/2/23.indd 3 09/02/2023 5:55 PM


Standard Prestasi Penguasaan

Tahap Menguasai (✓)


Tafsiran
Penguasaan Belum Menguasai (✗)
Bab 3 Kegravitian
Mengingat kembali pengetahuan dan kemahiran asas sains
1
mengenai Kegravitian.
2 Memahami Kegravitian serta dapat menjelaskan kefahaman tersebut.
Mengaplikasikan pengetahuan mengenai Kegravitian untuk
3 menerangkan kejadian atau fenomena alam dan melaksanakan
tugasan mudah.
Menganalisis pengetahuan mengenai Kegravitian dalam konteks
4
penyelesaian masalah mengenai kejadian atau fenomena alam.
Menilai pengetahuan mengenai Kegravitian dalam konteks
5 penyelesaian masalah dan membuat keputusan untuk melaksanakan
satu tugasan.
Mereka cipta menggunakan pengetahuan dan kemahiran sains
mengenai Kegravitian dalam konteks penyelesaian masalah atau
6 membuat keputusan dalam melaksanakan aktiviti/ tugasan dalam
situasi baharu secara kreatif dan inovatif dengan mengambil kira nilai
sosial/ ekonomi/ budaya masyarakat.
Bab 4 Haba
Mengingat kembali pengetahuan dan kemahiran asas sains
1
mengenai Haba.
2 Memahami Haba serta dapat menjelaskan kefahaman tersebut.

Mengaplikasikan pengetahuan mengenai Haba untuk menerangkan


3
kejadian atau fenomena alam dan melaksanakan tugasan mudah.

Menganalisis pengetahuan mengenai Haba dalam konteks


4
penyelesaian masalah mengenai kejadian atau fenomena alam.

Menilai pengetahuan mengenai Haba dalam konteks penyelesaian


5
masalah dan membuat keputusan untuk melaksanakan satu tugasan.
Mereka cipta menggunakan pengetahuan dan kemahiran sains
mengenai Haba dalam konteks penyelesaian masalah atau membuat
6 keputusan dalam melaksanakan aktiviti/ tugasan dalam situasi
baharu secara kreatif dan inovatif dengan mengambil kira nilai sosial/
ekonomi/ budaya masyarakat.

iv

Fizik Tg 4 (Prelims)2nd 9/2/23.indd 4 09/02/2023 5:55 PM


Standard Prestasi Penguasaan

Tahap Menguasai (✓)


Tafsiran
Penguasaan Belum Menguasai (✗)
Bab 5 Gelombang
Mengingat kembali pengetahuan dan kemahiran asas sains
1
mengenai Gelombang.
Memahami Gelombang serta dapat menjelaskan kefahaman
2
tersebut.
Mengaplikasikan pengetahuan mengenai Gelombang untuk
3 menerangkan kejadian atau fenomena alam dan melaksanakan
tugasan mudah.
Menganalisis pengetahuan mengenai Gelombang dalam konteks
4
penyelesaian masalah mengenai kejadian atau fenomena alam.
Menilai pengetahuan mengenai Gelombang dalam konteks
5 penyelesaian masalah dan membuat keputusan untuk melaksanakan
satu tugasan.
Mereka cipta menggunakan pengetahuan dan kemahiran sains
mengenai Gelombang dalam konteks penyelesaian masalah atau
6 membuat keputusan dalam melaksanakan aktiviti/ tugasan dalam
situasi baharu secara kreatif dan inovatif dengan mengambil kira nilai
sosial/ ekonomi/ budaya masyarakat.
Bab 6 Cahaya dan Optik
Mengingat kembali pengetahuan dan kemahiran asas sains
1
mengenai Cahaya dan Optik.
Memahami Cahaya dan Optik serta dapat menjelaskan kefahaman
2
tersebut.
Mengaplikasikan pengetahuan mengenai Cahaya dan Optik untuk
3 menerangkan kejadian atau fenomena alam dan melaksanakan
tugasan mudah.
Menganalisis pengetahuan mengenai Cahaya dan Optik dalam
4 konteks penyelesaian masalah mengenai kejadian atau fenomena
alam.
Menilai pengetahuan mengenai Cahaya dan Optik dalam konteks
5 penyelesaian masalah dan membuat keputusan untuk melaksanakan
satu tugasan.
Mereka cipta menggunakan pengetahuan dan kemahiran sains
mengenai Cahaya dan Optik dalam konteks penyelesaian masalah
6 atau membuat keputusan dalam melaksanakan aktiviti/ tugasan
dalam situasi baharu secara kreatif dan inovatif dengan mengambil
kira nilai sosial/ ekonomi/ budaya masyarakat.

Fizik Tg 4 (Prelims)2nd 9/2/23.indd 5 09/02/2023 5:55 PM


Objektif Pentaksiran
Fizik Tingkatan 4

Objektif pentaksiran (OP) berdasarkan matlamat dan objektif mata pelajaran Fizik adalah untuk
mentafsir pencapaian murid dalam keupayaan untuk:

OP1

Kebolehan mengamalkan sikap saintifik dan nilai murni serta mempamerkan pengetahuan dan
kemahiran melalui penyiasatan saintifik;

OP2

Kebolehan mengingat kembali pengetahuan fizik;

OP3

Kebolehan menjelaskan pengetahuan fizik untuk menyampaikan idea-idea dalam konteks yang
berkaitan;

OP4

Kebolehan mengaplikasi pengetahuan fizik untuk menjelaskan kejadian atau fenomena alam secara
kritis dan kreatif;

OP5

Kebolehan menganalisis pengetahuan fizik mengenai kejadian atau fenomena alam secara objektif;

OP6

Kebolehan menilai maklumat dengan menggunakan pengetahuan fizik dan membuat keputusan
secara objektif serta berfikiran terbuka;

OP7

Kebolehan mereka cipta dengan menggabungkan komponen-komponen untuk menghasilkan


keseluruhan idea atau sesuatu yang baharu dan kreatif menggunakan pengetahuan fizik.

vi

Fizik Tg 4 (Prelims)2nd 9/2/23.indd 6 09/02/2023 5:55 PM


Fizik Tingkatan 4 Bab 1

BAB
1
Pengukuran
Measurement

Tema Asas Fizik


Theme Elementary Physics

NOTA EFEKTIF

1 Kuantiti fizik terdiri daripada kuantiti asas dan kuantiti terbitan.


Physical quantities consists of base quantity and derived quantity.
(a) Kuantiti asas: Kuantiti yang tidak boleh ditakrifkan dalam kuantiti fizik yang lain.
Base quantity: A quantity that cannot be expressed in terms of other physical quantities.
(b) Kuantiti terbitan: Gabungan kuantiti asas secara pendaraban, pembahagian, atau kedua-dua operasi ini.
Derived quantity: Combination of base quantities by operation of multiplication, division or both.
2 (a) Kuantiti skalar: Kuantiti fizik yang mempunyai magnitud sahaja.
Scalar quantity: Physical quantity which has magnitude only.
(b) Kuantiti vektor: Kuantiti fizik yang mempunyai kedua-dua magnitud dan arah.
Vector quantity: Physical quantity which has magnitude and direction.
3 Mentafsir bentuk graf/ Interpret graphs
y y

y bertambah
y berkadar terus
secara linear
dengan x
dengan x
y directly
c y increases linearly
proportional to x
with x
0 x x

y y y

c y bertambah
y berkurang secara
linear dengan x dengan x
y decreases linearly y increases
with x with x

0 x 0 x 0 x

y y y y

1
0 x 0 x 0 x 0 x
y berkurang dengan x y berkadar songsang dengan x
y decreases with x y inversely proportional to x

4 Laporan lengkap bagi suatu eksperimen terdiri daripada inferens, hipotesis, tujuan, pemboleh ubah,
bahan dan radas, keputusan, perbincangan dan kesimpulan.
The complete report of an experiment consists of inference, hypothesis, variables, apparatus and materials, result,
discussion and conclusion.

1 © Penerbit Mahir Sdn. Bhd. (183897-P)

Fizik Tg 4 (Bab 1)6th 9/2/23.indd 1 09/02/2023 4:19 PM


Fizik Tingkatan 4 Bab 1

1.1 Kuantiti Fizik


Physical Quantities

Buku Teks: m.s 4-9

1 Apakah maksud kuantiti fizik? TP 1 OP2


What is the meaning of physical quantity?

Kuantiti yang boleh diukur atau dikira pada sesuatu objek atau fenomena.
The quantity that can be measured or calculated on an object or phenomena.

2 Labelkan kuantiti fizik, magnitud dan unit bagi hasil pengukuran di bawah. TP 1 OP2
Label the physical quantity, magnitude and unit for the measurement below.

Panjang buku = 20 cm
Length of book

(a) Kuantiti fizik (b) Magnitud (c) Unit S.I


Physical quantity Magnitude S.I unit

3 Apakah maksud kuantiti asas? TP 1 OP2


What is the meaning of base quantity?

Kuantiti yang tidak dapat ditakrifkan dalam sebutan kuantiti-kuantiti fizik yang lain.
A quantity that cannot be defined in terms of other physical quantities.

4 Lengkapkan jadual di bawah dengan simbol dan unit S.I bagi kuantiti asas diberi. TP 2 OP3
Complete the table below with their symbols and S.I units for the given base quantities.

Kuantiti asas Simbol Unit S.I dan simbol


Base quantity Symbol S.I unit and its symbol

Jisim
m kilogram kg
Mass

(a) Masa saat


t s
Time second

(b) Panjang meter


l m
Length metre

(c) Suhu
T kelvin K
Temperature

© Penerbit Mahir Sdn. Bhd. (183897-P) 2

Fizik Tg 4 (Bab 1)6th 9/2/23.indd 2 09/02/2023 4:19 PM


Fizik Tingkatan 4 Bab 1
5 Apakah maksud kuantiti terbitan? TP 1 OP2
What is the meaning of derived quantity?

Gabungan kuantiti-kuantiti asas secara pendaraban atau pembahagian atau gabungan kedua-duanya.
Combination of base quantities by multiplication, division or both these operations.

6 Tentukan unit bagi kuantiti terbitan berikut. TP 2 OP3


Determine the unit for the following derived quantities.

(a) Luas = panjang × lebar (f) perubahan halaju


Area = length × width Pecutan =
masa
change in velocity
Acceleration =
m × m = m2 time
m s–1
= m s−2
s

(b) Isi padu = panjang × lebar × tinggi (g) Daya = jisim × pecutan
Volume = length × width × height Force = mass × acceleration

m × m × m = m3 kg × m s–2 = kg m s−2 atau/ or N

(c) jisim (h) daya


Ketumpatan = Tekanan =
isi padu luas
mass force
Density = Pressure =
volume area
kg N
= kg m−3 = N m−2 atau/ or Pa
m 3
m2

(d) panjang (i) Kerja = daya × sesaran


Halaju = Work done = force × displacement
masa
length
Velocity =
time
m N × m = N m atau/ or J
= m s−1
s

(e) Momentum = jisim × halaju (j) Cas = arus × masa


Momentum = mass × velocity Charge = current × time

kg × m s−1 = kg m s−1 A × s = A s atau/ or C

3 © Penerbit Mahir Sdn. Bhd. (183897-P)

Fizik Tg 4 (Bab 1)6th 9/2/23.indd 3 09/02/2023 4:19 PM


Fizik Tingkatan 4 Bab 1
7 Lengkapkan jadual di bawah mengenai perbandingan antara kuantiti skalar dan kuantiti vektor. TP 2 OP3
Complete the table below of the comparison between scalar quantities and vector quantities.

Jenis Kuantiti skalar (b) Kuantiti terbitan


Type Scalar quantities Derived quantities
(a) Kuantiti fizik yang mempunyai
Kuantiti fizik yang mempunyai magnitud dan arah.
Definisi magnitud sahaja. Physical quantity which has both magnitude and
Definition Physical quantity which has magnitude
direction.
only.

(c) Sesaran, halaju, pecutan, momentum, tekanan,


Jarak, laju, jisim, kuasa, tenaga, masa, suhu
Contoh daya
Distance, speed, mass, power, energy, time,
Example Displacement, velocity, acceleration, momentum,
temperature
pressure, force

1.2 Penyiasatan Saintifik


Scientific Investigation

Buku Teks: m.s 10-16


1 Mentafsir bentuk-bentuk graf berikut untuk menentukan hubungan antara dua kuantiti fizik. TP 2 OP3
Interpret the following graphs to determine the relationship between two physical quantities.

(a) y
Jenis graf: Garis lurus yang melalui asalan dan
(x2, y2)
mempunyai kecerunan positif .
Type of graph: A straight line that passing through
Δy = y2 – y1
(x1, y1)
origin with a positive gradient.
Δx = x2 – x1
Tafsiran graf: y berkadar terus dengan x
0 x
Interpret graph: y directly proportional to x

(b) lurus yang tidak melalui asalan dan


y
Jenis graf: Garis
(x2, y2)
mempunyai kecerunan positif .
Δy = y2 – y1
(x1, y1) Type of graph: A straight line that does not pass through
Δx = x2 – x1 positive
(0, c) origin with a gradient.

Tafsiran graf: y bertambah secara linear dengan x


0 x
Interpret graph: y increases linearly with x

(c) y Jenis graf: Garis lurus yang tidak melalui

asalan dan mempunyai kecerunan negatif .


(0, c) (x1, y1)
Type of graph: A straight line that does not pass through

origin with a negative gradient.


(x2, y2)
Δx
Tafsiran graf: y berkurang secara linear dengan x
0 x
Interpret graph: y decreases linearly with x

© Penerbit Mahir Sdn. Bhd. (183897-P) 4

Fizik Tg 4 (Bab 1)6th 9/2/23.indd 4 09/02/2023 4:19 PM


Fizik Tingkatan 4 Bab 1

(d) y

Jenis graf: Garis melengkung yang melalui

asalan dan mempunyai kecerunan positif .


0 x
Type of graph: A curve that passing

y through origin with a positive gradient.

Tafsiran graf: y bertambah dengan x


Interpret graph: y increases with x

0 x

(e) y

Jenis graf: Garis melengkung yang


tidak melalui asalan dan mempunyai kecerunan

negatif .
0 x
Type of graph: A curve that does not pass
y
through origin with a negative gradient.

Tafsiran graf: y berkurang dengan x


Interpret graph: y decreases with x

0 x

(f)
Jenis graf:
y
Garis melengkung yang tidak memintas

paksi dan mempunyai kecerunan negatif .

Garis lurus yang melalui

asalan dan mempunyai kecerunan positif .


0 x

Type of graph: A curve that does not cut


y

both axes with a negative gradient.

A straight line that passes through the

origin with a positive gradient.

1 Tafsiran graf: y berkadar songsang dengan x


0
x
Interpret graph: y inversely proportional to x

5 © Penerbit Mahir Sdn. Bhd. (183897-P)

Fizik Tg 4 (Bab 1)6th 9/2/23.indd 5 09/02/2023 4:19 PM


Fizik Tingkatan 4 Bab 1

EKSPERIMEN 1.1

Inferens: Tempoh ayunan bandul bergantung kepada panjang talinya.


The period of oscillation of a simple pendulum depends on its length.
Inference:

Hipotesis: Semakin panjang bandul, semakin panjang tempoh ayunan.


The longer the length of the simple pendulum, the longer the period of oscillation.
Hypothesis:

Tujuan: Mengkaji hubungan antara panjang bandul dengan tempoh ayunan bandul.
To investigate the relationship between the length of a simple pendulum and the period of oscillation.
Aim:

Pemboleh ubah dimanipulasi: Panjang bandul, l


Length of pendulum, l
Manipulated variable:

Pemboleh ubah bergerak balas: Tempoh ayunan, T


Period of oscillation, T
Responding variable:

Pemboleh ubah dimalarkan: Jisim ladung


Mass of pendulum
Constant variable:

Radas: Kaki retort, jangka sudut, ladung bandul, jam randik, pembaris meter dan benang 100 cm
Retort stand, protractor, pendulum bob, stopwatch, metre ruler and 100 cm thread
Apparatus:

Prosedur/ Procedure:
(i) Laraskan panjang bandul, l = 20.0 cm
Adjust the length of pendulum, l = 20.0 cm
(ii) Sesarkan ladung bandul ke sisi dan lepaskan supaya ladung bandul itu
berayun dengan sudut yang kurang daripada 10°.
Displace the pendulum bob at an angle of less than 10° from the vertical and
release it.
(iii) Ukur dan rekodkan masa, t1 untuk 20 ayunan lengkap.
Measure and record the time, t1 taken for 20 complete oscillations.
(iv) Ulangi langkah (iii) dan rekodkan masa, t2.
Repeat (iii) and record the time as t2.
l 10°
(v) Hitungkan nilai masa purata, tpurata . Kaki retort
Calculate the average time, taverage. Retort stand
tpurata
(vi) Hitungkan tempoh ayunan, T = 20 dan nilai T 2. Ladung
taverage Bob
Calculate the period of oscillation, T = and T 2.
20
(vii) Ulangi eksperimen dengan l = 30.0 cm, 40.0 cm, 50.0 cm, 60.0 cm dan 70.0 cm.
Repeat the experiment with l = 30.0 cm, 40.0 cm, 50.0 cm, 60.0 cm and 70.0 cm.

© Penerbit Mahir Sdn. Bhd. (183897-P) 6

Fizik Tg 4 (Bab 1)6th 9/2/23.indd 6 09/02/2023 4:19 PM


Fizik Tingkatan 4 Bab 1
Keputusan/ Result:
(i) Jadualkan data bagi nilai l, t, T dan T2.
Tabulate data for values l, t, T and T2.

Masa diambil untuk 20 ayunan lengkap,


Panjang bandul, Time for 20 complete oscillations, t/s
Length of pendulum, l /cm T/s T2/s2
t1 t2 tpurata/average

20.0 20.0 22.0 21.0 1.05 1.1


30.0 24.0 24.0 24.0 1.20 1.4
40.0 26.0 26.0 26.0 1.30 1.7
50.0 27.0 27.0 27.0 1.40 1.8
60.0 30.0 30.0 30.0 1.50 2.3
70.0 34.0 33.0 34.0 1.70 2.9

(ii) Graf T 2 melawan l dilakarkan.


A graph of T 2 against l is plotted.
T2/ s2

3.0

2.5

2.0

1.5

1.0

0.5

l/ cm
0 10 20 30 40 50 60 70
Kesimpulan/ Conclusion:
T 2 berkadar terus dengan l / T 2 is directly proportional to l

Perbincangan/ Discussion:
(i) Tentukan kecerunan graf, m bagi graf T 2 melawan l. Nyatakan nilai m dalam unit S.I. Tunjukkan pada
graf.
Determine the gradient, m of the graph of T 2 against l. State the value of m in S.I. unit. Show on the graph.
2.5 − 0
m= = 0.417 s2 cm−1 = 4.17 s2 m−1
60 − 0
(ii) Mengapakah pengukuran masa 20 ayunan lengkap perlu diulang?
Why is the measurement of time taken for 20 complete oscillations repeated?
Untuk meningkatkan kejituan bacaan tempoh ayunan T.

To increase the accuracy of the period of oscillation, T.

7 © Penerbit Mahir Sdn. Bhd. (183897-P)

Fizik Tg 4 (Bab 1)6th 9/2/23.indd 7 09/02/2023 4:19 PM


Fizik Tingkatan 4 Bab 1

Praktis Sumatif
KERTAS 1

1 Maklumat berikut menunjukkan satu kumpulan 3 Antara berikut, rumus manakah merupakan
OP2 kuantiti fizik. OP3 terbitan bagi momentum?
The following information show a group of physical Which of the following formulas is derived for
quantities. momentum?
m
A m×a C v
• Jisim/ Mass F
• Laju/ Speed B m×v D A
• Suhu/ Temperature
• Masa/ Time 4 Kuantiti vektor merupakan kuantiti fizik yang
OP2 mempunyai magnitud dan arah.
Kumpulan ini boleh dikategorikan sebagai Apakah contoh kuantiti vektor?
The group can be categorised as Vector quantity is the physical quantity that has
magnitude and direction.
A kuantiti asas
What is the example of the vector quantity?
base quantity
B kuantiti skalar A Daya C Laju
Force Speed
scalar quantity
C kuantiti vektor B Tenaga D Ketumpatan
Energy Density
vector quantity
D kuantiti terbitan
derived quantity
5 Rajah 1 menunjukkan graf halaju, v melawan
OP3 masa, t.
Diagram 1 shows a graph of velocity, v against time, t.
2 Antara berikut, pasangan yang manakah betul?
OP2 Which of the following pairs is correct? v

Kuantiti fizik Unit S.I


Physical quantity S.I. unit

Arus elektrik
A candela
Electric current
t
Rajah 1
Berat Diagram 1
B kilogram
Weight Apakah hubungan antara v dengan t?
What is the relationship between v and t?
A v berkurang secara linear dengan t
Suhu
C kelvin v decreases linearly with t
Temperature
B v berkurang dengan t
v decreases with t
C v berkadar terus dengan t
Cas
D ampere v directly proportional to t
Charge
D v berkadar songsang dengan t
v inversely proportional to t

© Penerbit Mahir Sdn. Bhd. (183897-P) 8

Fizik Tg 4 (Bab 1)6th 9/2/23.indd 8 09/02/2023 4:19 PM


Fizik Tingkatan 4 Bab 1

KERTAS 2
Bahagian A
1 Persamaan bagi Hukum Kegravitian Semesta Newton adalah seperti berikut:
The equation for Newton’s Universal Law of Gravitational as follows:

GMm
F=
r2

iaitu F = daya, G = pemalar kegravitian, M atau m = jisim dan r = jarak.


in which F = force, G= gravitational constant, M or m = mass and r = distance.

(a) Berdasarkan persamaan tersebut, nyatakan satu contoh bagi setiap yang berikut: OP2
Based on the equation, give an example for each of the following:

(i) Kuantiti asas: Jisim // Jarak

Base quantity: Mass // Distance

(ii) Kuantiti terbitan: Daya // Pemalar kegravitian

Derived quantity: Force // Gravitational constant

(iii) Kuantiti vektor: Daya

Vector quantity: Force


[3 markah/ marks]

(b) Terbitkan unit bagi G dalam sebutan unit asas S.I. OP3
Derive the unit of G in terms of S.I. base unit.
Fr 2 N m2
G= → = N m2 kg–2
Mm kg × kg
[1 markah/ mark]

2 Rajah 1 menunjukkan bacaan jam randik pada awal dan akhir suatu eksperimen. Jam randik ini digunakan
untuk mengukur masa 20 ayunan lengkap suatu bandul ringkas yang panjangnya, l.
Diagram 1 shows the reading of a stopwatch at the beginning and end of a experiment. The stopwatch is used to measure
the time taken for 20 complete oscillations of a simple pendulum of length, l.

60 60 60 60
55 30
5 55 30
5 55 30
5 55 30
5
50 20 10 10 50 20 10 50 10 20 10 10 50 20 10 10

45 15 45 45 15 15 45 15

40 20 40 40 20 20 40 20
35 25 35 25 35 25 35 25
30 30 30 30

Awal eksperimen Akhir eksperimen


Beginning of experiment End of experiment

Rajah 1
Diagram 1

9 © Penerbit Mahir Sdn. Bhd. (183897-P)

Fizik Tg 4 (Bab 1)6th 9/2/23.indd 9 09/02/2023 4:19 PM


Fizik Tingkatan 4 Bab 1
(a) Berapakah masa yang diambil untuk bandul itu melengkapkan 20 ayunan lengkap? OP3
What is the time taken for the pendulum to make 20 complete oscillations?
t = 26 – 2
= 24 s
[1 markah/ mark]

(b) Mengapakah masa untuk 20 ayunan lengkap perlu diambil? OP4


Why is it necessary to take the time for 20 complete oscillations?
Untuk mendapatkan bacaan lebih jitu.

To increase the accuracy of the measurement.


[1 markah/ mark]

(c) Cadangkan satu langkah penambahbaikan untuk eksperimen ini. OP5 KBAT
Suggest one way to improve the experiment.
Ayunkan bandul dahulu dan kemudian hidupkan jam randik serta mula kira bilangan ayunan lengkap.
Swing the pendulum first, then start the stopwatch and start counting the complete oscillation.
[1 markah/ mark]

(d) (i) Tentukan tempoh ayunan lengkap, T bagi bandul ini. OP3
Determine the period of oscillation, T of this pendulum.
24.0
T=
20
= 1.2 s

[2 markah/ marks]

(ii) Hubungan antara panjang, l dan tempoh, T, suatu bandul ringkas diberikan melalui persamaan
berikut:
The relationship between length, l and period, T, of a simple pendulum is given by the following equation:

g
l= T2
4π2

Dengan menggunakan nilai T di 2(d)(i), hitungkan panjang bandul, l itu. OP3


Using the value of T in 2(d)(i), calculate the length of the pendulum, l.
[g = 10 m s–2]
gT 2
l=
4π2
10(1.2)2
=
4π2
= 0.36 m

[2 markah/ marks]

© Penerbit Mahir Sdn. Bhd. (183897-P) 10

Fizik Tg 4 (Bab 1)6th 9/2/23.indd 10 09/02/2023 4:19 PM


Fizik Tingkatan 4 Bab 2

BAB
2
Daya dan Gerakan I
Force and Motion I

Tema Mekanik Newton


Theme Newton Mechanics

NOTA EFEKTIF

1 Gerakan Linear/ Linear Motion

Jarak/ Distance, d Sesaran/ Displacement, s


Panjang lintasan yang dilalui oleh suatu objek yang bergerak Jarak dalam arah tertentu
The total path length travelled by an object Distance in a specified direction

Laju/ Speed, v Halaju/ Velocity, v


Kadar perubahan jarak/ The rate of change of distance Kadar perubahan sesaran/ The rate of change of
displacement
Jarak yang dilalui/ Distance travelled
Laju/ Speed = Sesaran/ Displacement
Masa diambil/ Time taken Halaju/ Velocity =
Masa diambil/ Time taken

Pecutan/ Acceleration, a Nyahpecutan/ Deceleration


Kadar perubahan halaju/ The rate of change of velocity. Kadar pengurangan halaju pada arah
Unit S.I./ S.I. unit: m s–2 tertentu.
Perubahan halaju/ Change in velocity The rate of decreased in speed in a specified
Pecutan/ Acceleration = direction.
Masa yang diambil/ Time taken
Halaju akhir/ Final velocity, v – Halaju awal/ Initial velocity, u
a=
Masa yang diambil/ Time taken, t
v–u
a=
t

2 Graf Gerakan Linear/ Linear Motion Graph

Jenis graf
Type of graph
Graf sesaran-masa Graf halaju-masa Graf pecutan-masa
Displacement-time graph Velocity-time graph Acceleration-time graph
Jenis gerakan
Type of motion

s v
Halaju sifar
Zero velocity
t t

s v a
Halaju seragam
Uniform velocity
t t t

s v a
Halaju negatif
t
Negative velocity
t t

11 © Penerbit Mahir Sdn. Bhd. (183897-P)

Fizik Tg 4 (Bab 2)5th 9/2/23.indd 11 09/02/2023 11:59 AM


Fizik Tingkatan 4 Bab 2

s v a
Pecutan seragam
Uniform acceleration
t t t

s v a
Nyahpecutan
seragam t
Uniform deceleration t t

Kecerunan = halaju Kecerunan = pecutan


Gradient = velocity Gradient = acceleration
Luas bawah graf =
sesaran
Area under graph
= displacement

3 Gerakan Jatuh Bebas/ Free Fall Motion


(a) Daya tarikan graviti ialah daya yang p = mv
menarik objek ke arah pusat Bumi.
Gravitational force is a force which pulls an object (b) Prinsip Keabadian Momentum:
towards the center of the Earth. Principle of Conservation of Momentum:
(b) Pecutan graviti, g ialah pecutan yang Jumlah momentum suatu sistem adalah
dialami oleh objek disebabkan tarikan daya kekal tidak berubah jika tiada daya
graviti. luar yang bertindak ke atas sistem itu.
Gravitational acceleration, g is the acceleration of The total momentum of a system remains
an object due to the pull of the gravitational force. unchanged if no external force acting on the
(c) Jatuh bebas ialah keadaan di mana objek system.
jatuh disebabkan daya tarikan graviti (c) Perlanggaran kenyal/ Elastic collision
sahaja. Jumlah momentum sebelum perlanggaran
Free fall is the situation where an object falls down = Jumlah momentum selepas perlanggaran
due to gravitational force only. Total momentum before collision =
4 Inersia/ Inertia Total momentum after collision
(a) Sifat suatu objek yang cenderung untuk
menentang sebarang perubahan keadaan m1u1 + m2u2 = m1v1 + m2v2
asalnya sama ada pegun atau bergerak
dalam satu garis lurus dengan halaju malar. (d) Letupan/ Explosion
The tendency of an object to remain at rest or to Jumlah momentum sebelum perlanggaran
continue its uniform motion in a straight line at = Jumlah momentum selepas perlanggaran
uniform velocity. Total momentum before collision =
(b) Hukum Gerakan Newton Pertama: Total momentum after collision
Newton’s First Law of motion: m1u1 + m2u2 = m1v1 + m2v2
Setiap objek akan kekal dalam keadaan 0 = m1v1 + m2v2
pegun atau bergerak dengan halaju malar m1v1 = – m2v2
jika tiada daya luar bertindak ke atasnya.
An object will remain at rest or move at uniform 6 Daya/ Force
velocity unless acted upon by an external force. Hukum Gerakan Newton Kedua:
(c) Semakin besar jisim, semakin besar inersia. Newton’s Second Law of Motion:
The larger the mass, the larger the inertia. Kadar perubahan momentum berkadar terus
5 Momentum/ Momentum dengan daya dan bertindak pada arah tindakan
(a) Momentum = Jisim × Halaju daya.
Momentum = Mass × Velocity The rate of change of momentum is directly
Unit S.I./ S.I. unit: kg m s–1 proportional to the force and acts in the direction of the
applied force.

© Penerbit Mahir Sdn. Bhd. (183897-P) 12

Fizik Tg 4 (Bab 2)5th 9/2/23.indd 12 09/02/2023 11:59 AM


Fizik Tingkatan 4 Bab 2

7 Impuls & Daya Impuls (c) Hukum Gerakan Newton Ketiga:


Impulse & Impulsive Force Newton’s Third Law of Motion:
(a) Impuls, Ft: Perubahan momentum Untuk setiap daya tindakan terdapat satu
Impulse, Ft: Change of momentum daya tindak balas yang sama magnitud
tetapi bertentangan arah.
Ft = mv – mu For every action there is a reaction of same as
Unit/ Unit: kg m s–1 atau/ or N s magnitude but in the opposite direction.
8 Berat/ Weight
(b) Daya impuls, F: Kadar perubahan momentum Berat = Jisim × Pecutan graviti
sewaktu perlanggaran atau letupan. Weight = Mass × Gravitational acceleration
Impulsive force, F: The rate of change of Unit S.I/ S.I unit: N
momentum in a collision or explosion.
W = mg
mv – mu
F=
t
Unit/ Unit: kg m s–2 atau/ or N

2.1 Gerakan Linear


Linear Motion

Buku Teks: m.s 26-36


1 Gerakan linear dijelaskan dengan jarak, sesaran, laju, halaju dan pecutan.
Linear motion can be described in terms of distance, displacement, speed, velocity and acceleration.

(a) Apakah definisi jarak? TP 1 OP2


What is the definition of distance?

Panjang lintasan yang dilalui oleh suatu objek yang bergerak.


The total path length travelled by an object.

(b) Apakah definisi sesaran? TP 1 OP2


What is the definition of displacement?

Jarak dalam suatu arah tertentu.


Distance in a specific direction.

(c) Seorang pengakap memulakan kembara dari stesen A ke stesen B yang terletak 400 m ke timur stesen A.
Apabila tiba di stesen B, dia diarahkan untuk ke stesen C yang terletak 400 m ke utara stesen B.
A scout starts his journey from station A to station B which is located 400 m to the east of station A. When he
reached station B, he is ordered to go to station C which is 400 m to the north of station B.

A B

13 © Penerbit Mahir Sdn. Bhd. (183897-P)

Fizik Tg 4 (Bab 2)5th 9/2/23.indd 13 09/02/2023 11:59 AM


Fizik Tingkatan 4 Bab 2
(i) Apabila pengakap itu di stesen C, berapakah jarak perjalanannya dari stesen A? TP 2 OP3
When the scout at station C, what is his distance travelled from station A?
d = 800 m

(ii) Apabila pengakap itu di stesen C, berapakah sesaran dari stesen A? Nyatakan magnitud dan arah.
When the scout at station C, what is his displacement from station A? State the magnitude and direction.
TP 2 OP3

s= (400)2 + (400)2
= 565.69 m pada arah AC / in the direction of AC

(iii) Mengapakah jawapan di (i) dan (ii) berbeza? TP 4 OP5


Why does the answers in (i) and (ii) different?
Kerana sesaran mempunyai magnitud dan arah

Because displacement has magnitude and direction

(d) Perbezaan antara laju dan halaju. TP 2 OP3


Difference between speed and velocity.

Laju Halaju
Speed Velocity

Kadar perubahan jarak (iii) Kadar perubahan sesaran


Rate of change in distance Rate of change in displacement

(i) Unit S.I: meter per saat (m s–1) Unit S.I: meter per saat (m s–1)
S.I unit: meter per second (m s )
–1
S.I unit: meter per second (m s–1)

Kuantiti skalar (iv) Kuantiti vektor


Scalar quantity Vector quantity

d s
(ii) v = v=
t t

(e) Apakah definisi dan rumus bagi pecutan? TP 1 OP2


What is the definition and formula of acceleration?

Kadar perubahan halaju / Rate of change in velocity.


Perubahan halaju Change in velocity
Pecutan = / Acceleration =
Masa Time
v–u
a=
t

© Penerbit Mahir Sdn. Bhd. (183897-P) 14

Fizik Tg 4 (Bab 2)5th 9/2/23.indd 14 09/02/2023 12:00 PM


SUPERB SBP FIZIK TINGKATAN 4 BATCH 2 ART
SUPERB SBP FIZIK TINGKATAN 4 BATCH 2 ART
Fizik Tingkatan 4 Bab 2
SUPERB SBP FIZIK TINGKATAN 4 BATCH 2 ART
(f) Nyatakan perubahan pada sesaran, halaju dan pecutan kereta bagi setiap rajah. TP 2 OP3
State the change in displacement, velocity and acceleration of the car for each diagram.

(a)

(b)
SB-16
SB-16
(c) SB-16

Rajah SB-15
(a) (b) (c)
Diagram SB-15
SB-15
Sesaran Sama Bertambah Berkurang
Displacement Same Increases Decreases

Halaju Sama Bertambah Berkurang


Velocity Same Increases Decreases

Pecutan Sifar Memecut Nyahpecut


Acceleration Zero Accelerates Decelerates

(g) Seorang penunggang basikal bermula dari keadaan rehat dan menambahkan halajunya pada kadar
seragam sehingga dia mencapai halaju 4.0 m s–1 dalam 5.0 s. Tentukan TP 2 OP3
A cyclist starts from rest and increases his velocity at a constant rate until he reaches a speed of 4.0 m s–1 in 5.0 s.
Determine

(ii) SB-18
4.0 – 0
(i) halaju awal = 0 halaju akhir = 4.0 m s–1 (iii) pecutan =
initial velocity SB-18
final velocity acceleration 5.0
= 0.8 m s–2
SB-18

2 Menentukan sesaran, halaju, pecutan dan masa.


To determine displacement, velocity, acceleration and time.

Sebuah pita detik dan jangka masa detik digunakan untuk menyiasat gerakan linear suatu objek. Rajah di
bawah menunjukkan bahagian pita detik yang ditarik oleh objek. Jangka masa detik dengan frekuensi 50 Hz
menghasilkan 50 titik dalam masa 1 saat. TP 4 OP5
A ticker timer and a ticker tape are used to investigate the linear motion of an object. The diagram below shows the part
of ticker tape that is pulled by the object. The ticker tape with frequency of 50 Hz makes 50 ticks in 1 second.
10 detik
10 ticks

A 8 cm B

15 © Penerbit Mahir Sdn. Bhd. (183897-P)

Fizik Tg 4 (Bab 2)5th 9/2/23.indd 15 09/02/2023 12:00 PM


Fizik Tingkatan 4 Bab 2
(a) Tentukan masa, t diambil dari A ke B.
Determine the time taken, t from A to B.
50 detik = 1 s → 1 detik = 0.02 s A ke B : 10 detik
50 ticks = 1 s → 1 tick = 0.02 s A to B : 10 ticks
t = 10 × 0.02
= 0.2 s

(b) Tentukan sesaran, s dari A ke B.


Determine the displacement, s from A to B.
s = 8 cm

(c) Tentukan halaju, v dari A ke B.


Determine the velocity, v from A to B.
8.0
v= = 40.0 cm s–1
0.2
(d) Rajah menunjukkan pita detik dengan 5 detik.
The diagram shows a ticker tape with 5 ticks.

Arah gerakan
Direction of motion

0.2 cm 1.4 cm
Hitung pecutan pita detik itu.
Calculate the acceleration of the ticker tape.
0.2 1.4
Halaju awal, u = = 10.0 cm s–1 Halaju akhir, v = = 60.0 cm s–1
Initial velocity
0.02 Final velocity
0.02
v – u 60.0 – 10.0
Masa, t = (5 – 1) × 0.02 = 0.08 s Pecutan, a = =
Time Acceleration
t 0.08
= 625 cm s–2
⸫ a = 625 cm s–2
3 Menerbitkan persamaan gerakan linear. TP 3 OP4
To derive the linear motion equations.

(a) Persamaan pertama (c) Persamaan kedua


First equation Second equation
v–u v+u
a= s=( )×t
t 2
at = v – u
1
s= (v + u)t
v = u + at 2

(b) Persamaan ketiga (d) Persamaan keempat


Third equation Fourth equation
1 v2 = (u + at)2
s = [u + (u + at)]t v2 = u2 + 2uat + a2t2
2
1 1
s = (2u + at)] t v2 = u2 + 2a(ut+ at2)
2 2
1 2 v2 = u2 + 2as
s = ut + at
2

© Penerbit Mahir Sdn. Bhd. (183897-P) 16

Fizik Tg 4 (Bab 2)5th 9/2/23.indd 16 09/02/2023 12:00 PM


Fizik Tingkatan 4 Bab 2
4 Menyelesaikan masalah melibatkan persamaan gerakan linear.
Solving problem related to linear motion equation.

(a) Seorang budak lelaki mengayuh basikal menuruni sebuah bukit. Halaju awalnya ialah 4 m s–1. Selepas
dia bergerak 35 m, halajunya menjadi 10 m s–1. TP 4 OP5
A boy is cycling down a hill. His initial velocity is 4 m s–1. After he moves 35 m, his velocity becomes 10 m s–1.

(i) Berapakah masa diambil untuk budak itu mengayuh sejauh 35 m?


What is the time taken for the boy to cycle at a distance 35 m?
u = 4 m s–1, v = 10 m s–1, s = 35 m

1
s= (v + u)t
2
1
35 = (10 + 4)t
2
35
t=
7
=5s

(ii) Berapakah pecutannya?


What is his acceleration?
v2 = u2 + 2as
102 = 42 + 2a(35)
70a = 84
a = 1.2 m s–2

(b) Sebuah kereta bergerak dengan halaju 20 m s–1 sepanjang jalan lurus. Pemandu itu menekan brek selama
5 s dan menyebabkan nyahpecutan 3 m s–2.
Berapakah halaju akhir kereta itu? TP 4 OP5
A car is traveling at 20 m s-1 along a straight road. The driver puts the brakes on for 5 s and causes a deceleration of
3 m s–2.
What is the car’s final velocity?
u = 20 m s–1, t = 5 s, a = 3 m s–2

v = u + at
= 20 + 3(5)
= 35 m s–1

(c) Sebuah kereta memecut dari keadaan rehat pada 3 m s–2 di sepanjang suatu jalan lurus.
Berapakah sesaran yang dilalui oleh kereta itu selepas 4 s? TP 4 OP5
A car accelerates from rest at 3 m s–2 along a straight road.
What is the displacement of the car travelled after 4 s?
u = 0, a = 3 m s–2, = 4 s,

1 2
s = ut + at
2
1
= 0(4) + (3) (4)2
2
= 24 m

17 © Penerbit Mahir Sdn. Bhd. (183897-P)

Fizik Tg 4 (Bab 2)5th 9/2/23.indd 17 09/02/2023 12:00 PM


Fizik Tingkatan 4 Bab 2

2.2 Graf Gerakan Linear


Linear Motion Graph

Buku Teks: m.s 37-45


1 Rajah menunjukkan graf sesaran-masa bagi suatu objek.
The diagram shows a graph of displacement-time for an object.
Sesaran / m
Displacement

B C
20

A D
Masa / s
0 10 30 35 Time

(a) Hitung halaju objek di TP 3 OP4


Calculate the velocity of the object at
(i) AB. (ii) BC. (iii) CD.
20 – 20 – 0
v= v=0 v=
10 35 – 30
= 2 m s–1 = – 4 m s–1
(b) Huraikan gerakan objek di TP 3 OP4
Describe the motion of the object at

(i) AB : Halaju malar/ Constant velocity

(ii) BC : Tidak bergerak/ At rest

(iii) CD : Halaju malar pada arah bertentangan/ Constant velocity in opposite direction

(c) (i) Hitung jumlah jarak pergerakan objek. TP 4 OP5


Calculate the total distance of the object motion.
20 + 0 + 20 = 40 m

(ii) Hitung jumlah sesaran pergerakan objek. TP 4 OP5


Calculate the total displacement of the object motion.
20 + 0 + (– 20) = 0

(d) (i) Cari laju purata objek itu. TP 4 OP5


Find the average speed of the object.
40
= 1.14 m s–1
35
(ii) Cari halaju purata objek itu. TP 4 OP5
Find the average velocity of the object.
0

© Penerbit Mahir Sdn. Bhd. (183897-P) 18

Fizik Tg 4 (Bab 2)5th 9/2/23.indd 18 09/02/2023 12:00 PM


Fizik Tingkatan 4 Bab 2
2 Rajah menunjukkan graf halaju-masa bagi sebuah kereta.
The diagram shows a graph of displacement-time for a car.
Halaju / m s–1
Velocity

K
20

L M
10

J O
Masa/ s
0 10 20 30 35 40 45
Time

–10
N
(a) Hitung pecutan kereta di TP 3 OP4
Calculate the acceleration of the car at

(i) JK. (ii) KL.


20 –(20 – 10)
a= a=
10 20 –10
= 2 m s–2 = –1 m s–2

(iii) LM. (iv) MN.


–(10 + 10)
a=0 a=
40 – 30
= –2 m s–2
(b) Huraikan gerakan kereta di TP 3 OP4
Describe the motion of the car at

(i) JK : Pecutan seragam/ Uniform acceleration

(ii) KL : Nyahpecutan seragam/ Uniform deceleration

(iii) LM : Halaju seragam/ Uniform velocity

(vi) MN : Nyahpecutan seragam/ Uniform deceleration

(c) (i) Hitung jumlah jarak perjalanan kereta. TP 4 OP5


Calculate the total distance travelled by the car.
Jarak = Luas di bawah graf, A
Distance = Area under the graph, A
1 1
A1 = (20 × 10) A2 = (10 + 20)10
2 2
= 100 m = 150 m
1 1
A3 = (10 + 15)10 A4 = (10 × 10)
2 2
= 125 m = 50 m
Jumlah jarak = 100 + 150 + 125 + 50 = 425 m
Total distance

(ii) Hitung jumlah sesaran kereta. TP 4 OP5


Calculate the total displacement of the car.
Jumlah sesaran = 100 + 150 + 125 – 50 = 325 m
Total displacement

19 © Penerbit Mahir Sdn. Bhd. (183897-P)

Fizik Tg 4 (Bab 2)5th 9/2/23.indd 19 09/02/2023 12:00 PM


Fizik Tingkatan 4 Bab 2
3 Terjemahkan graf berikut untuk melakarkan graf yang dinyatakan. TP 4 OP5
Interpret the following graphs to sketch the stated graph.

(a) Graf halaju-masa Graf pecutan-masa


Graph of velocity-time Graph of acceleration-time
Halaju Pecutan
Velocity Acceleration

Masa
0 T 2T Time
Masa
0 T 2T Time

(b) Graf pecutan -masa Graf halaju-masa


Graph of acceleration-time Graph of velocity-time
Pecutan Halaju
Acceleration Velocity

0 Masa 0 Masa
T 2T T 2T Time
Time

2.3 Gerakan Jatuh Bebas


Free Fall Motion

Buku Teks: m.s 46-51

INFO
Sebiji epal jatuh ke tanah dari pokoknya. Isaac Newton menyatakan bahawa semua objek ditarik
oleh satu daya yang menyebabkan objek jatuh ke permukaan Bumi. Suatu objek dikatakan
mengalami gerakan jatuh bebas jika pergerakan objek itu dipengaruhi oleh daya graviti sahaja.
An apple fall on the ground from a tree. Isaac Newton said all objects are pulled by a force which causes them
to fall to the surface of the Earth. An object experiences free fall if the motion of the object is affected only by
gravitational force.

1 Apakah maksud jatuh bebas? TP 1 OP2


What is the meaning of free fall?

Keadaan di mana objek jatuh disebabkan oleh daya tarikan graviti sahaja.
An object is falling freely when it is falling under the gravitational force only.

© Penerbit Mahir Sdn. Bhd. (183897-P) 20

Fizik Tg 4 (Bab 2)5th 9/2/23.indd 20 09/02/2023 12:00 PM


Fizik Tingkatan 4 Bab 2
2 Rajah 1 menunjukkan sebiji bola golf dan sehelai kertas dipegang pada ketinggian yang sama dan dijatuhkan
serentak.
Diagram 1 shows a golf ball and a piece of paper are held at the same height and drop them simultaneously.

Bola golf Kertas Kertas renyuk


Golf ball Paper Crumpled
paper

Tanah Tanah
Ground Ground
Rajah 1 Rajah 2
Diagram 1 Diagram 2

(a) Objek yang manakah mencecah tanah dahulu? TP 2 OP3


Which object reaches the ground first?
Bola golf/ Golf ball

(b) Jelaskan alasan bagi jawapan di 2(a). TP 3 OP4


Explain a reason for the answer in 2(a).
Luas permukaan kertas lebih besar, maka rintangan udara lebih besar.

The surface area of the paper larger, then the air resistance larger.

(c) Aktiviti diulangi dengan bola golf yang sama dan sehelai kertas yang direnyukkan seperti dalam Rajah 2.
The activity is repeated with the same golf ball and a piece of paper which is crumpled as in Diagram 2.

Objek yang manakah mencecah tanah dahulu? TP 3 OP4


Which object reaches the ground first?
Kedua-duanya/ Both objects

(d) Jelaskan alasan bagi jawapan di 2(c). TP 4 OP5


Explain a reason for the answer in 2(c).
Luas permukaan kertas renyuk menjadi kecil, maka rintangan udara turut kecil.

The surface area of the paper become smaller, then the air resistance also small.

EKSPERIMEN 2.1

Tujuan: Menentukan nilai pecutan graviti.


To determine the value of gravitational acceleration.
Aim:

Radas: Jangka masa detik, pita detik, bekalan kuasa 12 V (a.u), pemberat berslot, pengapit-G dan bangku
Ticker timer, ticker tape, 12 V (a.c) of power supply, slotted weight, G-clamp and stool
Apparatus:

21 © Penerbit Mahir Sdn. Bhd. (183897-P)

Fizik Tg 4 (Bab 2)5th 9/2/23.indd 21 09/02/2023 12:00 PM


Fizik Tingkatan 4 Bab 2
Prosedur/ Procedure:

Pengapit – G
G – clamp
Jangka masa detik
Ticker timer

Bekalan kuasa
Power supply

Bangku
Stool

Pemberat berslot
Sotted weight

(i) Potong sehelai pita detik dan lalukan melalui jangka masa detik yang diapit ke bangku
menggunakan pengapit-G.
Cut a piece of ticker tape and pass through the ticker timer which is clamped to a stool using G-clamp.
(ii) Lekatkan satu hujung pita detik pada pemberat berslot 100 g.
Attach one end of the tape to the 100 g of slotted weight.
(iii) Hidupkan suis dan lepaskan pemberat berslot supaya ia jatuh bebas.
Switch on the ticker timer and release the slotted weight so that it falls freely.
(iv) Analisa pita detik untuk menentukan nilai pecutan graviti, g.
Analyse the ticker tape to determine the value of the acceleration due to gravity, g.

Perbincangan/ Discussion:
(i) Apakah jenis pergerakan objek jika jatuh di bawah tarikan graviti?
What is the type of motion of the objects as they fall under the pull of gravity?
Jatuh bebas/ Free fall

(ii) Bandingkan nilai pecutan graviti daripada eksperimen dengan nilai yang sebenar. Berikan alasan.
Compare the value of gravitational acceleration from the experiment with the actual value. Give a reason.
Nilai daripada eksperimen lebih rendah daripada nilai sebenar kerana terdapat rintangan udara.

The value from the experiment is lower than the actual value because there is air resistance.

(iii) Nilai g berubah dari satu tempat ke tempat yang lain. Sebagai contoh, nilai g di Khatulistiwa ialah
9.78 m s–2 manakala g di kutub Bumi ialah 9.83 m s–2. Mengapa?
The value of gravitational acceleration, g changes from one place to another. For example, the value of at the
equator is 9.78 m s–2 while the value of at the Earth’s poles is 9.83 m s–2. Why?
Hal ini kerana Bumi sebenarnya bukan berbentuk sfera yang sempurna. Jarak dari Khatulistiwa ke

pusat Bumi lebih jauh daripada jarak dari kutub ke pusat Bumi. Nilai g lebih kecil di Khatulistiwa

daripada di kutub Bumi./ This is because Earth is actually not a perfect sphere. The distance from the Equator
to the center of the Earth is further than the distance from the poles to the center of the Earth. The value of g is

smaller at the equator than at the Earth’s poles.

© Penerbit Mahir Sdn. Bhd. (183897-P) 22

Fizik Tg 4 (Bab 2)5th 9/2/23.indd 22 10/02/2023 10:04 AM


Fizik Tingkatan 4 Bab 2
3 Bagi objek yang jatuh dengan pecutan, g, berikut adalah persamaan yang berkaitan:
For an object falling with acceleration g, the following equations apply:

1
v = u + gt s = ut + gt v2 = u2 + 2gs
2

Objek dilontar ke atas/ The object is thrown upward: a = g = –9.81 m s–2

4 Amir menjatuhkan seketul batu ke dalam perigi. Nilai pecutan graviti adalah 9.81 m s–2. Jika jarak di antara
bahagian atas perigi dengan permukaan air ialah 20 m,
Amir releases a stone into a well. The gravitational acceleration is 9.81 m s–2. If the distance between the top of the well and
the water surface is 20 m, [g = 9.81 m s–2]

(a) berapakah masa yang diambil oleh batu itu untuk sampai ke permukaan air? TP 3 OP4
what is the time taken for the stone to reach the surface of the water?
a = g = 9.81 m s–2
1 2
s = ut + gt
2
1
20 = 0 + (9.81)(t2)
2
20
t =
2
4.9
t = 2.02 s
(b) berapakah halaju batu itu apabila sampai di permukaan air? TP 3 OP4
what is the velocity of the stone when it reaches the surface of the water?
a = g = 9.81 m s–2
v = u + gt
= 0 + 9.81(2)
= 19.6 m s–1

5 Amirah melontar bola ke atas secara menegak dengan halaju awal 10 m s–1. Hitung tinggi maksimum yang
boleh dicapai oleh bola itu. TP 3 OP4
[Abaikan rintangan udara dan nilai pecutan graviti adalah 9.81 m s–2]
Amirah threw a ball vertically upwards with an initial velocity of 10 m s–1. Calculate the maximum height that can be
reached by the ball.
[Ignore the air resistance and the value of gravitational acceleration 9.81 m s–2]
a = g = –9.81 m s–2
v2 = u2 + 2gs
0 = 102 + 2(–9.81)s
s = 5.10 m

2.4 Inersia
Inertia

Buku Teks: m.s 52-57


1 Apakah maksud inersia? TP 1 OP2
What is the meaning of inertia?

Sifat suatu objek yang cenderung untuk menentang sebarang perubahan keadaan asalnya sama ada
keadaan rehat atau keadaan gerakan dalam satu garis lurus.
The tendency of the object to remain at rest or, if moving, to continue its uniform motion in a straight line.

23 © Penerbit Mahir Sdn. Bhd. (183897-P)

Fizik Tg 4 (Bab 2)5th 9/2/23.indd 23 09/02/2023 12:00 PM


Fizik Tingkatan 4 Bab 2

2 Hukum Gerakan Newton Pertama menyatakan bahawa sesuatu objek akan kekal dalam keadaan rehat atau
bergerak dengan halaju malar jika tiada daya luar bertindak ke atasnya.
every object continues in its state of rest, or of uniform motion in a straight line,
Newton’s First Law of Motion states that
unless it is acted on by an external force.

3 Rajah menunjukkan sekeping kadbod di atas sebuah gelas. Sekeping duit syiling diletakkan di atas kadbod.
The diagram shows a cardboard on a glass. A coin is placed on the cardboard.
Syiling
Coin

Kadbod
Cardboard

(a) Apakah yang berlaku kepada duit syiling itu apabila kadbod ditarik dengan cepat? TP 3 OP4
What happens to the coin when the cardboard is pulled quickly?
Duit syiling jatuh terus ke dalam gelas.

The coin drops straight into the glass.

(b) Jelaskan pemerhatian di 3(a). TP 3 OP4


Explain the observation in 3(a).
Inersia duit syiling kekal pegun. Duit syiling jatuh ke dalam gelas disebabkan daya tarikan graviti.

The inertia of the coin maintains its state at rest. The coin falls into the glass due to gravitational force.

4 Hubungan antara inersia dan jisim.


Relationship between inertia and mass.

EKSPERIMEN 2.2
Rajah 2.1 menunjukkan seorang penarik beca membawa seorang penumpang. Rajah 2.2 menunjukkan
penarik beca yang sama membawa dua orang penumpang. Beliau mendapati lebih sukar untuk
menggerakkan dan memberhentikan beca apabila membawa dua orang penumpang.
Diagram 2.1 shows a trishaw rider carrying a passenger. Diagram 2.2 shows the same trishaw rider carrying two
passengers. He found that when carrying two passengers is more difficult to move and stop the trishaw.

Rajah 2.1 Rajah 2.2


Diagram 2.1 Diagram 2.2

© Penerbit Mahir Sdn. Bhd. (183897-P) 24

Fizik Tg 4 (Bab 2)5th 9/2/23.indd 24 09/02/2023 12:00 PM


Fizik Tingkatan 4 Bab 2

Inferens: Inersia objek bergantung kepada jisim objek itu


The inertia of an object depends on the mass of the object
Inference:

Hipotesis: Semakin bertambah jisim objek, semakin bertambah tempoh ayunan.


The larger the mass of object, the longer the oscillation period.
Hypothesis:

Tujuan: Mengkaji hubungan antara inersia dengan jisim objek.


To study the relationship between inertia and mass of object.
Aim:

Pemboleh ubah dimanipulasi: Jisim objek


Mass of object
Manipulated variable:

Pemboleh ubah bergerak balas: Tempoh ayunan


Oscillation period
Responding variable:

Pemboleh ubah dimalarkan: Panjang bilah gergaji


Length of hacksaw blade
Constant variable:

Radas: Bilah gergaji, pengapit-G, jam randik, plastisin, neraca tiga alur
Hacksaw blade, G-clamp, stopwatch, plasticine balls, triple beam balance
Apparatus:

Prosedur/ Procedure:
(i) Letakkan sejumlah plastisin dengan jisim 30 g pada hujung bilah gergaji.
Place a lump of plasticine with a mass of 30 g at the free end of the hacksaw blade.
(ii) Sesarkan sedikit bilah gergaji dan lepaskannya supaya ia berayun secara mengufuk.
Displace the hacksaw blade and release it so that it oscillates.
(iii) Tentukan dan rekodkan masa yang diambil untuk 10 ayunan lengkap, t saat. Hitungkan tempoh
ayunan, T = t/10 saat.
Determine and record the time taken for 10 complete oscillations, t. Calculate the oscillation period,
T = t/10 seconds.
(iv) Ulangi eksperimen dengan jisim plastisin 40 g, 50 g, 60 g dan 70 g.
Repeat the experiment with mass of plasticine of 40 g, 50 g, 60 g and 70 g.

Pengapit – G
G – clamp

Bilah gergaji
Hacksaw blade

Plastisin
Plasticine

25 © Penerbit Mahir Sdn. Bhd. (183897-P)

Fizik Tg 4 (Bab 2)5th 9/2/23.indd 25 09/02/2023 12:00 PM


Fizik Tingkatan 4 Bab 2
Keputusan/ Result:
(i) Jadualkan data bagi m, t, T dan T2.
Tabulate data for m, t, T and T2.

Masa untuk 10 ayunan lengkap


Jisim plastisin Time for 10 complete oscillations, t/ s
Mass of plasticine
T/ s T2/ s2
m/ g
t1 t2 tpurata/average

30 4.0 4.2 4.1 0.41 0.17

40 5.0 5.1 5.05 0.50 0.25

50 6.2 6.3 6.25 0.63 0.39

60 7.0 7.1 7.05 0.71 0.48

70 8.1 7.8 7.45 0.75 0.63

(ii) Lakarkan graf tempoh ayunan melawan jisim.


Sketch the graph of oscillation period against mass.
T 2/ s2

0 m/ g

Perbincangan/ Discussion:
(i) Namakan kuantiti fizik yang mewakili inersia.
Name the physical quantity represented by inertia.
Tempoh ayunan/ Oscillation period

(ii) Berdasarkan graf, nyatakan hubungan antara tempoh ayunan dan jisim objek.
Based on the graph, state the relationship between oscillation period and mass of object.
Semakin bertambah jisim objek, semakin lama tempoh ayunan

The higher the mass of object, the longer the oscillation period

(iii) Berdasarkan graf, nyatakan hubungan antara inersia dan jisim objek.
Based on the graph, state the relationship between the inertia and mass of object.
Semakin bertambah jisim objek, semakin besar inersia objek

The higher the mass of object, the larger the inertia of object

© Penerbit Mahir Sdn. Bhd. (183897-P) 26

Fizik Tg 4 (Bab 2)5th 9/2/23.indd 26 09/02/2023 12:00 PM


Fizik Tingkatan 4 Bab 2
5 Inersia dalam kehidupan harian. TP 5 OP6
Inertia in daily life.

(a) Mengapakah sos tomato lebih mudah dikeluarkan daripada botol jika botol digerakkan dengan laju dan
berhenti secara mendadak?
Why the tomato sauce can be easily poured out if the bottle is moved down fast with a sudden stop?
Sos tomato bergerak bersama-sama dengan botol. Apabila botol dihentikan secara mendadak, inersia

sos terus bergerak dan keluar daripada botol.

The sauce moves together with the bottle. When the bottle stops suddenly, the inertia of the sauce continues to move

and poured out of the bottle.

(b) Terangkan menggunakan konsep inersia untuk mengetatkan kepala penukul dengan menghentak
pemegangnya pada permukaan keras.
Explain by using the concept of inertia to tight in the head of a hammer by knocking the handle on a hard surface.
Kepala penukul bergerak bersama pemegang ke bawah. Apabila pemegang diberhentikan tiba-tiba,

inersia kepala penukul ingin kekal bergerak ke depan. Oleh itu, hujung pemegang masuk ke dalam

kepala penukul.

The head of hammer is moving together with the handle as it moves down. When the handle is stopped suddenly,

the inertia of the head of hammer still want to move forward. So, the top end of the handle will slotted deeper into

the head of the hammer.

(c) Payung yang basah boleh dikeringkan dengan menggerakkan payung dengan cepat dan kemudian
diberhentikan dengan tiba-tiba. Terangkan.
Wet umbrella can be dried by moving it very fast and then stop suddenly. Explain.
Air hujan dan payung bergerak bersama-sama. Apabila payung diberhentikan dengan tiba-tiba, inersia

air hujan ingin terus bergerak dan meninggalkan payung.

The water droplet move together with the umbrella. When the umbrella is stopped suddenly, the inertia of the

droplet continues its state of motion and left the umbrella.

2.5 Momentum
Momentum

Buku Teks: m.s 58-63


1 Apakah maksud momentum? TP 1 OP2
What is the meaning of momentum?

Momentum ialah jisim × halaju


Momentum is mass × velocity

27 © Penerbit Mahir Sdn. Bhd. (183897-P)

Fizik Tg 4 (Bab 2)5th 9/2/23.indd 27 09/02/2023 12:00 PM


Fizik Tingkatan 4 Bab 2
2 Momentum dapat dihitung dengan rumus:
Momentum can be calculated by formula:

di mana/ where
p = mv p : Momentum objek/ Momentum of object
m: Jisim objek/ Mass of object
v : Halaju objek/ Velocity of object

3 Unit S.I bagi momentum adalah kg m s–1 .


kg m s–1
The S.I unit for the momentum is .

4 Apakah prinsip keabadian momentum? TP 1 OP2


What is the principle of conservation of momentum?

Jumlah momentum sebelum perlanggaran sama dengan jumlah momentum selepas perlanggaran jika
tiada daya luar yang bertindak ke atas sistem itu.
Total momentum before collision same as total momentum after collision if no external force acts on the system.

5 Perlanggaran kenyal dan perlanggaran tak kenyal.


Elastic collision and inelastic collision.

Perlanggaran kenyal Perlanggaran tak kenyal


Elastic collision Inelastic collision
Jenis
u1 u2 v1 v2 u1 u2 v
perlanggaran
Type of collision
m1 m2 m1 m2 m1 m2 m1 m2

Keadaan
selepas Bergerak berasingan dengan halaju Bergerak bersama dengan halaju yang
perlanggaran berbeza sama
State after Move separately with different velocity Move together with same velocity
collision

Keabadian
momentum Diabadikan Diabadikan
Conservation of Conserved Conserved
momentum

Keabadian
tenaga Diabadikan Diabadikan
Conservation of Conserved Conserved
energy

Keabadian
tenaga kinetik Diabadikan Tidak diabadikan
Conservation of Conserved Not conserved
kinetic energy

Persamaan m1 u1 + m2 u2 = m1 v1 + m2 v2 m1 u1 + m2 u2 = (m1 + m2)v


Equation

© Penerbit Mahir Sdn. Bhd. (183897-P) 28

Fizik Tg 4 (Bab 2)5th 9/2/23.indd 28 09/02/2023 12:00 PM


Fizik Tingkatan 4 Bab 2
6 Letupan
Explosion
Troli pegun
Stationary trolleys
Pin
–v2 v1

m2 m1 m2 m1

Sebelum letupan Selepas letupan


Before explosion After explosion

Daripada prinsip keabadian momentum:


From the principle of conservation of momentum:

Jumlah momentum sebelum letupan = Jumlah momentum selepas letupan


Total momentum before explosion = Total momentum after explosion

0 = m1v1 + m2v2
m1v1 = – m2v2

7 (a) Dalam permainan bola sepak, seorang pemain A berjisim 70 g bergerak dengan halaju 4 m s–1 dan
seorang pemain B yang berjisim 75 kg bergerak dengan halaju 3 m s–1 pada arah bertentangan.
Hitungkan momentum kedua-dua pemain itu. TP 3 OP4
In a football game, a player A of mass 70 kg is moving with velocity of 4 m s-1 and a player B of mass 75 kg is
moving with velocity of 3 m s–1 in opposite direction.
Calculate the momentum of both players.
Pemain/ Player A:
pA = mv
= 70 × 4
= 280 kg m s–1

Pemain/ Player B:
pB = mv
= 75 × (–3)
= –225 kg m s–1

(b) Kereta A yang berjisim 1 000 kg bergerak pada 20 m s–1 berlanggar dengan kereta B yang berjisim
1 200 kg dan bergerak pada 10 m s–1 dalam arah yang sama. Akibatnya, kereta B, bergerak ke depan
pada halaju 15 m s–1.
Berapakah halaju, v, bagi kereta A sebaik sahaja selepas perlanggaran? TP 3 OP4
Car A of mass 1 000 kg moving at 20 m s–1 collides with a car B of mass 1 200 kg moving at 10 m s–1 in
same direction. The car B is shunted forwards at velocity of 15 m s–1 due to the impact.
What is the velocity of the car A immediately after the crash?
Perlanggaran kenyal/ Elastic collision
m1 u1 + m2 u2 = m1 v1 + m2 v2
(1 000)(20) + (1 200)(10) = (1 000)v1 + (1 200)(15)
1 000v1 = 14 000
v1 = 14 m s–1

29 © Penerbit Mahir Sdn. Bhd. (183897-P)

Fizik Tg 4 (Bab 2)5th 9/2/23.indd 29 09/02/2023 12:00 PM


Fizik Tingkatan 4 Bab 2
(c) Sebiji bola yang berjisim 15 kg dibalingkan pada halaju 20 km j–1 kepada Meilin yang berjisim 60 kg pada
keadaan rehat di atas ais. Dia menangkap bola itu dan kemudian menggelongsor dengan bola di atas ais.
Tentukan halaju Meilin dengan bola selepas perlanggaran. TP 3 OP4
A 15 kg ball is thrown at a velocity of 20 km h–1 to Meilin with mass of 60 kg who is at rest on ice. She catches the
ball and subsequently slides with the ball across the ice.
Determine the velocity of Meilin with the ball after the collision.
Perlanggaran tak kenyal/ Inelastic collision
m1u1 + m2u2 = (m1 + m2 )v
(15)(20) + 0 = (15 + 60)v
75v = 300
v = 4 m s–1

(d) Seorang lelaki menembak sepucuk pistol yang berjisim 1.5 kg. Jika peluru itu berjisim 10 g dan
mempunyai halaju 300 m s–1 selepas tembakan, berapakah halaju sentakan pistol itu? TP 3 OP4
A man fires a pistol which has a mass of 1.5 kg. If the mass of the bullet is 10 g and it reaches a velocity of
300 m s–1 after shooting, what is the recoil velocity of the pistol?

v 300 m s–1
Peluru
Pistol 10 g Bullet
Pistol 1.5 kg

Jumlah momentum sebelum letupan = jumlah momentum selepas letupan


Total momentum before explosion = total momentum after explosion
0 = momentum peluru + momentum pistol
0 = momentum of bullet + momentum of pistol
0 = m1v1 + m2v2
0 = (0.01)(300) + (1.5)v2
1.5v2 = – 3.0
v2 = – 2.0 m s–1
(Pistol tersentak ke belakang/ Pistol recoiled backwards)

2.6 Daya
Force

Buku Teks: m.s 64-66


1 Hubungan antara daya jisim dan pecutan. INFO
The relationship between force mass and acceleration.
Perubahan momentum
Change in momentum
mv – mu mv – mu
F ∝ ma F∝
t
Kadar perubahan momentum
Rate change in momentum
mv – mu
t

2 Hukum Gerakan Newton Kedua menyatakan bahawa kadar perubahan momentum berkadar terus dengan daya
dan bertindak pada arah tindakan daya.
the rate of change of momentum is directly proportional to the force and acts
Newton’s Second Law of Motion states that
in the direction of the applied force.

© Penerbit Mahir Sdn. Bhd. (183897-P) 30

Fizik Tg 4 (Bab 2)5th 9/2/23.indd 30 09/02/2023 12:00 PM


Fizik Tingkatan 4 Bab 2
3 Persamaan bagi Hukum Gerakan Newton kedua adalah
The equation for Newton’s Second Law of Motion is

di mana/ where
F = ma
F : Daya/ Force
m: Jisim objek/ Mass of object
a : Pecutan objek/ Acceleration of object

4 Seorang lelaki menolak troli yang berisi kotak dengan jisim 5 kg di atas permukaan licin. Jika dia
menggunakan daya 30 N untuk menolak troli itu, apakah magnitud dan arah pecutan troli itu?
A man pushes a trolley with 5 kg box on a smooth surface. If he uses a force of 30 N to move the trolley, what is the
magnitude and direction of the acceleration of the trolley?

F = 30 N

Kotak
5 kg Box

Troli
Trolley
F = ma
F 30
a= =
m 5
a = 6 m s–2 (ke kanan/ to the right)

5 Sebuah bas berjisim 2 000 kg bergerak dengan halaju seragam 40 m s–1 sejauh 2 500 m sebelum ia berhenti.
Hitungkan TP 3 OP4
A bus of mass 2 000 kg travels with a uniform velocity 40 m s–1 for a distance 2 500 m before it comes to rest.
Calculate

(a) pecutan purata bas.


the average acceleration of the bus.
v2 = u2 + 2as
0 = (40)2 + 2a(2 500)
5 000a = –1 600
a = – 0.32 m s–2 (nyahpecutan/ deceleration)

(b) daya purata brek untuk memberhentikan bas.


the average force applied by the brakes to stop the bus.
F = ma
= 2 000 × (– 0.32)
= – 640 N

2.7 Impuls dan Daya Impuls


Impulse and Impulsive Force

Buku Teks: m.s 67-69


1 Apakah maksud impuls? 1
TP OP2
What is the meaning of impulse?

Perubahan momentum
Change in momentum

31 © Penerbit Mahir Sdn. Bhd. (183897-P)

Fizik Tg 4 (Bab 2)5th 9/2/23.indd 31 09/02/2023 12:00 PM


Fizik Tingkatan 4 Bab 2
2 Persamaan untuk impuls adalah
The equation for impulse is

J = mv – mu di mana/ where
= Ft J: Impuls/ Impulse
m: Jisim/ Mass
u: Halaju awal/ Initial velocity
v: Halaju akhir/ Final velocity
F: Daya/ Force
t: Masa impak/ Impact time

3 Apakah maksud daya impuls? TP 1 OP2


What is the meaning of impulsive force?

Kadar perubahan momentum semasa perlanggaran atau hentaman dalam masa yang singkat.
The rate of change of momentum in a collision or impact in a short period of time.

4 Persamaan untuk daya impuls adalah


The equation for impulsive force is

mv – mu di mana/ where
F= F: Daya impuls/ Impulsive force
t
mv – mu: Perubahan momentum/ Change in momentum
t: Masa impak/ Impact time

5 Hukum Gerakan Newton Ketiga menyatakan bahawa untuk setiap daya tindakan terdapat satu daya tindak
balas yang sama magnitud tetapi bertentangan arah.
for every action there is a reaction of equal magnitude, but in the opposite
Newton’s Third Law of Motion states that
direction.

Daya tindak Daya tindak


balas balas
Daya tindakan Reaction Reaction
Daya tindakan
Action
Action

6 Terangkan mengapa pemain bola sepak melemparkan bola dengan cara seperti dalam rajah diberi.
Explain why the football player throws the ball as shown in the given diagram. TP 4 OP5

Masa impak lama, impuls besar./ Longer impact time, large impulse.
Perubahan momentum lebih besar./ Larger change in momentum.

Bola bergerak dengan halaju tinggi dan bergerak jauh./ Ball moves at higher velocity
and moves further.

© Penerbit Mahir Sdn. Bhd. (183897-P) 32

Fizik Tg 4 (Bab 2)5th 9/2/23.indd 32 09/02/2023 12:00 PM


Fizik Tingkatan 4 Bab 2
7 (a) Menambah momentum melalui tindakan ikut lajak
Increasing the magnitude of impulse by follow through action

Daya tindak Daya tindak


balas ke atas kaki balas ke atas bola Tindakan ikut lajak
Reaction on foot Reaction on ball Follow through action

(i) Sepakan yang kuat diikuti tindakan ikut lajak menghasilkan impuls yang besar .
large
A follow through action after a strong kick produces a impulse.

(ii) Daya besar dan masa impak lebih lama .


longer
Large force and impact time.

(iii) Bola mengalami perubahan momentum besar dan bergerak dengan halaju tinggi .
high
The ball experiences a large change in momentum and moves at a velocity.

(b) Mengurangkan daya impuls dengan memanjangkan masa impak


Reducing the magnitude of impulse by extending impact time

Situasi Penerangan
Situation Explanation

Zon remuk kereta • Perlanggaran menyebabkan kereta dihentikan dan mengalami


Crumple zone of a car suatu perubahan momentum.
Collision causes a car to stop and experience a change in momentum.
• Bahagian hadapan kereta yang mudah remuk memanjangkan
masa impak semasa perlanggaran.
The crumple zone of the car extends the impact time during collision.
• Daya impuls ke atas kereta dikurangkan.
The impulsive force on the car is reduced.

(c) Meningkatkan daya impuls dengan memendekkan masa impak


Increasing the magnitude of impulse by reducing impact time

Situasi Penerangan
Situation Explanation

Penggunaan alu dan lesung • Alu bergerak ke lesung yang keras permukaannya.
The use of pestle and mortar The pestle moving at high speed to a hard mortar.
• Alu berhenti dalam masa yang singkat.
The pestle to be stopped in a very short time.
• Daya impuls besar dihasilkan menyebabkan cili mudah
dihancurkan.
A large impulsive force is produced and causes these spices to be
crushed easily.

33 © Penerbit Mahir Sdn. Bhd. (183897-P)

Fizik Tg 4 (Bab 2)5th 9/2/23.indd 33 09/02/2023 12:00 PM


Fizik Tingkatan 4 Bab 2
8 Dalam satu perlawanan tenis, pemain memukul bola tenis yang mempunyai jisim 0.2 kg yang menuju ke
arahnya dengan halaju 20 m s–1. Bola itu memantul dengan halaju 40 m s–1. Masa yang diambil semasa
perlanggaran antara bola dan raket tenis ialah 0.01 s. TP 3 OP4
In a tennis match, a player hits an on-coming tennis ball with mass of 0.2 kg and velocity of 20 m s–1. The ball rebounds
with a velocity of 40 m s–1. The time taken in the collision between the tennis ball and the tennis racket is 0.01 s.

(a) Berapakah impuls yang dialami oleh bola tenis itu?


What is the impulse experienced by the tennis ball?
J = Ft = mv – mu
= 0.2(40) – (0.2)(–20)
=8+4
J = 12 kg m s–1

(b) Berapakah daya impuls yang dikenakan ke atas bola tenis?


What is the impulsive force exerted on the tennis ball?
mv – mu
F=
t
12
=
0.01
= –1 200 N

2.8 Berat
Weight

Buku Teks: m.s 70-71


1 Apakah maksud berat? 1
TP OP2
What is the meaning of weight?

Daya tarikan graviti yang bertindak ke atas objek.


Gravitational force acting on an object.

2 Menurut Hukum Gerakan Newton Kedua,


According to the Newton Second’s Law of Motion,

F = ma di mana/ where
W: Berat/ Weight
m: Jisim/ Mass
W = mg g: Pecutan graviti/ Gravitational acceleration

3 Unit S.I bagi berat adalah Newton, N. .


Newton. N.
The S.I unit for weight is .

4 Apakah maksud kekuatan medan graviti, g? TP 1 OP2


What is the meaning of gravitational field strength, g?
Daya tarikan graviti yang bertindak ke atas jisim 1 kg.

Gravitational force acts on a 1 kg of mass.

© Penerbit Mahir Sdn. Bhd. (183897-P) 34

Fizik Tg 4 (Bab 2)5th 9/2/23.indd 34 09/02/2023 12:00 PM


Fizik Tingkatan 4 Bab 2
5 Tulis persamaan bagi kekuatan medan graviti, g. TP 2 OP3
Write the equation for gravitational field strength, g.
W = mg
W
g=
m
6 Apakah nilai dan unit bagi g? TP 2 OP3
What is the value and unit for g?
g = 9.81 N kg-1

7 Nyatakan maksud lain bagi g. TP 1 OP2


State other meaning for g.
Pecutan graviti/ Gravitational acceleration

8
1
Nilai pecutan graviti, g di Bulan ialah daripada nilai g di Bumi
6
1
The value of gravitational acceleration, g on the Moon is of the value of g on the Earth
6

Kuantiti fizik Di angkasa lepas Di Bulan Di Bumi


Physical quantity In the space On the Moon On Earth

Jisim
100 kg 100 kg 100 kg
Mass

Berat 1 100 × 9.81


0N 100 × 9.81 ×
Weight 6 = 98.1 N
= 163.5 N

9 Sebuah kapal angkasa bergerak dari Bumi ke Marikh. Kekuatan medan graviti di permukaan Marikh ialah
3.7 N kg–1. Kapal angkasa itu membawa kapsul berjisim 100 kg diukur di Bumi. TP 3 OP4
A spacecraft moves from Earth to Mars, where the gravitational field strength near the surface is 3.7 N kg–1. The
spacecraft is carrying capsule which has mass of 100 kg when measured on Earth.

(a) Berapakah berat kapsul di Bumi?


What is the weight of capsule on Earth?
W = 100 × 9.81
= 981 N

(b) Berapakah jisim kapsul di angkasa lepas?


What is the mass of capsule in the space?
m = 100 kg

(c) Berapakah jisim kapsul di Marikh?


What is the mass of capsule on Mars?
m = 100 kg

(d) Berapakah berat kapsul di Marikh?


What is the weight of capsule on Mars?
W = 100 × 3.7
= 370 N

35 © Penerbit Mahir Sdn. Bhd. (183897-P)

Fizik Tg 4 (Bab 2)5th 9/2/23.indd 35 09/02/2023 12:00 PM


Fizik Tingkatan 4 Bab 2

Praktis Sumatif
KERTAS 1

1 Pecutan kereta yang bergerak dengan halaju B s


OP3 seragam akan menjadi
The acceleration of a moving car with uniform velocity
will be
A bertambah t
increase
B seragam C s
uniform
C sifar
zero
D berkurang t
decrease
D s

2 Rajah 1 menunjukkan pita detik yang ditarik oleh


OP3 troli melalui jangka masa detik dengan frekuensi
50 Hz.
t
Diagram 1 shows a ticker timer pulled by a trolley
through ticker tape with frequency of 50 Hz.
4 Rajah 2 menunjukkan graf halaju-masa sebuah
Arah gerakan OP3 bas yang membawa penumpang. Selepas
Direction of motion
10 minit, pemandu bas menekan pedal brek untuk
menghentikan bas.
Diagram 2 shows a graph of velocity-time of a bus
that carry passengers. After 10 minutes, the bus driver
0.8 cm 1.8 cm
pressed the brake pad to stop the bus.
Rajah 1
v/ m s–1
Diagram 1

Hitung pecutan troli itu.


Calculate the acceleration of the trolley. 6
A 2ms –2

B 5 m s–2
C 4 m s–2
D 10 m s–2
t/ min
0 10 12
3 Graf manakah yang mewakili objek sedang
OP4 bergerak dengan pecutan seragam?
Rajah 2
Which graph represents a moving object at uniform Diagram 2
acceleration?
A s Berapakah halaju bas sebaik sahaja pedal brek
ditekan?
What is the velocity of the bus after the brake pad is
pressed?
t A 0 m s–1 C 6 m s–1
B 2 m s–1 D 72 m s–1

© Penerbit Mahir Sdn. Bhd. (183897-P) 36

Fizik Tg 4 (Bab 2)5th 9/2/23.indd 36 09/02/2023 12:00 PM


Fizik Tingkatan 4 Bab 2
5 Rajah 3 menunjukkan duit syiling, guli dan daun A konsep inersia
OP4 kering dijatuhkan serentak di dalam silinder vakum. the concept of inertia
ATCH 2 Diagram 3 shows a coin, a ARTWORK
marble and aBY FREELANCER
dried leaf M.ZAIDEE
B konsep keseimbangan daya
being released at the same time in a vacuum cylinder. the concept of equilibrium of forces
C prinsip keabadian momentum
the principle of conservation of momentum
D prinsip keabadian tenaga
the principle of conservation of energy
Vakum
Vacuum
Tapak
8 Rajah 5 menunjukkan dua blok kayu, R dan S,
Base OP5 yang serupa, masing-masing di atas troli P dan Q.
KBAT Diagram 5 shows two identical blocks, R and S, placed
Rajah 3 on trolleys P and Q respectively.
SB-16
Diagram 3
SB-17
Pemerhatian manakah yang betul? R S
Which observation is correct?
A Ketiga-tiga objek terapung di dalam silinder
All three objects float in the cylinder P
X QY
B Ketiga-tiga objek tiba di tapak pada masa
yang sama Arah gerakan P Pegun
All three objects reach the base at the same time Direction of Stationary
C Guli tiba di tapak lebih awal daripada duit movement of P
syiling
The marble reaches the base earlier than the coin Rajah 5
D Duit syiling dan guli tiba di tapak lebih awal Diagram 5
daripada daun kering
The coin and the marble reach the base earlier
Apakah yang berlaku kepada blok kayu R dan S
than the dried leaf apabila troli P melanggar troli Q yang pegun?
What happens to the wooden blocks R and S when
6 Sebiji batu bata jatuh daripada ketinggian 10 m. trolley P collides with the stationary trolley Q?
OP4 Hitung masa yang diambil untuk batu bata itu
tiba ke tanah. R S
A brick falls from a height of 10 m. Calculate the time
taken for the brick to reach the ground. Terhumban ke kanan Terhumban ke kanan
A
A 0.5 s C 1.0 s Pushed to the right Pushed to the right
B 0.7 s D 1.4 s
Terhumban ke kanan Terhumban ke kiri
B
7 Rajah 4 menunjukkan keadaan pembonceng Pushed to the right Pushed to the left
OP2 motosikal yang bergerak ke belakang apabila
motosikal mula memecut. Terhumban ke kiri Terhumban ke kiri
C
Diagram 4 shows a pillion rider moving backwards Pushed to the left Pushed to the left
when the motorcycle accelerates.
Terhumban ke kiri Terhumban ke kanan
D
Pushed to the left Pushed to the right

9 Sebutir peluru berjisim 10 g ditembak daripada


OP4 senapang berjisim 490 g. Peluru meninggalkan
senapang dengan halaju 120 m s–1.
Cari halaju sentakan senapang itu.
A bullet of mass 10 g is fired from a gun of mass
Rajah 4 490 g. The bullet leaves the gun with a speed of
Diagram 4 120 m s–1. Find the speed of recoil of the gun.
A 2.4 m s–1
Pergerakan pembonceng yang ke belakang boleh
B 4.8 m s–1
diterangkan oleh
The backwards movement of the pillion rider can be
C 6.4 m s–1
explained by D 7.4 m s–1

37 © Penerbit Mahir Sdn. Bhd. (183897-P)

Fizik Tg 4 (Bab 2)5th 9/2/23.indd 37 09/02/2023 12:00 PM


Fizik Tingkatan 4 Bab 2
10 Situasi yang manakah mengaplikasikan prinsip Jika jumlah jisim basikal dan atlet ialah 80 kg,
OP2keabadian momentum? maka pecutan basikal adalah
In which situation can the principle of conservation of If the total mass of the bicycle and the athlete is 80 kg,
momentum be applied? then the acceleration of the bicycle is
A A 0.67 m s–2
B 0.80 m s–2
C 1.25 m s–2
D 1.50 m s–2

12 Rajah 7 menunjukkan seorang pemain bola lisut


menggerakkan tangannya ke belakang semasa
OP5
KBAT menangkap bola yang laju.
B 2
TINGKATAN 4 BATCH ARTWORK BY FREELANCER M.ZAIDEE
Diagram 7 shows a softball player moves his hand
backwards while catching a fast-moving ball.

D SB-16
Rajah
SB-177
Diagram 7

Pergerakan tangan itu bertujuan untuk


This movement of the hand is to
SB-15 A menambahkan daya impuls
11 Rajah 6 menunjukkan seorang atlet mengayuh increase the impulsive force
OP4 basikalnya dengan daya 120 N. B menambahkan masa menghentikan bola
Diagram 6 shows an athlete cycling his bicycle with a increase the stopping time of the ball
force of 120 N. C mengawal bola daripada jatuh
control the ball to stop falling

13 Berapakah berat sebuah objek berjisim 20 kg di


OP4permukaan Bulan?
What is the weight of an object with mass of 20 kg on
the surface of the Moon?
A 196.2 N
20 N B 3.3 N
C 20.0 N
120 N D 32.7 N
SB-18 Rajah 5
Diagram 5

© Penerbit Mahir Sdn. Bhd. (183897-P) 38

Fizik Tg 4 (Bab 2)5th 9/2/23.indd 38 09/02/2023 12:00 PM


Fizik Tingkatan 4 Bab 2

KERTAS 2
Bahagian A
1 Rajah 1.1 dan Rajah 1.2 menunjukkan dua pita detik yang berlainan panjang dihasilkan oleh gerakan dua
troli.
Diagram 1.1 and Diagram 1.2 show two ticker tapes with different lengths produced by the motion of two trolleys.

Rajah 1.1
Diagram 1.1

Rajah 1.2
Diagram 1.2

(a) Apakah yang dimaksudkan dengan panjang? OP2


What is meant by the length?
Jarak di antara dua titik

The distance between two dots


[1 markah/mark]

(b) Berdasarkan Rajah 1.1 dan Rajah 1.2, bandingkan OP3


Based on Diagram 1.1 and Diagram 1.2, compare

(i) bilangan detik.


the number of ticks.
Sama/ Same

(ii) panjang pita detik.


the length of the ticker tapes.
Panjang pita detik Rajah 1.2 lebih panjang daripada pita detik Rajah 1.1.

The length of the ticker tape in Diagram 1.2 is longer than the ticker tape in Diagram 1.1.

(iii) laju purata pita detik.


the average speed of the ticker tapes.
Laju purata dalam Rajah 1.2 lebih tinggi daripada dalam Rajah 1.1.

The average speed in Diagram 1.2 is higher than in Diagram 1.1.


[3 markah/marks]

(c) Menggunakan jawapan anda di 1(b), nyatakan hubungan antara panjang pita detik dengan laju purata.
Using your answer in 1(b), state the relationship between the length of the ticker tape and average speed. OP4

Berkadar terus/ Directly proportional


[1 markah/mark]

39 © Penerbit Mahir Sdn. Bhd. (183897-P)

Fizik Tg 4 (Bab 2)5th 9/2/23.indd 39 09/02/2023 12:00 PM


Fizik Tingkatan 4 Bab 2
(d) Rajah 1.3 menunjukkan satu carta pita detik yang dihasilkan daripada gerakan sebuah troli.
Diagram 1.3 shows a ticker tape chart produced from the motion of a trolley.
Panjang (cm)
Length

X Y
8.0

6.0

4.0

W
2.0

Masa (s)
Time

Rajah 1.3
Diagram 1.3

Berdasarkan RajahSUPERB
1.3, terangkan jenis
SBP FIZIK gerakan dari
TINGKATAN W hingga
4 BATCH 2 X. OP3 ARTWORK BY FREE
Based on Diagram 1.3, explain the type of motion from W to X.
Halaju bertambah secara seragam // Pecutan malar / /Pertambahan panjang malar // Jarak di antara titik

bertambah secara seragam

The velocity increases uniformly // Constant acceleration // Increasing in length uniformly // The distance between

dots increases uniformly


[2 markah/marks]
SB-16

Bahagian B

2 Rajah 2.1 menunjukkan pemain golf masih meneruskan


SB-15 ayunan kayu golf selepas bola golf dipukul.
Tindakan ini dinamakan tindakan ikut lajak.
Diagram 2.1 shows a golfer continues his swing after the golf ball has been hit. This action is called follow through
action.

Rajah 2.1
SB-18
Diagram 2.1
Tujuan tindakan ikut lajak adalah untuk meningkatkan impuls yang bertindak ke atas bola.
The purpose of follow through action is to increase the impulse acting on the ball.

© Penerbit Mahir Sdn. Bhd. (183897-P) 40

Fizik Tg 4 (Bab 2)5th 9/2/23.indd 40 09/02/2023 12:00 PM


Fizik Tingkatan 4 Bab 2
(a) Apakah maksud impuls? OP2
What is the meaning of impulse?
[1 markah/mark]

(b) Terangkan bagaimana tindakan ikut lajak boleh meningkatkan impuls yang bertindak ke atas bola. OP3
Explain how the follow through action can increase the impulse acting on the ball.
[2 markah/marks]

(c) Rajah 2.2 menunjukkan empat batang tiang bendera P, Q, R dan S dengan spesifikasi yang berbeza
didirikan di atas tanah. OP7 KBAT
Diagram 2.2 shows four flag poles, P, Q, R and S with different specifications mounted on the ground.

Ketegangan Ketegangan
maksimum tali P Q maksimum tali
Maximum tension of Maximum tension of
Tiang
string string
Pole Tiang
= 1 000 N Pole = 2 000 N
Tali Tali
String String
Ketumpatan tiang Ketumpatan tiang
Density of pole Tanah Tanah Density of pole
1m 1m 1m 1m
= 1 500 kg m–3 Ground Ground = 900 kg m–3

Ketegangan Ketegangan
maksimum tali R S maksimum tali
Maximum tension of Maximum tension of
Tiang
string Pole Tiang string
= 2 000 N Pole = 1 000 N
Tali Tali
String String
Ketumpatan tiang Ketumpatan tiang
Density of pole Tanah Tanah Density of pole
1m 1m 1m 1m
= 1 500 kg m–3 Ground Ground = 900 kg m–3

Rajah 2.2
Diagram 2.2
Anda dikehendaki menentukan ciri-ciri tiang bendera yang paling sesuai untuk menahan tiupan angin
yang kuat. Kaji spesifikasi keempat-empat tiang bendera itu daripada aspek berikut:
You are required to determine the most suitable characteristics of flag pole that can withstand strong wind. Study
the specifications of all the four flag poles from the following aspects:

(i) Ketumpatan tiang


The density of the pole
(ii) Keterlenturan tiang
The fl exibility of the pole
(iii) Ketegangan tali
The tension of the string
(iv) Ketinggian tali yang diikat pada tiang
The height of strings tied to the pole

Terangkan kesesuaian aspek-aspek itu dan tentukan tiang bendera yang paling sesuai. Beri sebab bagi
pilihan anda.
Explain the suitability of the aspects and determine the most suitable flag pole. Give reasons for your choice.
[10 markah/marks]

41 © Penerbit Mahir Sdn. Bhd. (183897-P)

Fizik Tg 4 (Bab 2)5th 9/2/23.indd 41 09/02/2023 12:00 PM


Fizik Tingkatan 4 Bab 3

BAB
3
Kegravitian
Gravitation

Tema Mekanik Newton


Theme Newton Mechanics

NOTA EFEKTIF

1 Hukum Kegravitian Semesta/ Newton’s Universal Law of Gravitation


(a) Daya graviti, F adalah berkadar terus dengan hasil darab dua jasad jisim m1 dan m2 dan berkadar
songsang dengan kuasa dua jarak r di antara kedua-dua jasad.
The gravitational force, F between two bodies is directly proportional to the product of the masses, m1 and m2 of
both bodies and inversely proportional to the square of the distance r between the centres of the two bodies.
Jasad 2
Jasad 1 Body 2
Body 1
F F
m1m2
F =G
m1 m2 r2
G = Pemalar kegravitian/ Gravitational constant
Jarak, r (6.6.7 × 10–11 N m2 kg–2)
Distance

(b) Daya memusat: Daya yang bertindak ke atas jasad yang sedang membuat gerakan membulat.
Centripetal force: A force acts on the body in a direction towards the centre of the circle.
2 Hukum Kepler Pertama/ Kepler’s First Law
Orbit bagi setiap planet adalah elips dengan Matahari berada di salah satu fokusnya.
All the planets move in elliptical orbits with the Sun at one focus.
Paksi minor
Minor axis
Planet
Paksi major
Major axis

Matahari
Sun

3 Hukum Kepler Kedua/ Kepler’s Second Law


(a) Satu garis yang menyambungkan planet dengan B
Matahari akan mencakupi luas yang sama dalam C Lebih
selang masa yang sama apabila planet bergerak perlahan
Lebih laju Slower
dalam orbitnya.
Faster
A line that connects a planet to the Sun sweeps out Matahari D
equal areas in equal times. Sun
(b) Planet mengambil masa yang sama untuk A
Planet
bergerak dari A ke B dan C ke D. Hal ini kerana
planet bergerak lebih laju dari A ke B berbanding dari C ke D.
A planet takes the same time to move from A to B and from C to D. This is because the planet moves faster
from A to B rather than from C to D.
4 Hukum Kepler Ketiga/ Kepler’s Third Law
Kuasa dua tempoh planet adalah berkadar terus dengan kuasa tiga jejari orbitnya, T 2 ∝ r3.
The square of the orbital period of any planet is directly proportional to the cube of the radius of its orbit, T 2 ∝ r3.

© Penerbit Mahir Sdn. Bhd. (183897-P) 42

Fizik Tg 4 (Bab 3)6th 9/2/23.indd 42 09/02/2023 4:20 PM


Fizik Tingkatan 4 Bab 3

3.1 Hukum Kegravitian Semesta Newton


Newton’s Universal Law of Gravitation

Buku Teks: m.s 78-95

INFO
Sehelai daun gugur ke tanah (Bumi) daripada sebatang pokok. Kedua-dua daun dan Bumi mengalami daya graviti
yang sama. Daun dan Bumi bergerak mendekati antara satu sama lain. Namun, disebabkan jisim Bumi jauh lebih
besar daripada jisim daun, kita hanya dapat melihat daun itu jatuh ke arah Bumi.
A leaf falls to the ground from a tree. Both the leaf and the Earth experience the same gravitational force. The leaf and the Earth moves
towards one another. As the mass of the Earth is very much larger than the mass of the leaf, we only observe the leaf falling to the ground.

1 Daya graviti dikenali sebagai daya semesta kerana daya graviti bertindak antara mana-mana dua jasad dalam
alam semesta.
it acts between any two bodies in the universe.
Gravitational force is known as universal force because .

Bumi
Daya graviti antara Matahari Earth
dengan Bumi
The gravitational force between Daya graviti antara Bumi
the Sun and the Earth dengan Bulan
The gravitational force between the
Earth and the Moon

Bulan
Moon
Matahari
Sun Daya graviti antara Matahari dengan Bulan
The gravitational force between the Sun and the Moon

2 Dua hubungan yang melibatkan daya graviti antara dua jasad: TP 2 OP3
Two relationships involves gravitational force between two bodies:

(i) Daya graviti berkadar terus dengan hasil darab jisim dua jasad.
directly proportional
The gravitational force is to the product of the masses of the two bodies.

(ii) Daya graviti berkadar songsang dengan kuasa dua jarak di antara pusat dua jasad.
inversely proportional
The gravitational force is to the square of the distance between the centres of the
two bodies.

1
F ∝ m1 m 2 F∝
r2

m1 m2
F∝
r2

Gm1 m2 Hukum Kegravitian Semesta Newton


F= Newton’s Universal Law of Gravitation
r2

43 © Penerbit Mahir Sdn. Bhd. (183897-P)

Fizik Tg 4 (Bab 3)6th 9/2/23.indd 43 09/02/2023 4:20 PM


Fizik Tingkatan 4 Bab 3

3 Hukum Kegravitian Semesta Newton menyatakan bahawa daya graviti antara dua jasad adalah berkadar terus
terus dengan hasil darab jisim kedua-dua jasad dan berkadar songsang dengan kuasa dua jarak di antara pusat

dua jasad tersebut.


the gravitational force between two bodies is directly proportional to
Newton’s Universal Law of Gravitation states that
the product of the masses of both bodies and inversely proportional to the square of the distance between the centers

of the two bodies.

4 Jasad 2
Jasad 1 Body 2
Body 1
Gm1m2
F= F F
r2 m1 m2

Jarak, r
Distance, r

F Daya graviti antara dua jasad / Gravitational force between two bodies

G Pemalar kegravitian/ Gravitational constant (6.67 × 10–11 N m2 kg–2)

m1 Jisim jasad 1/ Mass of body 1

m2 Jisim jasad 2 / Mass of body 2

r Jarak di antara pusat dua jasad/ Distance between centres of two bodies

5 Menyelesaikan masalah melibatkan Hukum Kegravitian Semesta Newton.


Solve problem involving Newton’s Universal Law of Gravitation.

(a) Rajah menunjukkan Ahmad dan Alisa berdiri dengan jarak 2.0 m di antara satu sama lain.
Diagram shows Ahmad and Ali standing in distance of 2.0 m between each other.

2.0 m
Ahmad Alisa

55 kg 45 kg

(i) Hitung daya graviti antara Ahmad dan Alisa. TP 3 OP4


Calculate the gravitational force between Ahmad and Alisa.
Gm1 m2 (6.67 × 10–11)(55)(45)
F= =
r2 (2.0)2
F = 4.1 × 10–8 N

© Penerbit Mahir Sdn. Bhd. (183897-P) 44

Fizik Tg 4 (Bab 3)6th 9/2/23.indd 44 09/02/2023 4:20 PM


Fizik Tingkatan 4 Bab 3
(ii) Hitung daya graviti jika jarak di antara mereka ditambah sehingga 4.0 m. TP 2 OP3
Calculate the gravitational force if their distance is increased to 4.0 m.
Gm1 m2 (6.67 × 10–11)(55)(45)
F= =
r2 (4.0)2
F = 1.0 × 10–8 N

(iii) Nyatakan hubungan antara jisim jasad dengan daya graviti. TP 2 OP3
State the relationship between the mass of body and the gravitational force.
Berkadar terus/ Directly proportional

(iv) Apakah kesan jarak di antara dua jasad ke atas daya graviti? TP 3 OP4
What is the effect of distance between two bodies on gravitational force?
Semakin bertambah jarak di antara dua jasad, semakin kecil daya graviti.

The further the distance between two bodies, the smaller the gravitational force.

(b) Rajah menunjukkan kedudukan Matahari, Bulan, Bumi dan satelit buatan.
The diagram shows the position of the Sun, the Moon, the Earth, and the man-made satellite.

Bulan
Moon

Satelit buatan
Man-made satellite
Bumi
Earth

Matahari
Sun

Jadual menunjukkan ciri-ciri Matahari, Bulan, Bumi dan satelit buatan.


The table shows the details of the Sun, the Moon, the Earth, and the man-made satellite.

Matahari/ Sun Bumi/ Earth


Jisim/ Mass = 1.99 × 1030 kg Jisim/ Mass = 5.97 × 1024 kg
Jarak di antara Bumi dan Matahari/ Distance Jejari/ Radius = 6.37 × 106 m
between the Earth and the Sun = 1.50 × 1011 m

Satelit buatan/ Man-made satellite Bulan/ Moon


Jisim/ Mass = 1.20 × 103 kg Jisim/ Mass = 7.35 × 1022 kg
Jarak di antara Bumi dan satelit/ Distance between
the Earth and the satellite = 4.22 × 107 m

(i) Berapakah daya graviti ke atas satelit buatan itu sebelum dilancarkan? TP 3 OP4
What is the gravitational force on the man-made satellite before it is launched?
GMm (6.67 × 10–11)(5.97 × 1024)(1.20 × 103)
F= =
R2 (6.37 × 106)2
F = 11 776.13 N

45 © Penerbit Mahir Sdn. Bhd. (183897-P)

Fizik Tg 4 (Bab 3)6th 9/2/23.indd 45 09/02/2023 4:20 PM


Fizik Tingkatan 4 Bab 3
(ii) Hitungkan daya graviti antara Bumi dengan satelit buatan. TP 3 OP4
Calculate the gravitational force between the Earth and the man-made satellite.
GMm (6.67 × 10–11)(5.97 × 1024)(1.20 × 103)
F= =
(R + h)2 [(6.37 × 106) + (4.22 × 107 )]2
F = 2.03 × 102 N

(iii) Ramalkan perbezaan antara magnitud daya graviti Bumi dan satelit buatan dengan daya graviti
Matahari dan Bumi. TP 4 OP5
Predict the difference in the magnitude of the gravitational force between the Earth and man-made satellite
and the gravitattional force between the sub and the Earth.
Daya graviti Matahari dan Bumi lebih besar.

The gravitational force between the Sun and the Earth is bigger.

(iv) Daya graviti antara Bumi dengan Bulan ialah 2.0 × 1020 N. Berapakah jarak di antara pusat Bumi
dengan pusat Bulan? TP 3 OP4
The gravitational force between the Earth and the Moon is 2.0 × 1020 N. What is the distance between the
centre of the Earth and the centre of the Moon?
GMm GMm
F= →r=
r 2
F
(6.67 × 10–11)(5.97 × 1024)(7.35 × 1022)
=
2.0 × 1020
= 3.82 × 108 m

6 Menerbitkan rumus pecutan graviti. TP 3 OP4


To derive the formula for gravitational acceleration.

(a) (i) Hukum Gerakan Newton Kedua F = mg


Newton’s Second Law of Motion

(ii) Hukum Kegravitian Semesta Newton GMm


Newton’s Universal Law of Gravitation F=
r2

(iii) Menyamakan kedua-dua persamaan GMm


Equate two equations mg =
r2

(iv) Maka, pecutan graviti, g GM


Then, the gravitational acceleration, g g=
r2

(b) Pecutan graviti, g adalah berkadar terus dengan pemalar kegravitian, G.


directly proportional
The gravitational acceleration, g is to the gravitational constant, G.

(c) Faktor faktor yang mempengaruhi nilai pecutan graviti: TP 3 OP4


Factors that influence the value of gravitational acceleration:
• Jisim Bumi/ Mass of the Earth

• Jarak di antara Bumi dengan objek/ Distance between Earth and object

© Penerbit Mahir Sdn. Bhd. (183897-P) 46

Fizik Tg 4 (Bab 3)6th 9/2/23.indd 46 09/02/2023 4:20 PM


Fizik Tingkatan 4 Bab 3
7 Rajah menunjukkan graf bagi variasi nilai pecutan graviti dengan jarak dari pusat Bumi. TP 3 OP4
Graph shows the variation of gravitational acceleration with the distance from the centre of the Earth.
g

GM
g=
r2

0 r
R

(a) Bagaimanakah nilai pecutan graviti berubah apabila jarak dari pusat Bumi bagi kedudukan r < R?
How does the value of gravitational acceleration change when the distance from the centre of the Earth, r < R?
Berkadar terus/ Directly proportional

(b) Bagaimanakah nilai pecutan graviti berubah apabila jarak dari pusat Bumi bertambah r ≥ R?
How does the value of gravitational acceleration change when the distance from the centre of the Earth r ≥ R?
Berkadar songsang/ Inversely proportional

(c) Sebuah statelit berjisim 80 kg mengorbit Bumi pada ketinggian 1.5 × 106 km dari permukaan
Bumi.
Hitungkan pecutan graviti pada orbit tersebut.
A satellite has a mass of 80 kg is orbiting the Earth at a height of 1.5 × 106 km from the surface of the Earth.
Calculate the gravitational acceleration on the orbit.
GM (6.67 × 10–11)(5.97 × 1024)
g= =
(R +h)2 [(6.37 × 106) + (1.5 × 106)]2
g = 177.0 m

8 Apakah maksud daya memusat? TP 1 OP2


What is the meaning of centripetal force?

Suatu daya yang bertindak ke atas suatu jasad yang melakukan gerakan membulat dengan arah yang
sentiasa menuju ke pusat bulatan itu.
Force acts on the body in a direction towards the center of the circle for a body in circular motion.

9
F Daya memusat/ Centripetal force

m Jisim jasad/ Mass of body


mv2
F=
r
v Laju linear/ Linear speed

r Jejari orbit/ Radius of orbit

47 © Penerbit Mahir Sdn. Bhd. (183897-P)

Fizik Tg 4 (Bab 3)6th 9/2/23.indd 47 09/02/2023 4:20 PM


Fizik Tingkatan 4 Bab 3
10 Dengan F= ma, maka rumus pecutan memusat diterbitkan.
By F = ma, then the formula of centripetal acceleration can be derived.
mv2
F = ma =
r
v2
a=
r

11 (a) Menentukan jisim Bumi dan Matahari. TP 3 OP4


To determine the mass of the Earth and the Sun.

Hukum Kegravitian Semesta Newton Daya memusat


Newton’s Universal Law of Gravitational Centripetal force
GMm mv2
F= F=
r2 r

Menyamakan dua persamaan berikut: TIP


Equate the following two equations: Pintar
GMm mv2
= Jarak Bulan membuat satu
r2 r
orbit lengkap mengelilingi
GM = v2 r …………(1) Bumi adalah 2πr.
Distance of the Moon making
2πr one complete orbit around the
Laju linear, v =
Linear speed T Earth is 2πr.

Gantikan v ke dalam persamaan (1):


Subtitute v into the equation (1):
2
2πr
GM = r
T
4π2 r3
GM =
T2
Maka, rumus jisim Bumi atau Matahari adalah
Then, the formula of the mass of the Earth

4π2 r3
M=
GT 2
di mana/ where
r = jejari orbit planet// Bulan// satelit/ radius of orbit of planet// Moon//satellite
T = tempoh peredaran/ period of revolution

(b) Tempoh peredaran Bulan mengelilingi Bumi, ialah T = 2.36 × 106 s dan jejari orbit Bulan, r
= 3.83 × 108 m. Hitungkan jisim Bumi, M. TP 3 OP4
Period of revolution of the Moon around the Earth, T = 2.36 × 106 s and the radius of the Moon’s orbit, r
= 3.83 × 108 m. Calculate the mass of the Earth, M.
4π 2r 3 4π2(3.83 × 108)3
M= =
GT 2 (6.67 × 10–11)(2.36 ×106)2
M = 5.97 × 1024 kg

© Penerbit Mahir Sdn. Bhd. (183897-P) 48

Fizik Tg 4 (Bab 3)6th 9/2/23.indd 48 09/02/2023 4:20 PM


Fizik Tingkatan 4 Bab 3
(c) Bumi bergerak mengelilingi Matahari dengan tempoh satu tahun dan jejari orbit adalah 1.50 × 1011 m.
Hitungkan jisim Matahari. TP 3 OP4
The Earth revolves the Sun in a period of one year and the radius of the orbit is 1.50 × 1011 m. Calculate the
mass of the Sun.
4π 2r 3 4π2(1.50 × 1011)3
M= =
GT 2 (6.67 × 10–11)(365 × 24 × 60 × 60)2
M = 2.0 × 1030 kg

3.2 Hukum Kepler


Kepler’s Law

Buku Teks: m.s 96-102


1 (a) Apakah Hukum Kepler Pertama? TP 1 OP2
What is Kepler’s Firs Law?

Orbit setiap planet adalah elips dengan Matahari berada di satu daripada fokusnya.
All planets move in elliptical orbits with the Sun at one focus.

(b) Bentuk orbit planet-planet dalam Sistem Suria adalah elips .


eclipse
The shape of orbits used by the planets in the Solar System is .

(c) Paksi major lebih panjang daripada paksi minor. Kebanyakan orbit planet mempunyai paksi

major hampir sama panjang dengan paksi minor.


longer
The major axis is than the minor axis. Most orbits of the planets have minor axis and major
same
axis of almost the length.

(d) Jejari orbit adalah nilai purata bagi jarak di antara planet dengan Matahari.
radius of orbit
The is the average value of the distance between the planet and the Sun.

Paksi minor
Minor axis
Planet
Paksi major
Major axis

Matahari
Sun

2 (a) Apakah Hukum Kepler Kedua? TP 1 OP2


What is Kepler’s Second Law?

Garis yang menyambungkan planet dengan Matahari akan mencakupi luas yang sama dalam selang
masa yang sama apabila planet bergerak dalam orbitnya.
A line that connect a planet to the Sun sweeps out equal areas in equal times.

49 © Penerbit Mahir Sdn. Bhd. (183897-P)

Fizik Tg 4 (Bab 3)6th 9/2/23.indd 49 09/02/2023 4:20 PM


Fizik Tingkatan 4 Bab 3
(b)
B
C
F

Matahari D
Sun
A
Planet

Jika sebuah planet mengambil masa yang sama untuk bergerak dari A ke B dan C ke D, maka:
If a planet takes the same amount of time to move from A to B and C to D, then:

(i) Luas kawasan AFB adalah sama dengan luas kawasan CFD.
the same as
The area of AFB is the area of CFD.

(ii) Jarak AB adalah lebih jauh daripada jarak CD.


further
The distance of AB is than the distance CD.

(iii) Laju linear A ke B adalah lebih tinggi daripada laju linear C ke D.


higher
The linear speed from A to B is than the linear speed from C to D.

3 (a) Apakah Hukum Kepler Ketiga? TP 1 OP2


What is Kepler’s Third Law?

Kuasa dua tempoh orbit planet adalah berkadar terus dengan kuasa tiga jejari orbitnya.
The square of the orbital period of any planet is directly proportional to the cube of the radius of its orbit.

(b) Hukum Kepler Ketiga secara matematik adalah T 2 α r3 .


T 2 α r3
Mathematically, the Kepler’s Third law is .

(c) Planet yang mengorbit dengan jejari orbit planet besar, mempunyai tempoh orbit lebih
panjang .
longer
A planet which orbits with a larger radius has a orbital period.

(d) Planet yang lebih jauh daripada Matahari mengambil masa lebih panjang untuk melengkapkan
satu orbit mengelilingi Matahari.
longer
Planets which are further from the Sun take a time to complete one orbit around the Sun.

4 Merumuskan Hukum Kepler Ketiga.


Formulating Kepler’s Third Law.
v
Planet
M m
F

Matahari
Sun

© Penerbit Mahir Sdn. Bhd. (183897-P) 50

Fizik Tg 4 (Bab 3)6th 9/2/23.indd 50 10/02/2023 10:07 AM


Fizik Tingkatan 4 Bab 3

(a) Daya graviti bertindak ke atas planet, GMm


Gravitational force acting on the planet, F=
r2

(b) Daya memusat planet, mv2


Centripetal force of the planet, F=
r

(c) Menyamakan kedua-dua persamaan, GMm mv2 GM


Equate the two equations, = → v2 =
r 2
r r

(d) Gantikan ke dalam persamaan: 2


Subtitute into the equation: 2πr GM
=
T r
4π2
T2 = r3
GM

(e) k = pemalar, maka Hukum Kepler Ketiga, T 2 = kr3


k = constant, then Kepler’s Third law,

5 Menyelesaikan masalah melibatkan hukum Kepler Ketiga.


Solve problem involving Kepler’s Third law.

(a) Jika dua planet dibandingkan, persamaan digunakan untuk menghitung tempoh orbit atau jejari orbit
adalah
If two planets are compared, the equation used to calculate the orbital period or the radius or orbit is

T12 r13
=
T 22 r23

(b) Sebuah satelit mengorbit pada ketinggian 420 km dari permukaan Bumi. Berapakah tempoh orbit satelit
itu? TP 3 OP4
A satellite orbiting at a height of 420 km from the surface of the Earth. What is the orbital period of the satellite?
[Jejari orbit Bulan = 3.83 × 108 m, tempoh orbit Bulan = 655.2 jam, jejari Bumi = 6.37 × 106 m]
[Radius of the orbit of the Moon = 3.83 × 108 m, orbital period of the Moon = 655.2 hours, radius of Earth
= 6.37 × 106 m]
Jejari orbit satelit/ Radius of orbit of the satellite, r1 = (6.37 × 106) + (420 × 103) = 6.8 × 106 m
Tempoh orbit satelit/ Orbital period of the satellite = T1
Jejari orbit Bulan/ Radius of orbit of the Moon, r2 = 3.83 × 108 m
Tempoh orbit Bulan/ Orbital period of the Moon, T2 = 655.2 jam/ hours
T12 r13
=
T 22 r23
(6.8×106)3 × (655.2)2
T12 =
(3.83 × 108)3
T1 = 1.55 jam/ hours

51 © Penerbit Mahir Sdn. Bhd. (183897-P)

Fizik Tg 4 (Bab 3)6th 9/2/23.indd 51 09/02/2023 4:20 PM


Fizik Tingkatan 4 Bab 3

3.3 Satelit Buatan Manusia


Man-made Satellite

Buku Teks: m.s 103-110


1 Rajah menunjukkan orbit sebuah satelit yang mengelilingi Bumi. Satelit yang bergerak dalam orbit
mengelilingi Bumi akan mengalami daya memusat, iaitu daya graviti.
The diagram shows the orbit of a satellite around the Earth. A satellite moving in a circular orbit around the Earth
experiences centripetal force, which is gravitational force.
v
Satelit
F Satellite
M m

r
Bumi
Earth

GM GM
Laju linear satelit
v2 = → v=
r r Linear speed of satellite

GM adalah malar. Laju linear bergantung kepada jejari orbitnya. Jika sebuah satelit berada pada ketinggian, h
di atas permukaan Bumi dan jejari orbit, r = R + h. Maka, laju linear satelit:
GM is constant. Linear speed depends on the radius of its orbit. If a satellite is at a height, h above the surface of the Earth
and the radius of orbit, r = R + h. Then, the linear speed of the satellite:

INFO
Laju linear satelit adalah kelajuan diperlukan satelit
GM untuk kekal mengorbit pada ketinggian tertentu.
v= Linear speed of satellite is the speed required by the satellite to
R+h
remain orbiting at a specific height.

2 (a) Sebuah satelit Sistem Kedudukan Sejagat (GPS) mengorbit Bumi. Hitung laju linear satelit itu jika ia
berada pada ketinggian h = 20 000 km. TP 4 OP5
A GPS satellite is orbiting the Earth. Calculate the linear speed of the satellite if it is at an altitude of h = 20 000 km.
[R = 6.37 × 106 m, G = 6.67 × 10–11 N m2 kg-2, M = 5.97 × 1024 kg]
GM
v=
R+h
(6.67 × 106)(5.97 ×1024)
=
(6.63 × 106) + (2.02 × 107)
= 3.87 × 103 m s–1

(b) Ramalkan apakah yang bakal berlaku kepada satelit itu jika laju linear kurang daripada jawapan di 2(a).
Predict what will happen to the satellite if the linear speed lower than the answer in 2(a). TP 5 OP6

Satelit akan jatuh ke orbit yang lebih rendah dan terus memusar mendekati Bumi sehingga memasuki

atmosfera.

The satellite will fall to a lower orbit and continue to revolve towards the Earth until it enters the atmosphere.

© Penerbit Mahir Sdn. Bhd. (183897-P) 52

Fizik Tg 4 (Bab 3)6th 9/2/23.indd 52 09/02/2023 4:20 PM


Fizik Tingkatan 4 Bab 3
3 Persamaan satelit geopegun dengan satelit bukan geopegun.
Similarities between geostationary satellite and non-geostationary satellite.

Orbit mengelilingi Bumi GM 4π2 r3


v= T=
Orbiting around the Earth R GM

4 Perbezaan satelit geopegun dengan satelit bukan geopegun. TP 3 OP4


Differences between geostationary satellite and non-geostationary satellite.

Satelit geopegun Satelit Satelit bukan geopegun


Geostationary satellite Satellite Non-geostationary satellite

Arah gerakan sama dengan arah Arah gerakan tidak perlu sama dengan
putaran Bumi (a) Arah gerakan arah putaran Bumi
Direction of motion same as the Direction of motion Direction of motion need not be the same
direction of Earth rotation as the direction of Earth rotation

T lebih singkat atau lebih lama daripada


T = 24 jam (b) Tempoh orbit
24 jam
T = 24 hours Orbital period
T is shorter or longer than 24 hours

Berada di atas tempat yang sama di (c) Kedudukan di Berada di atas tempat yang berubah-
muka Bumi muka Bumi ubah di muka Bumi
Above the same geopraphical location Position on the Earth Above different geographical location

Satelit komunikasi (d) Fungsi Pengimejan Bumi // GPS // Kaji Cuaca


Communication satellite Function Earth imaging // GPS // Weather forecast

(e) Contoh
MEASAT TiungSAT // RazakSAT // Pipit ISS
Example

5 (a) Apakah maksud halaju lepas? TP 1 OP2


What is the meaning of escape velocity?

Halaju minimum yang diperlukan oleh objek di permukaan Bumi untuk mengatasi daya graviti
dan terlepas ke angkasa lepas.
The minimum velocity needed by an object on the surface of the Earth to overcome the gravitational force and
escape.

53 © Penerbit Mahir Sdn. Bhd. (183897-P)

Fizik Tg 4 (Bab 3)6th 9/2/23.indd 53 09/02/2023 4:20 PM


Fizik Tingkatan 4 Bab 3
(b) Menerbitkan rumus halaju lepas.
Derived the formula for escape velocity.

M m INFO
v
r
Tenaga keupayaan graviti,
Objek Gravitational potential energy,
Object GMm
U=
1 2 r
Tenaga kinetik = mv
Kinetic energy 2
Bumi
Earth

(i) Tenaga kinetik minimum + Tenaga keupayaan = 0


Minimum kinetic energy + Potential energy = 0

1 GMm
mv2 + – =0
2 r
2GM
v2 =
r

(ii) Halaju lepas,


Escape velocity, 2GM
v=
r

(c) Hitung halaju lepas bagi Bumi. TP 3 OP4


Calculate the escape velocity from the Earth.
[Jisim Bumi, M = 5.97 × 1024 kg, jejari Bumi, R = 6.37 × 106 m]
[Mass of the Earth, M = 5.97 × 1024 kg, radius of the Earth, R = 6.37 × 106 m]

2GM 2 (6.67 × 10–11)(5.97 × 1024)


v= =
r (6.37 ×106)
v = 1.12 × 104 m s–1

6 Bagaimanakah roket boleh mencapai halaju lepas dari Bumi dan menghantar kapal angkasa ke angkasa lepas?
How can a rocket achieve escape velocity from the Earth and send the spacecraft into outer space? TP 4 OP5

Pelancaran roket memerlukan kuantiti bahan api yang besar. Pembakaran bahan api perlu menghasilkan

kuasa rejang yang tinggi bagi membolehkan roket mencapai halaju lepas dari Bumi dan menghantar kapal

angkasa ke angkasa lepas.

The launching of rockets requires large quantities of fuel to produce high thrust that enables the rocket to achieve escape

velocity of the Earth. Hence, it can send the spacecraft into outer space.

VIDEO
bit.ly/40ESOgm
Satelit buatan manusia
Man-made satellites

© Penerbit Mahir Sdn. Bhd. (183897-P) 54

Fizik Tg 4 (Bab 3)6th 9/2/23.indd 54 09/02/2023 4:20 PM


Fizik Tingkatan 4 Bab 3

Praktis Sumatif
KERTAS 1

1 Antara berikut, pernyataan manakah 3 Sebuah satelit di permukaan Bumi mengalami


OP3 menerangkan tentang Hukum Kegravitian OP4 daya graviti W.
Semesta Newton? KBAT Apakah daya graviti yang bertindak ke atas satelit
Which of the following statements explain the Newton’s itu apabila ia ditempatkan ke dalam orbit yang
Universal Law of Gravitation? ketinggiannya, 3R dari permukaan Bumi di mana
I Daya graviti berkadar terus dengan kuasa dua R ialah jejari Bumi?
jarak di antara dua pusat A satellite on the surface of the Earth experiences a
The gravitational force is directly proportional to gravitational force, W.
the square of the distance between the centres of What is the gravitational force that acts on the satellite
the bodies when it is at an orbit of height 3R from the Earth’s
II Daya graviti berkadar songsang dengan kuasa surface, where R is the radius of the Earth?
dua jarak di antara dua pusat W
A
The gravitational force is inversely proportional to 16
the square of the distance between the centres of W
the bodies B
9
III Daya graviti berkadar terus dengan kuasa tiga
C 9W
jejari orbit planet
D 16W
The gravitational force is inversely proportional to
the square of the distance between the centres of
the bodies 4 Rajah 1 menunjukkan Bumi dan Marikh yang
IV Daya graviti berkadar terus dengan hasil OP3 mengorbit mengelilingi Matahari.
Diagram 1 shows the Earth and the Mars that orbiting
darab jisim dua jasad
around the Sun.
The gravitational force is directly proportional to
the product of the masses of the two bodies
A I dan II
I dan II
B II dan III
II and III
C II dan IV
II dan IV
Rajah 1
D I dan IV Diagram 1
I and IV
Berapakah tempoh orbit Marikh?
2 Hitung pecutan graviti di Bumi dengan jisim dan What is the orbital period of the Mars?
OP3 jejari Bumi masing- masing adalah [Jejari orbit Bumi = 1.50 × 1011 m, tempoh orbit
5.97 × 1024 kg dan 6.37 × 106 m. Bumi = 365 hari, jejari orbit Marikh
Calculate the gravitational acceleration on the Earth = 2.28 × 1011 m]
with the mass and radius of the Earth are [Radius of orbit of Earth = 1.50 × 1011 m, orbital
5.97 × 1024 kg and 6.37 × 106 m respectively. period of Earth= 365 days, radius of orbit of Mars
[G = 6.67 × 10–11 N m2 kg–2] = 2.28 × 1011 m]
A 8.9 m s–1 A 400 hari/ days
B 8.8 m s–1 B 648 hari/ days
C 9.8 m s–1 C 548 hari/ days
D 7.9 m s–1 D 684 hari/ days

55 © Penerbit Mahir Sdn. Bhd. (183897-P)

Fizik Tg 4 (Bab 3)6th 9/2/23.indd 55 09/02/2023 4:20 PM


Fizik Tingkatan 4 Bab 3
5 Kapal angkasa Colombus berjisim 3 × 104 kg D Jejari orbit planet adalah sama dengan jarak
OP3 dilancarkan dari Bumi ke Bulan. Jisim Bumi dan purata planet dari Matahari.
Bulan masing-masing adalah 6 × 1024 kg dan The radius of planet’s orbit is the same as the
7.4 × 1022 kg. average value of distance between the planet and
the Sun.
Hitungkan daya graviti antara Bumi dan kapal
angkasa itu apabila jarak kapal angkasa adalah
7 Sebuah satelit ditempatkan dalam satu orbit tetap
1.2 × 109 m dari pusat Bumi.
A spacecraft Columbus has mass of 3 × 104 kg is
OP4 mengelilingi Bumi. Satelit itu boleh berada dalam
launched from the Earth to the Moon.
KBAT orbitnya tanpa terlepas bebas ke ruang angkasa
Calculate the gravitational force between the kerana
spacecraft and the Earth when the distance between A satellite is positioned in a fix orbit around the Earth.
the spacecraft is 1.2 × 109 m from the center of the The satellite is orbiting without escaping freely in the
Earth. space because
A 2.06 × 1019 N C 0.103 N A ia telah menjadi tidak berberat.
B 8.338 N D 3.338 N it has been weightless.
B ia sedang bergerak dalam vakum.
6 Pernyataan yang manakah betul tentang Hukum it is moving in vacuum.
OP2 Kepler Kedua? C ia sedang bergerak dengan kelajuan yang
Which statement is correct about Kepler’s Second sangat tinggi.
Law? it is moving at a very high speed.
A Planet mengelilingi Matahari dalam orbit D ia sentiasa ditarik oleh daya graviti Bumi.
berbentuk elips, dan Matahari berada pada it always pulled by the Earth’s gravitational force.
satu fokus.
Planets orbiting the Sun in elliptical orbits with 8 Apakah data-data yang diperlukan untuk
the Sun at one focus. OP2 menghitung jisim Matahari?
B Kuasa dua tempoh orbit berkadar terus What is the data required to calculate the mass of the
Sun?
dengan kuasa tiga jarak purata planet dari
A Tempoh peredaran dan jejari orbit Bulan
Matahari
Period of revolution and radius of orbit of the
The square of the orbital period of planet is
Moon
directly proportional to the cube of the radius of
its orbit. B Tempoh peredaran dan jejari orbit Bumi
Period of revolution and radius of orbit of the
C Satu garis yang menyambungkan sebuah
Earth
planet ke Matahari mencakupi luas yang
C Tempoh peredaran dan jejari orbit satelit
sama dalam sela masa yang sama Period of revolution and radius of orbit of the
A line that connects a planet to the Sun sweeps out
satellite
equal areas in equal times.

© Penerbit Mahir Sdn. Bhd. (183897-P) 56

Fizik Tg 4 (Bab 3)6th 9/2/23.indd 56 09/02/2023 4:20 PM


Fizik Tingkatan 4 Bab 3

KERTAS 2
Bahagian A
1 Rajah 1.1 menunjukkan sebuah roket yang ingin mendarat di permukaan Bulan. Diberi pemalar kegravitian =
6.67 x 10-11 N m2 kg-2, jisim Bulan = 7.35 × 1022 kg dan jejari Bulan = 1.75 × 106 m.
Diagram 1.1 shows a rocket trying to land on the surface of the Moon. Given that the gravitational constant =
6.67 × 10–11 N m2 kg–2, the mass of the Moon = 7.35 × 1022 kg and the radius of the Moon = 1.75 × 106 m.

Rajah 1.1
Diagram 1.1

(a) Hitungkan OP4


Calculate

(i) pecutan graviti di permukaan Bulan.


the acceleration due to gravity on the surface of the Moon.
GM
g=
r2
(6.67 × 10–11)(7.35 × 1022)
=
(1.75 × 106)2
= 1.60 N

[2 markah/ marks]

(ii) halaju lepas bagi roket jika ia ingin meninggalkan permukaan Bulan.
the escape velocity of the rocket if to leave the surface of the Moon.
2GM
v=
r

2(6.67 × 10–11)(7.35 × 1022)


=
1.75 ×106
= 2.37 × 103 N

[2 markah/ marks]

57 © Penerbit Mahir Sdn. Bhd. (183897-P)

Fizik Tg 4 (Bab 3)6th 9/2/23.indd 57 09/02/2023 4:20 PM


Fizik Tingkatan 4 Bab 3
(b) Jadual 1 menunjukkan ciri-ciri bagi tiga satelit P, Q dan R yang direka oleh sebuah syarikat komunikasi
untuk menerima isyarat siaran langsung bagi sebuah stesen televisyen.
Table 1 shows the characteristics of three satellites P, Q and R designed by a communication company to receive
live broadcast for a television station.

Satelit
P Q R
Satellite

Sumber tenaga satelit Tenaga nuklear Arang batu/ Gas Tenaga solar
Source of satellite energy Nuclear energy Coal/ Gas Solar energy

Gelombang
Jenis gelombang digunakan Gelombang radio Gelombang mikro
inframerah
Type of wave used Radio wave Microwave
Infrared wave

Jadual 1
Table 1

Berdasarkan Jadual 1, nyatakan ciri-ciri yang sesuai bagi satelit yang boleh berfungsi dengan cekap
sebagai sebuah satelit komunikasi. Berikan sebab untuk kesesuaian ciri tersebut. OP4 KBAT
Based on Table 1, state the suitable characteristics for the satellite that can function efficiently as a communication
satellite. Give reason for the suitability of the characteristics.

(i) Sumber tenaga satelit.


Source of satellite energy.
Tenaga solar/ Solar energy

Sebab / Reason:
Tenaga boleh diperbaharui/ Renewable energy
[2 markah/ marks]

(ii) Jenis gelombang untuk komunikasi.


Type of wave for communication.
Gelombang mikro/ Microwave

Sebab / Reason:
Frekuensi tinggi// Kuasa penembusan tinggi

High frequency// High penetration energy


[2 markah/ marks]

(c) Berdasarkan jawapan di 1 (b), tentukan satelit yang paling sesuai untuk dibina sebagai satelit
komunikasi. OP5 KBAT
Based on the answers in 1 (b), determine the most suitable satellite to be designed as communication satellite.
Satelit R/ Satellite R
[1 markah/ mark]

© Penerbit Mahir Sdn. Bhd. (183897-P) 58

Fizik Tg 4 (Bab 3)6th 9/2/23.indd 58 09/02/2023 4:20 PM


Fizik Tingkatan 4 Bab 4

BAB
4
Haba
Heat

Tema Haba
Theme Heat

NOTA EFEKTIF

1 Keseimbangan terma/ Thermal equilibrium The amount of heat, Q required during a change of
(a) Tiada pemindahan bersih haba antara dua phase of 1 kg substance at constant temperature.
objek yang berada dalam keseimbangan
Q
terma. l=
There is no net flow of heat between two objects
m
that are in thermal equilibrium. (a) Haba pendam tentu pelakuran/ Specific
(b) Dua objek yang berada dalam latent heat of fusion, lf
keseimbangan terma mempunyai suhu Kuantiti haba yang diperlukan untuk
yang sama. menukarkan 1 kg bahan daripada pepejal
Two objects in thermal equilibrium have the same kepada cecair tanpa perubahan suhu.
temperature.
The amount of heat, Q required to change 1 kg of
2 Muatan haba tentu/ Specific heat capacity, c the substance from solid to liquid phase without a
Kuantiti haba yang diperlukan untuk menaikkan change in temperature.
suhu 1 kg bahan sebanyak 1 °C. (b) Haba pendam tentu pengewapan/
The quantity of heat needed to raise the temperature of Specific latent heat of vaporisation, lv
1 kg mass of a substance by 1 °C.
Kuantiti haba yang diperlukan untuk
Q menukar 1 kg bahan daripada cecair
c= kepada gas tanpa perubahan suhu.
m∆θ
The amount of heat required to change 1 kg
3 Haba pendam tentu/ Specific latent heat, l of the substance from the liquid to gaseous
phase without a change in temperature.
Kuantiti haba, Q yang diperlukan untuk
mengubah fasa bagi 1 kg bahan pada suhu malar.

Hukum Gas/ Gas Laws

Hukum Boyle Hukum Charles Hukum Gay-Lussac


Boyle’s law Charles’ law Gay-Lussac’s law

Bagi suatu gas yang jisimnya Bagi suatu gas yang berjisim Bagi suatu gas yang berjisim
tetap, tekanan gas, P itu adalah tetap, isi padu gas, V berkadar tetap, tekanan gas, P berkadar
berkadar songsang dengan isi terus dengan suhu mutlak, T terus dengan suhu mutlak, T
padu, V apabila suhu gas, T apabila tekanan gas, P kekal apabila isi padu, V gas kekal
kekal malar. malar. malar.
For a fixed mass of gas, the pressure of For a fixed mass of gas, the volume of For a fixed mass of gas, the pressure
the gas, P is inversely proportional to the gas, V is directly proportional to of the gas, P is directly proportional to
its volume, V when the temperature, T its absolute temperature, T when its its absolute temperature, T when its
is kept constant. pressure, P is kept constant. volume, V is kept constant.
1 V∝T P∝T
P∝
V V V1 V2 P P1 P2
P1V1 = P2 V2 = pemalar = = pemalar =
PV = pemalar T constant T constant T1 T2
T1 T2
constant

59 © Penerbit Mahir Sdn. Bhd. (183897-P)

Fizik Tg 4 (Bab 4)6th 9/2/23.indd 59 09/02/2023 4:20 PM


Fizik Tingkatan 4 Bab 4

Keseimbangan Terma
4.1
Thermal Equilibrium SB-16
SB-17
Buku Teks: m.s 120-123
1 Rajah menunjukkan susunan radas bagi membuktikan keseimbangan terma berlaku di antara dua jasad yang
bersentuhan secara terma.
15 The diagram shows the apparatus arrangement to prove the thermal equilibrium took place between two bodies that are
thermally in contact.

Kaki retort
Retort stand

Termometer A Termometer B

°c

°c
42

42
Thermometer A Thermometer B

41

41
40

40
39

39
38

38
37

37
Kertas tisu
36

36
Tissue paper
35

35
Air paip
Air panas
Tap water
Hot water
Bikar A Bikar B
Beaker A Beaker B

Selepas 10 minit, bacaan kedua-dua termometer A dan B adalah sama.


After 10 minutes, the reading of both thermometers ASB-19
and B is the same.

(a) Huraikan perubahan suhu air paip dan air panas. TP 3 OP4

18 Explain the change in temperature for tap water and hot water.
Apabila air panas dan air paip bersentuhan secara terma, suhu air panas akan menurun, manakala suhu

air sejuk akan meningkat sehingga suhu kedua-dua air menjadi sama.

When the tap water and the hot water are in thermal contact, the temperature of the hot water decreases while the

temperature of tap water increases until the temperature of both water become the same.

(b) Bagaimanakah pemindahan haba berlaku dalam rajah di atas?


SB-22
TP 3 OP4
How does the heat transfer happened in the diagram above?
SB-21
Haba dipindahkan daripada air panas di dalam bikar ke air paip dengan kadar tinggi. Haba dipindahkan

daripada air paip ke air panas di dalam bikar dengan kadar rendah.

Heat is transferred from hot water in the beaker to the tap water with high rate. Heat is transferred from the tap

water to the hot water in the beaker with low rate.

(c) Apakah yang berlaku kepada kadar pemindahan haba selepas 10 minit? TP 4 OP5
What happen to the rate of heat transfer after 10 minutes?
Haba dipindahkan pada kadar yang sama antara air panas dan air paip.

Heat is transferred at the same rate between the hot water and the tap water.

(d) Nyatakan nama bagi keadaan di 1 (c). TP 1 OP2


State the name for the state in 1 (c).
24 Keseimbangan terma/ Thermal equilibrium

© Penerbit Mahir Sdn. Bhd. (183897-P) 60


SB-23

Fizik Tg 4 (Bab 4)6th 9/2/23.indd 60 09/02/2023 4:20 PM


MODEL ZON UTARA FIZIK

Fizik Tingkatan 4 Bab 4


2 Apakah maksud keseimbangan terma? TP 1 OP2
What is the meaning of thermal equilibrium?
Tiada pemindahan haba bersih antara dua objek// Pemindahan bersih haba sifar

There is no net flow of heat between two objects// The net flow of heat is zero

3 Jelaskan keseimbangan terma yang berlaku dalam kehidupan harian berikut. TP 4 OP5
Explain the thermal equilibrium took place in the following daily life.

(a) Tuala basah diletakkan di atas dahi seseorang yang mengalami demam panas. s - aw08
A wet towel is placed on the forehead of a person who has high fever.

Tuala basah
Wet towel

s - aw09

Pada awalnya, suhu tuala basah lebih rendah daripada suhu badan seseorang. Haba dipindahkan

daripada dahi ke tuala sehingga capai keseimbangan terma. Maka, tenaga haba seseorang dikurangkan.

Suhu badan akan turut berkurang.

Initially the temperature of the wet towel is lower than the body temperature of the person. Heat energy is

transferred from the forehead to the towel until thermal equilibrium is reached. Then, the heat energy of the person

will reduce. The body temperature also will reduce.

(b) Minuman panas disejukkan dengan menambah beberapa ketul kiub ais ke dalam minuman.
A hot drink can be cooled by adding a few ice cubes to the drink.
Haba dari air panas dipindahkan ke ais kiub yang sejuk sehingga capai keseimbangan terma antara ais

dan air minuman. Kiub ais cair. Suhu air minuman berkurang dan minuman menjadi sejuk.
Heat from the hot drink is transferred to the colder ice until thermal equilibrium between the ice and water is

reached. Ice cubes melt. The temperature of the drink decreases and the drink cools.

(c) Makanan disimpan di dalam peti sejuk.


Food store in a refrigerator.
Apabila makanan disimpan di dalam peti sejuk, haba dari makanan mengalir ke udara di dalam peti

sejuk sehingga keseimbangan terma berlaku. Suhu makanan menurun dan makanan kekal segar untuk

tempoh yang lama.

When food is kept in the refrigerator, heat from the food flows to the air in the refrigerator until thermal equilibrium

occurs. Temperature of the food drops and the food stays fresh for a longer period.

61 © Penerbit Mahir Sdn. Bhd. (183897-P)

Fizik Tg 4 (Bab 4)6th 9/2/23.indd 61 09/02/2023 4:20 PM


Fizik Tingkatan 4 Bab 4
4 Menentu ukur sebuah termometer cecair dalam kaca menggunakan dua takat tetap.
Calibrate a liquid in glass thermometer using two fixed points.

(a) Pada skala Celsius, dua takat tetap suhu dipilih iaitu:
At the scale of degree Celsius, two fixed temperature are chosen which are:

(i) Takat lebur ais (ii) Takat didih air


Melting point of ice Boiling point of water

0°C 100°C

(b) Penentu ukuran adalah proses penskalaan pada termometer untuk membuat pengukuran suhu.
a process of making a scale of reading on a thermometer.
Calibration is

5 Rajah menunjukkan sebuah termometer yang menunjukkan bacaan.


The diagram shows a thermometer reading.
0°C θT 100°C

3.6 cm
9.3 cm

13.6 cm

Berapakah nilai suhu θT? TP 3 OP4


What is the value of temperature θT?
L0 = 3.6 cm; L100 = 13.6 cm; LT = 9.3 cm
Lθ – L0
θT = × 100
L –L100 0
9.3 – 3.6
= × 100
13.6 – 3.6
= 57°C

4.2 Muatan Haba Tentu


Specific Heat Capacity

Buku Teks: m.s 125-136

1 Muatan haba, C bagi satu objek ialah kuantiti haba yang diperlukan untuk menaikkan suhunya sebanyak 1°C.
the quantity of heat needed to raise the temperature of the object by 1°C.
Heat capacity, C of an object is

Q Q = Kuantiti haba/ Heat quantity


C= Δθ = Perubahan suhu/ Change in temperature
Δθ

Unit bagi muatan haba adalah J °C–1 .


Unit for heat capacity is J °C–1 .

© Penerbit Mahir Sdn. Bhd. (183897-P) 62

Fizik Tg 4 (Bab 4)6th 9/2/23.indd 62 09/02/2023 4:20 PM


Fizik Tingkatan 4 Bab 4
2 Rajah menunjukkan dua cerek, P dan Q diisi dengan kuantiti air yang berbeza. Kedua-dua cerek itu
dihidupkan untuk panaskan air sehingga mendidih. TP 4SB-18
OP5
The diagram shows two kettles, P and Q filled with difference quantity of water. Both kettles are switched on to heat up
water until boiling.
Cerek elektrik P Cerek elektrik Q
Electric kettle P Electric kettle Q

1 liter air 2 liter air


1 litre of water 2 litre of water

(a) Cerek manakah mendidih lebih lama? SB-20


Which kettle boils for longer time?
Cerek Q/ Kettle Q

(b) Cerek manakah mempunyai muatan haba yang lebih tinggi?


Which kettles has higher heat capacity?
Cerek Q/ Kettle Q

(c) Hubung kait antara jisim bahan dengan muatan haba bahan.
Relate between the mass of substance and the heat capacity of substance.
Semakin besar jisim bahan, semakin besar muatan haba bahan itu.

The greater the mass of a substance, the greater the heat capacity of the substance.

(d) Apakah faktor yang mempengaruhi muatan haba bahan selain jisim?
SB-24
What is the factor influences the heat capacity of substance other than mass?
Peningkatan suhu/ Rise of temperature

3 Jelaskan bagaimana muatan haba diaplikasikan dalam aktiviti harian. TP 5 OP6


Explain how the heat capacity applied in daily activities.

(a) Pemegang logam pada kereta lebih cepat panas berbanding kusyen fabrik kereta.
The metal parts of a car get hot faster compared to the fabric cushion of the car.
Bahagian logam pada kereta mempunyai muatan haba yang lebih kecil berbanding kusyen farik.

Penyerapan haba daripada sinaran Matahari menyebabkan logam mengalami peningkatan suhu yang

lebih tinggi berbanding kusyen fabrik.


SB-25
The metal parts of a car has a lower heat capacity compared to the cushion. Absorption of heat energy from the
SB-26
Sun causes the metal to get hot faster compared to the fabric cushion.

(b) Di pantai, kanak-kanak lebih suka berjalan di kawasan air laut berbanding di atas pasir semasa cuaca
panas terik. Mengapa?
At beach, children prefer to walk on the sea water region compared to the sand during hot day. Why?
Pasir mempunyai muatan haba yang rendah dan cepat menjadi panas manakala air laut mempunyai

muatan haba yang tinggi dan lambat menjadi panas.

Sand has low heat capacity and heats up faster. Water has high heat capacity and heats up slowly.

63 © Penerbit Mahir Sdn. Bhd. (183897-P)

Fizik Tg 4 (Bab 4)6th 9/2/23.indd 63 09/02/2023 4:20 PM


Fizik Tingkatan 4 Bab 4

4 Muatan haba tentu, c ialah haba yang diperlukan bagi 1 kg bahan untuk menaikkan suhu sebanyak 1 °C.

the quantity of heat needed to raise the temperature of 1 kg mass of the substance by 1 °C.
Specific heat capacity, c is

Q Q = Kuantiti haba/ Heat quantity


c= m = Jisim/ Mass
mΔθ
Δθ = Perubahan suhu/ Change in temperature

Unit bagi muatan haba tentu adalah J kg–1 °C–1 atau J kg–1 K–1.
Unit for specific heat capacity is J kg–1 °C–1 or J kg–1 K–1.

5 Satu logam berjisim 2 kg. Hitung kuantiti haba yang dipindahkan ke logam untuk menaikkan suhu daripada
30°C ke 70°C.
A metal has mass 2 kg. Calculate the amount of heat that must be transferred to the metal to raise the temperature from
30°C to 70°C.
[Muatan haba tentu logam = 500 J kg–1 °C–1 ]
[Specific heat capacity of the metal = 500 J kg –1 °C–1]
m = 2 kg; Δθ = 70 –30 = 40°C; c = 500 J kg–1 °C–1
Q = mcΔθ
= 2(500)(40)
= 40 000 J

6 Berapakah kuantiti haba yang diperlukan untuk menaikkan suhu 100 g emas daripada 20°C ke 100°C?
[Muatan haba tentu emas = 129 J kg–1 °C–1]
What is the heat quantity that are required to raise the temperature of 100 g of gold from 20°C to 100°C?
[Specific heat capacity of gold = 129 J kg–1 °C–1]
m =100 g = 0.1 kg; Δθ = 100 – 20 = 80°C; c = 129 J kg–1 °C–1
Q = mcΔθ
= 0.1(129)(80)
= 1 032 J

EKSPERIMEN 4.1

Tujuan: Menentukan muatan haba tentu air


To determine the specific heat capacity of water
Aim:

Radas: Bekalan kuasa, pemanas rendam, bikar, jam randik, termometer, kaki retort, neraca elektronik, air
dan kertas tisu
Power supply, immersion heater, beaker, stopwatch, thermometer, retort stand, electronic beam, water and
Apparatus:
tissue paper

Prosedur/ Procedure:
(i) Balut bikar dengan kertas tisu.
Wrap the beaker with tissue paper.

© Penerbit Mahir Sdn. Bhd. (183897-P) 64

Fizik Tg 4 (Bab 4)6th 9/2/23.indd 64 09/02/2023 4:20 PM


Fizik Tingkatan 4 Bab 4
(ii) Letakkan bikar di atas neraca elektronik dan set semula bacaan neraca itu kepada nilai sifar.
Put the beaker on an electric beam and set the reading of the balance to zero.
(iii) Isi air ke dalam bikar sehingga 3/4 penuh.
Fill water in the beaker until 3/4 full.
(iv) Ambil bacaan jisim air, m yang ditunjukkan oleh neraca elektronik.
Take the reading of water, m as shown on the electronic balance.
(v) Ambil bacaan suhu awal air, θ1. Kaki retort
Take the reading of the water initial temperature, θ1. Retort stand
(vi) Hidupkan pemanas rendam dan pada masa
Termometer Pemanas rendam
yang sama mulakan jam randik. Immersion heater
Switch on the heater and start the stopwatch. Thermometer Bekalan kuasa
Air Power suppy
(vii) Selepas lima minit, matikan pemanas rendam. Water
Ambil bacaan termometer tertinggi sebagai Bikar
suhu akhir θ2. Beaker
After five minutes, turn off the immersion heater. P

Take the reading of the highest temperature as the


final temperature, θ2. Kertas tisu
Tissue paper
Keputusan/ Result:

Kuasa pemanas rendam, P


36 W
Power of immersion heater

Jisim air, m
0.42 kg
Mass of water

Masa pemanasan, t
Heating time 5 × 60 = 300 s

Peningkatan suhu, θ = θ2 – θ1
33°C – 27°C = 6°C
Rise of temperature

Haba dibekalkan oleh pemanas rendam, Q = Pt = 36 × 300 = 10 800 J


Heat supplied by immersion heater
Haba diserap oleh air, Q = mcθ = 10 800 J
Heat absorbed by water
Andaikan tiada haba hilang ke persekitaran, maka muatan haba tentu air,
Assume no heat loss to surrounding, then the specific heat capacity of water,
Pt
Pt = mcθ → c =

10 800
=
(0.42)(6)
= 4 286 J kg–1 °C–1

Perbincangan/ Discussion:
(i) Mengapakah bikar perlu dibalut dengan kain?
Why does the beaker need to be wrapped with cloth?
Mengurangkan kehilangan haba ke persekitaran/ To reduce heat lost to the surrounding

(ii) Mengapakah suhu air akhir, θ2 tidak diambil sebaik sahaja masa pemanasan lima minit tamat?
Why the final water temperature, θ2, not taken as soon as the five minutes heating time ends?
Memastikan keseimbangan terma antara air dan pemanas dicapai.

To ensure thermal equilibrium is achieved between the water and the heater.

65 © Penerbit Mahir Sdn. Bhd. (183897-P)

Fizik Tg 4 (Bab 4)6th 9/2/23.indd 65 09/02/2023 4:20 PM


Fizik Tingkatan 4 Bab 4

EKSPERIMEN 4.2

Tujuan: Menentukan muatan haba tentu aluminium


To determine the specific heat capacity of aluminium
Aim:

Radas: Bekalan kuasa, pemanas rendam, jam randik, termometer, 1 kg bongkah aluminium, sedikit
minyak masak dan kain felt
Power supply, immersion heater, stopwatch, thermometer, 1 kg of aluminum block, a little cooking oil and
Apparatus:
felt cloth

Prosedur/ Procedure:
(i) Susun radas seperti dalam rajah.
Set up the apparatus as shown in the diagram.
(ii) Ambil bacaan suhu awal blok aluminium, θ1. Pemanas rendam Bekalan kuasa
Termometer Immersion heater Power suppy
Take the reading of the initial temperature of the Thermometer
aluminium block, θ1.
(iii) Hidupkan pemanas rendam dan pada masa Blok
yang sama, mulakan jam randik. alumminium
Switch on the immersion heater and at the same Aluminium P

time start the stopwatch. block


(iv) Selepas masa lima minit, matikan pemanas
rendam. Ambil bacaan tertinggi termometer Kain felt
sebagai suhu akhir aluminium blok, θ2. Felt cloth
After five minutes, turn off the heater. Take the reading of the highest temperature as the final
temperature of aluminium block, θ2.
Keputusan/ Result:
Kuasa pemanas rendam, P
50 W
Power of immersion heater

Jisim blok aluminium, m


1 kg
Mass of aluminium block

Masa pemanasan, t
5 × 60 = 300 s
Heating time

Peningkatan suhu, θ = θ2 – θ1
42°C – 27°C = 15°C
Rise of temperature

Haba dibekalkan oleh pemanas rendam, Q = Pt = 50 × 300 = 15 000 J


Heat supplied by immersion heater

Haba diserap oleh blok aluminium, Q = mcθ = 15 000 J


Heat absorbed by aluminium block

Andaikan tiada haba hilang ke persekitaran, maka muatan haba tentu aluminium,
Assume no heat loss to surrounding, then the specific heat capacity of aluminium,
Pt
Pt = mcθ → c =

15 000
=
(1)(15)
= 1 000 J kg–1 °C–1

© Penerbit Mahir Sdn. Bhd. (183897-P) 66

Fizik Tg 4 (Bab 4)6th 9/2/23.indd 66 09/02/2023 4:20 PM


Fizik Tingkatan 4 Bab 4
Perbincangan/ Discussion:
(i) Apakah yang boleh dilakukan untuk memperoleh sentuhan terma yang lebih baik antara bebuli
termometer dengan blok aluminium?
What can be done to obtain a better thermal contact between the bulb of the thermometer and the aluminium block?
Letakkan sedikit minyak masak ke dalam lubang./ Put a small amount of oil in the hole.

(ii) Diberi muatan haba tentu aluminium adalah 900 J kg–1 °C–1. Bandingkan nilai muatan haba tentu
daripada eksperimen dengan nilai sebenar yang diberi. Terangkan.
Given specific heat capacity of aluminium is 900 J kg–1 °C–1. Compare the value of specific heat capacity of
aluminium from the experiment with the actual value given. Explain.
Nilai muatan haba tentu daripada eksperimen lebih besar daripada nilai sebenar. Hal ini kerana terdapat

kehilangan haba.

The value of specific heat capacity is larger than the actual value. This is due to the loss of heat.

7 Aplikasi Muatan Haba Tentu TP 5 OP6


Applications of Specific Heat Capacity

(a) Periuk memasak


Cooking pot
Badan
Body Pemegang
Handle
Tapak
Base

Bahagian periuk Bahan Muatan haba tentu Alasan


Part of the pot Material Specific heat capacity Reason

(i) Tapak Kuprum Rendah Cepat panas


Base Copper Low Heats up very quickly

(ii) Badan Aluminium Rendah Cepat panas


Body Aluminium Low Heats up very quickly

(iii) Pemegang Plastik Tinggi Konduktor haba lemah


Handle Plastic High Poor heat conductor

(b) Air digunakan sebagai agen penyejuk dalam sistem radiator kereta kerana:
Water used as cooling agent in car radiator system because of:

Ciri-ciri Alasan
Characteristics Reason

(i) Takat didih tinggi Tidak mudah menjadi gas


High boiling point Not change to gas easily

(ii) Muatan haba tentu tinggi Menyerap lebih banyak haba


High specific heat capacity Absorb more heat

(iii) Mudah didapati Murah dan jimat


Available Cheap and cost saving

67 © Penerbit Mahir Sdn. Bhd. (183897-P)

Fizik Tg 4 (Bab 4)6th 9/2/23.indd 67 09/02/2023 4:20 PM


Fizik Tingkatan 4 Bab 4
(c) Perbandingan antara bayu laut dan bayu darat. TP 4 OP5
Comparison between sea breeze and land breeze.

Bayu laut Bayu darat


Sea breeze Land breeze
Udara panas
Udara panas Hot air
Hot air

Udara sejuk Udara sejuk


Cold air Cold air

Laut (Sejuk)
Daratan (Panas) Daratan (Sejuk) Sea (Cold)
Land (Hot) Land (Cold)

Berlaku pada waktu siang. Berlaku pada waktu malam.


Occurs during daytime. Occurs at night.

Muatan haba tentu laut tinggi . Muatan haba tentu laut rendah .
high low
Sea has specific of heat capacity. Sea has specific of heat capacity.

Muatan haba tentu darat rendah . Muatan haba tentu daratan tinggi .
low high
Land has specific of heat capacity. Land has specific of heat capacity.

Daratan lebih panas daripada laut. Laut lebih panas daripada laut.
warmer warmer
Land is than sea. Sea is than land.

Udara panas di daratan naik ke atas. Udara panas di laut naik ke atas.
land sea
Hot air on rises up. Hot air above the rises up.

Udara sejuk dari laut bergerak ke Udara sejuk dari daratan bergerak ke
daratan. laut.
Cold Cold
air moves toward the land. air moves towards the sea.

8 Sebuah cerek elektrik mempunyai kuasa 1 000 W. Hitung TP 3 OP4


An electric kettle is rated 1 000 W. Calculate

(a) kuantiti haba yang dijanakan dalam masa 5 minit.


the quantity of heat generated in 5 minutes.
Q = Pt = 1 000 × 5 × 60
= 300 000 J

(b) kenaikan suhu bagi 750 g air jika cerek elektrik itu dihidupkan selama 5 minit.
[Muatan haba tentu air = 4 200 J kg–1 °C–1]
the rise in temperature of 750 g of water if the electric kettle is switched on for 5 minutes.
[Specific heat capacity of water is 4 200 J kg-1 °C-1]
Q = mcθ
300 000 = (0.75)(4 200)θ
θ = 95.24°C

© Penerbit Mahir Sdn. Bhd. (183897-P) 68

Fizik Tg 4 (Bab 4)6th 9/2/23.indd 68 09/02/2023 4:20 PM


Fizik Tingkatan 4 Bab 4

4.3 Haba Pendam Tentu


Specific Latent Heat

Buku Teks: m.s 137-147


1 Apakah haba pendam? TP 1 OP2
What is latent heat?

Haba yang diserap atau dibebaskan semasa perubahan fasa tanpa perubahan suhu.
Heat that is absorbed during melting and boiling without change in temperature.

2 Lengkapkan proses perubahan fasa jirim dalam rajah di bawah. TP 1 OP2


Complete the process of phase change in the diagram below.

Haba pendam diserap


Latent heat is absorbed

Peleburan Pendidihan
Melting Boiling

Solid Cecair Gas


Solid Liquid Gas

Pembekuan Kondensasi
Freezing Condensation

Haba pendam dibebaskan


Latent heat is released

3 Haba pendam tentu, l ialah haba diserap atau dibebaskan oleh 1 kg bahan untuk berubah fasa tanpa perubahan
suhu .
Specific latent heat, l is heat absorbed or released for 1 kg substance to change phase without change in temperature .

Q Q = Kuantiti haba/ Heat quantity


l= m = Jisim objek/ Mass of object
m

Unit bagi haba pendam tentu adalah J kg–1 .


Unit for specific latent heat is J kg–1 .

4 Haba pendam tentu pelakuran, lf ialah haba diserap bagi 1 kg bahan untuk berubah dari pepejal ke cecair tanpa
perubahan suhu .
heat absorbed for 1 kg substance to change from solid to liquid without change
Specific latent heat of fusion, lf is
in temperature
.

69 © Penerbit Mahir Sdn. Bhd. (183897-P)

Fizik Tg 4 (Bab 4)6th 9/2/23.indd 69 09/02/2023 4:20 PM


Fizik Tingkatan 4 Bab 4

5 Haba pendam tentu pengewapan, lv ialah haba diserap oleh 1 kg bahan untuk berubah dari cecair ke gas tanpa
perubahan suhu .
heat absorbed for 1 kg substance to change from liquid to gas wout change in
Specific latent heat of vaporisation, lv is
temperature
.

6 Lengkung pemanasan
Heating curve
Suhu/ °C
Temperature/ °C

Gas
Q
Air + Gas
Water + Gas
Takat didih
O Air + Gas P
Boilling point Air
Water Water + Gas

Ais + Air
Ice + Water
Takat lebur
Melting point Ais M N
Ice

L
Masa/minit
Time/minute

Fasa Haba Suhu Tenaga kinetik molekul Keadaan


Phase Heat Temperature Kinetic energy of molecule State

Diserap Bertambah Bertambah Pepejal


LM
Absorbed Increases Increases Solid

Diserap Malar Malar Pepejal dan cecair


MN
Absorbed Constant Constant Solid and liquid

Diserap Bertambah Bertambah Cecair


NO
Absorbed Increases Increases Liquid

Diserap Malar Malar Cecair dan gas


OP
Absorbed Constant Constant Liquid and gas

Diserap Bertambah Bertambah Gas


PQ
Absorbed Increases Increases Gas

© Penerbit Mahir Sdn. Bhd. (183897-P) 70

Fizik Tg 4 (Bab 4)6th 9/2/23.indd 70 09/02/2023 4:20 PM


Fizik Tingkatan 4 Bab 4
7 Lengkung penyejukan
Cooling curve
Suhu/ °C
Temperature/ °C

Gas
R
Gas + Air
Gas + Water
Takat didih
Boilling point S T
Air
Water

Air + Ais
Water + Ice
Takat beku
Freezing point U V Ais
Ice

W Masa/minit
Time/minute

Fasa Haba Suhu Tenaga kinetik molekul Keadaan


Phase Heat Temperature Kinetic energy of molecule State

Dibebaskan Berkurang Berkurang Gas


RS
Released Decreases Decreases Gas

Dibebaskan Malar Malar Gas dan cecair


ST
Released Constant Constant Gas and liquid

Dibebaskan Berkurang Berkurang Cecair


TU
Released Decreases Decreases Liquid

Dibebaskan Malar Malar Cecair dan pepejal


UV
Released Constant Constant Liquid and solid

Dibebaskan Berkurang Berkurang Pepejal


VW
Released Decreases Decreases Solid

EKSPERIMEN 4.3
(a) Haba pendam tentu pelakuran ais
Specific latent heat of fusion of ice

Tujuan: Menentukan haba pendam tentu pelakuran ais


To determine the specific latent heat of vaporisation of ice
Aim:

Radas: Pemanas rendam (12 V, 50 W), dua bikar, kaki retort dan pengapit, bekalan kuasa, dua corong turas,
ais dan neraca elektronik.
Immersion heater (12 V, 50 W), two beakers, retort stand and clamps, power supply, two filter funnels, ice
Apparatus:
and electronic balance.

71 © Penerbit Mahir Sdn. Bhd. (183897-P)

Fizik Tg 4 (Bab 4)6th 9/2/23.indd 71 09/02/2023 4:20 PM


Fizik Tingkatan 4 Bab 4

Prosedur/ Procedure:
(i) Ukur jisim bikar 1 dan bikar 2 dan catat bacaan masing-masing sebagai m1 dan m2.
Measure the masses of beaker 1 and beaker 2 and record as m1 and m2 respectively.
(ii) Susun radas seperti dalam rajah. Pada mulanya, kedua-dua bikar tidak diletakkan di bawah corong turas.
Arrange the apparatus as shown in the diagram. At first, both the beakers are not placed under the filter funnels.
(iii) Hidupkan bekalan kuasa untuk set eksperimen dan tunggu sehingga air mengalir melalui corong turas
dengan kadar yang seragam. Letakkan bikar 1 dan bikar 2 di bawah corong turas masing-masing dan
mulakan jam randik.
Switch on the power supply of experiment set and wait until water flows from the filter funnels at a steady rate.
Place beaker 1 and beaker 2 under respective funnels and start the stopwatch.
(iv) Matikan bekalan kuasa selepas 5 minit.
Switch off the power supply after 5 minutes.
(v) Keluarkan bikar. Sukat jisim kedua-dua bikar semula dan bacaan m3 dan m4 masing-masing dicatat.
Remove the beakers. Measure the masses of both beakers again and record as m3 and m4 respectively.
Pemanas rendam
Immersion heater
Ais hancur
Crushed ice
Corong turas Bekalan kuasa
Filter funnels Power supply

Kaki retort
Retort stand P

Neraca elektronik
Set kawalan Electronic balance Set eksperimen
Control set Experiment set
Keputusan/ Result:

Jisim bikar 1/ Mass of beaker 1 m1 = 50 g

Jisim bikar 2/ Mass of beaker 2 m2 = 52 g

Jisim bikar 1 + air/ Mass of beaker 1 + water m3 = 108 g

Jisim bikar 2 + air/ Mass of beaker 2 + water m4 = 78 g

Tenaga dibekalkan oleh pemanas Q = Pt = 36 × 5 × 60


Energy supplied by heater = 10 800 J

Jisim ais cair dalam bikar 1 m3 – m1 = 108 – 50


Mass of melted ice in beaker 1 = 58 g

Jisim ais cair dalam bikar 2 m4 – m2 = 78 – 52


Mass of melted ice in beaker 2 = 26 g

Jisim ais yang cair disebabkan haba pemanas rendam, m = (m3 – m1) – (m4 – m2)
Mass of melted ice due to heat of immersion heater,
m = 158 – 26 = 32 g = 0.032 kg

Haba pendam tentu pelakuran ais/ Specific latent heat of fusion of ice,
Pt 10 800
lf = = = 337 500 J kg–1
m 0.032

© Penerbit Mahir Sdn. Bhd. (183897-P) 72

Fizik Tg 4 (Bab 4)6th 9/2/23.indd 72 09/02/2023 4:20 PM


Fizik Tingkatan 4 Bab 4

Perbincangan/ Discussion:
(i) Terangkan keperluan mempunyai set kawalan dalam eksperimen ini.
Explain the necessity of control set in this experiment.
Untuk memastikan ais cair hanya disebabkan oleh haba dari pemanas sahaja

To ensure that the ice melts due to the heat from the heater only

(ii) Bandingkan nilai haba pendam tentu pelakuran air yang diperolehi dengan nilai sebenar. Jelaskan.
Compare the value of specific heat of fusion of water with the exact value. Explain.
Nilai sebenar ialah 3.34 × 105 J kg–1. Nilai daripada eksperimen lebih tinggi. Ais juga menyerap haba

dari sekeliling, jadi lebih banyak ais cair.

The exact value is 3.34 × 105 J kg–1. The value from the experiment is higher. The ice also absorbs heat from the

surrounding, so more ice will melt.

(b) Haba pendam tentu pengewapan air


Specific latent heat of vaporisation of water

Tujuan: Menentukan haba pendam tentu pengewapan air


To determine the specific latent heat of vaporisation of water
Aim:

Radas: Pemanas rendam (1 000 W), jam randik, neraca elektronik dan air. Pemanas rendam
Apparatus: Immersion heater (1 000 W), stopwatch, electronic balance and water. Immersion heater

Prosedur/ Procedure:
1 Apabila air mendidih, catatkan bacaan awal neraca elektronik dan
mulakan jam randik pada masa yang sama. Air
When the water is boiling, record the initial reading of the electronic Water
balance and start the stopwatch at the same time.
2 Selepas 5 minit, catat bacaan neraca elektronik. Neraca elektronik ON
After 5 minutes, record the reading of the electronic balance. Electronic balance
OFF

Keputusan/ Result:

Jisim awal air/ Initial mass of water m1 = 685 g

Jisim akhir air/ Final mass of water m2 = 565 g

Masa diambil/ Time taken t = 5 × 60 = 300 s

Kuasa pemanas rendam/ Power of immersion heater P = 1 000 W

m = m1 – m2 = 685 – 565
Jisim air mendidih
= 120 g
Mass of boiling water
= 0.12 kg

Haba pendam tentu pengewapan air/ Specific latent heat of vaporisation of water,
Pt 1 000 × 300
lv = = = 2 500 000 J kg–1
m 0.12

73 © Penerbit Mahir Sdn. Bhd. (183897-P)

Fizik Tg 4 (Bab 4)6th 9/2/23.indd 73 09/02/2023 4:20 PM


Fizik Tingkatan 4 Bab 4

Perbincangan/ Discussion:
Adakah nilai haba pendam tentu pengewapan yang diperolehi dalam eksperimen lebih kecil atau lebih besar
daripada nilai sebenar? Beri sebab.
Is the value of specific latent heat of vaporisation from this experiment smaller or bigger than the actual value? Give reason.
SB-16
Nilai eksperimen lebih besar kerana air juga menyerap haba dari persekitaran yang menyumbang
SB-17 lebih
banyak stim dilepaskan.

The experiment value is bigger because the water also absorbs heat from surroundings which contributes more steam
15 to be released.

8 Nyatakan contoh bagi setiap perubahan fasa jirim melibatkan haba pendam tentu.
State example of each phase change of matter involve specific latent heat.
°c

°c
42

42
Proses/ Process Contoh/ Example
41

41
40

40
39

39
(a) Peleburan Penyejukan minuman oleh ketulan ais
38

38
37

37

Melting Cooling drinks with ice cubes


36

36
35

35

(b) Penyejatan Sistem penyejukan pendingin udara


Evaporation Cooling system of air conditioner

(c) Pendidihan Penyejukan oleh nitrogen cecair


Boiling Cooling by liquid nitrogen
SB-19
(d) Kondensasi Proses mengukus makanan
Condensation Process of steaming food
8
9 Sistem penyejukan dalam peti sejuk. TP 5 OP6
Cooling system of refrigerator.
Tiub penyejat
Evaporator tube

4 Bahagian pembeku
VIDEO SB-22
Freezer section

SB-21
3 Injab pengembangan bit.ly/3X9NoYa
Bagaimana peti sejuk berfungsi?
Expansion valve How does refrigerator work?
2 Kondenser
5
Condenser
Udara
Air

1 Pam pemampat
Compressor pump

Proses/ Process Penerangan/ Explanation

1 (a) Memampatkan agen penyejuk (gas) untuk menambah tekanan dan


Pemampat suhunya.
Compressor Compresses cooling agent (gas) to increase pressure and temperature.

4 © Penerbit Mahir Sdn. Bhd. (183897-P) 74

SB-23
Fizik Tg 4 (Bab 4)6th 9/2/23.indd 74 09/02/2023 4:20 PM
Fizik Tingkatan 4 Bab 4

Proses/ Process Penerangan/ Explanation

2
(b) Agen penyejuk membebaskan haba pendam semasa proses kondensasi.
Kondenser Cooling agent (gas) releases latent heat during condensation.
Condenser

(c) Agen penyejuk (cecair) mengalir melalui injap pengembang. Injap ini
mempunyai satu lubang kecil yang membenarkan cecair pada tekanan
3 tinggi disembur ke dalam tiub penyejat yang bertekanan rendah. Cecair
Injap pengembangan berubah menjadi gas.
Expansion valve Cooling agent (liquid) flows through expansion valve. This valve has tiny hole
which allows liquid at high pressure is ejected into the evaporator tube which is at
lower pressure. Liquid changes to gas.
(d) Dalam penyejat, agen penyejuk (cecair) menyejat dan menyerap haba
4 pendam daripada udara di dalam peti sejuk. Bahagian pembeku akan
Bahagian pembeku menjadi dingin.
Freezer section In the evaporator, cooling agent (in liquid) evaporates and absorbs latent heat from
inside the refrigerator.

5
Agen penyejuk (e) Agen penyejuk (gas) mengalir keluar daripada penyejat ke pemampat.
Cooling agent Cooling agent in gas flows out from evaporator to the compressor.

10 Menyelesaikan masalah melibatkan haba pendam tentu. TP 4 OP5


Solving problem involving specific latent heat.

(a) Sebuah pemanas elektrik 500 W digunakan untuk mendidihkan air. Berapakah masa diperlukan untuk
mengurangkan jisim air sebanyak 70 g selepas air mencapai takat didihnya.
A 500 W electric heater is used to boil water. What is the time required to reduce the mass of water by 70 g after the
water has reached its boiling point?
[Haba pendam tentu pengewapan air = 2.26 × 106 J kg–1]
[Specific latent heat of vaporisation of water = 2.26 × 106 J kg–1]
Pt = mlv
mlv
t=
P
(0.07)(2.26 ×106)
=
500
= 316.4 s

(b) 0.5 kg air pada suhu 30°C dipanaskan sehingga semuanya bertukar ke stim. Berapakah haba yang dibekalkan
itu?
0.5 kg of water of 30°C is heated until all it is converted to steam. What is the amount of heat energy supplied to it?
[Muatan haba tentu air = 4 200 J kg–1 °C–1, haba pendam tentu pengewapan air = 2.26 × 106 J kg-1]
[Specific heat capacity of water = 4 200 J kg–1 °C–1, specific latent heat of vaporisation = 2.26 × 106 J kg–1]
Air (30 °C) Air (100 °C) Stim (100 °C)
Water Q1 Water Q2 Steam
Q1 = mcθ Q2 = mlv
= 0.5 × 4 200 × 70 = 0.5 × 2.26 × 106
= 147 000 J = 1 130 000 J
Q = Q 1 + Q2
= 147 000 + 1 130 000
= 1 277 000 J

75 © Penerbit Mahir Sdn. Bhd. (183897-P)

Fizik Tg 4 (Bab 4)6th 9/2/23.indd 75 09/02/2023 4:20 PM


Fizik Tingkatan 4 Bab 4

4.4 Hukum Gas


Gas Law

Buku Teks: m.s 148-163

INFO
Udara di dalam tayar kereta boleh menyokong berat kereta manakala gas nitrogen di dalam beg udara menahan
pemandu daripada bergerak ke hadapan semasa perlanggaran. Udara di dalam tayar dan beg udara mengenakan
tekanan. Udara yang berada di dalam suatu ruang tertutup mempunyai isi padu dan suhu tetap.
The air in the tyre of a car is able to support the weight of the car while the nitrogen gas in the air bag stops the driver from moving
forward during a crash. The air inside the tyre and air bag exerts a pressure. When the air is in a closed space, it has a certain volume and
temperature.

1 Tekanan, suhu dan isi padu gas. TP 1 OP2


Pressure, temperature and volume of gas.

Kuantiti dan simbol Unit S.I Simbol unit S.I Unit-unit lain
Quantity and symbol S.I unit Symbol of S.I unit Others unit
SUPERB SBP FIZIK TINGKATAN 4 BATCH 2 ARTWORK BY FREELANCER M.ZAIDEE

(a) Tekanan, P
Pascal Pa cm Hg
Pressure

(b) Tekanan,V
Kelvin K °C, °F
Volume
SB-16
SB-17
(c) Tekanan, T
(meter) 3
m 3
mm , cm3
3
Temperature
SB-15

2 Hubungan antara tekanan dengan isi padu bagi suatu gas.


Relationship between pressure and volume of a gas.
°c

°c
42

42
41

41
40

40
39

39
38

38
37

37
36

36
35

35

EKSPERIMEN 4.4
Perhatikan perbezaan saiz gelombang udara yang dibebaskan oleh seekorSB-19
ikan semasa
ia naik ke atas permukaan air.
Observe the change in theSB-18
air bubbles released by a fish as they rise towards the surface of the
water.

Inferens: Isi padu gas mempengaruhi tekanan gas


Volume of gas influences pressure of gas
Inference: SB-22
SB-21
Hipotesis: SemakinSB-20
berkurang isi padu gas, semakin bertambah tekanan gas

The lower the volume of gas, the higher the pressure of gas
Hypothesis:

Tujuan: Mengkaji hubungan antara isi padu gas dengan tekanan gas pada suhu malar
To study the relationship between volume and pressure of gas at constant temperature
Aim:

© Penerbit Mahir Sdn. Bhd. (183897-P) 76


SB-24

SB-23

Fizik Tg 4 (Bab 4)6th 9/2/23.indd 76 09/02/2023 4:20 PM


Fizik Tingkatan 4 Bab 4

Pemboleh ubah dimanipulasi: Isi padu gas


Volume of gas
Manipulated variable:

Pemboleh ubah bergerak balas: Tekanan gas


Pressure of gas
Responding variable:

Pemboleh ubah dimalarkan: Suhu


Temperature
Constant variable:

Radas: Picagari 100 ml, tiub getah, tolok tekanan dan kaki retort.
Apparatus: Syringe of 100 ml, rubber tube, pressure gauge and retort stand

Prosedur/ Procedure:
(i) Tolak omboh supaya isi padu udara, V = 50 ml terperangkap di dalam picagari.
Push the piston so that volume, V = 50 ml of air is trapped inside the syringe.
(ii) Tekanan, P udara di dalam picagari diberikan oleh bacaan tolok tekanan.
The pressure, P of the air inside the syringe is given by the reading on the pressure gauge.
(iii) Eksperimen diulang dengan mengurangkan isi padu udara terperangkap, V = 40 ml, 30 ml, 20 ml dan
10 ml.
The experiment is repeated by reducing the volume of the trapped air, V = 40 ml, 30 ml, 20 ml and 10 ml.
Daya
Force

Omboh
Piston

Picagari
Syringe

Tolok tekanan
Pressure gauge

Tiub getah
Rubber tube

Keputusan/ Result:
(i) Jadualkan nilai- nilai berikut.
Tabulate the following values.

Isi padu, V/ ml 1 Tekanan, P/ kPa


/ ml–1
Volume V Pressure

10 0.100 1.35

20 0.050 0.65

30 0.033 0.50

40 0.025 0.30

50 0.020 0.15

77 © Penerbit Mahir Sdn. Bhd. (183897-P)

Fizik Tg 4 (Bab 4)6th 9/2/23.indd 77 09/02/2023 4:20 PM


SB-15 SB-15
Fizik Tingkatan 4 Bab 4

(ii) Berdasarkan jadual di atas, lakarkan graf berikut.


Based on the table above, sketch the following graphs.

°c

°c

°c

°c
42

42

42

42
P P

41

41

41

41
40

40

40

40
39

39

39

39
38

38

38

38
37

37

37

37
36

36

36

36
35

35

35

35
1
V
V

SB-19 SB-19
Kesimpulan: Isi padu berkadar songsang dengan tekanan
Volume is inversely proportional to pressure
Conclusion: SB-18 SB-18

(a) Hukum Boyle menyatakan bahawa isi padu gas berkadar songsang dengan tekanan bagi suatu gas
tetap pada suhu malar .
the volume of the gas is inversely proportional to gas pressure if its mass and temperature are
Boyle’s law states that
constant SB-21 SB-21
.
SB-20 SB-20

P1

P2
V1
V2

(b) Ungkapan matematik bagi hukum Boyle:


SB-24 SB-24
P1V1 = P2V2
The mathematical expression for Boyle’s law:

(c) Sebuah belon kaji cuaca diisi dengan gas helium sehingga isi padu 30 liter dan tekanan 1.0 atm. Belon
itu kemudiannya dilepaskan. Semasa belon itu naik ke atas langit, isi padunya bertambah.
Berapakah tekanan gas helium apabila isi padu belon itu bertambah sehingga 120 liter. TP 3 OP4
A weather balloon is filled with helium gas to a volume of 30 litres of 1.0 atm pressure. The balloon is then release.
As the balloon rises up to the sky, its volume increases.
What is the pressure of the helium gas when the volume of the balloon increases to 120 litres?
Mengikut Hukum Boyle/ According to Boyle’s law:
P1V1 = P2V2 SB-25 SB-25
1 × 30 = P2 × 120
SB-26 SB-26
P2 = 0.25 atm

© Penerbit Mahir Sdn. Bhd. (183897-P) 78

Fizik Tg 4 (Bab 4)6th 9/2/23.indd 78 09/02/2023 4:20 PM


Fizik Tingkatan 4 Bab 4
3 Hubungan antara isi padu dengan suhu bagi suatu gas.
Relationship between volume and temperature of a gas.

EKSPERIMEN 4.5
Sebiji belon dihembuskan udara dan dipasangkan pada mulut botol. Apabila botol diletakkan dalam bikar
berisi air panas, ia akan lebih mengembang.
A balloon is inflated and fix onto the bottle’s mouth. When the bottle is placed in a beaker of hot water the balloon is
inflated further.

Inferens: Suhu gas mempengaruhi isi padu gas


Temperature of gas influences volume of gas
Inference:

Hipotesis: Semakin tinggi suhu gas, semakin besar isi padu gas
The higher the temperature of gas, the higher the volume of gas
Hypothesis:

Tujuan: Mengkaji hubungan antara suhu gas dengan isi padu gas pada tekanan malar
To study the relationship between temperature of gas and volume of gas at constant pressure
Aim:

Pemboleh ubah dimanipulasi: Suhu gas


Temperature of gas
Manipulated variable:

Pemboleh ubah bergerak balas: Isi padu gas// Panjang turus udara di dalam tiub kapilari
Volume of gas// Length of air column in capillary tube
Responding variable:

Pemboleh ubah dimalarkan: Tekanan


Pressure
Constant variable:

Radas: Termometer, tiub kapilari yang mengandungi udara terperangkap oleh merkuri, bikar 500 ml,
termometer, pembaris, penunu Bunsen, tungku kaki tiga, kasa dawai, pengacau, kaki retort, air, ais
dan gelang getah.
Apparatus: Thermometer, capillary tube, beaker, retort stand, Bunsen burner, tripod stand, wire gauze, ruler, sulphuric
acid, water and ice.
Minyak parafin
Paraffin oil
Prosedur/ Procedure:
(i) Ais diletakkan ke dalam air dan berterusan dikacau. Termometer
51

Thermometer
41

Ice is placed into the water and continuously stirred.


31
21

(ii) Rekodkan suhu awal air, θ = 20°C.


11

Tiub kapilari
01

Pembaris
9

Record the initial temperature of the water, θ = 20°C. Capillary tube


8
7

Ruler
(iii) Rekodkan panjang turus udara, L. Ais
6
5

Pengacau
4

Record the length of air column, L. Ice


3
2

Stirrer
(iv) Ulang eksperimen dengan θ = 30°C, 40°C, 50°C,
1 mc 0

Air Bikar
60°C dan 70°C. Water Beaker
The experiment is repeated θ = 30°C, 40°C, 50°C, 60°C
and 70°C.

79 © Penerbit Mahir Sdn. Bhd. (183897-P)

Fizik Tg 4 (Bab 4)6th 9/2/23.indd 79 09/02/2023 4:20 PM


Fizik Tingkatan 4 Bab 4

Keputusan/ Result:
(i) Jadualkan nilai- nilai berikut.
Tabulate the following values.

Suhu, θ / °C Panjang turus udara, L / cm


Temperature Length of air column

20 6.6

30 6.9

40 7.1

50 7.4

60 7.6

70 7.8

(ii) Berdasarkan jadual di atas, lakarkan graf V melawan θ (Kelvin).


Based on the table above, sketch the graphs graph of V against θ (Kelvin).

V/cm3 V/cm3

– 273 0 θ/°C 0 θ/K

(iii) Suhu –273°C, ialah suhu paling rendah yang mungkin dan dikenali sebagai sifar mutlak .
absolute zero
Temperature of –273°C is the possible lowest temperature and known as .

Kesimpulan: Isi padu gas berkadar terus dengan suhu mutlak


Volume of gas is directly proportional to absolute temperature
Conclusion:

(a) Hukum Charles menyatakan bahawa isi padu berkadar terus dengan suhu mutlak bagi suatu gas berjisim

tetap pada tekanan dimalarkan .


the volume is directly proportional to its absolute temperature for a fixed mass of gas at
Charles’ law states that
constant pressure
.

© Penerbit Mahir Sdn. Bhd. (183897-P) 80

Fizik Tg 4 (Bab 4)6th 9/2/23.indd 80 09/02/2023 4:20 PM


SB-21 SB-21

SB-24
Fizik Tingkatan 4 Bab 4
SB-23

T2

T1 V2

V1
SB-25
B-24
SB-26
(b) Ungkapan matematik bagi hukum Charles: SB-23 V1 SB-23
V2
The mathematical expression for Charles’ law: =
T1 T2

(c) Satu silinder mengandungi 200 cm3 gas pada suhu 27°C. Gas itu dipanaskan sehingga suhunya meningkat
sebanyak 30°C.
Jika omboh silinder mengembang di bawah tekanan yang malar, berapakah isi padu akhir gas? TP 3 OP4
A cylinder contains 200 cm3 of gas at a temperature of 27°C. The gas is heated until its temperature increases by 30°C.
If the piston of theSB-25
SB-25 cylinder expands under constant pressure, what is the final volume of the gas?
T1 = 27 + 273 = 300 K; T2 = 30 + 273 = 303 K
Mengikut hukum Charles/ According to Charles’ law:
V1 V2
=
T1 T2
200 V2
=
300 303
V2 = 202 cm3

4 Hubungan antara tekanan dengan suhu bagi suatu gas.


Relationship between pressure and temperature of gas.

EKSPERIMEN 4.6 MODEL ZON UTARA FIZIK aw by zaidee

Sebelum membuat suatu perjalanan yang jauh, tekanan udara dalam tayar diukur.
Selepas perjalanan jauh, didapati bahawa tekanan udara dalam tayar bertambah.
Before going on a long journey, the air pressure in the tyre of a car is measured. After the
journey, it is found that the air pressure in the tyre had increased.

Inferens: Suhu gas mempengaruhi tekanan gas s - aw08

Temperature of gas influences pressure of gas


Inference:
s - aw09

Hipotesis: Semakin tinggi suhu gas, semakin bertambah tekanan gas


The higher the temperature of gas, the higher the pressure of gas
Hypothesis:

Tujuan: Mengkaji hubungan antara suhu gas dengan tekanan gas pada isi padu malar
To study the relationship between temperature and pressure of gas at constant volume
Aim:

Pemboleh ubah dimanipulasi: Suhu gas


Temperature of gas
Manipulated variable:

81 © Penerbit Mahir Sdn. Bhd. (183897-P)

Fizik Tg 4 (Bab 4)6th 9/2/23.indd 81 09/02/2023 4:20 PM


Fizik Tingkatan 4 Bab 4

Pemboleh ubah bergerak balas: Tekanan gas


Pressure of gas
Responding variable:

Pemboleh ubah dimalarkan: Isi padu gas


Volume of gas
Constant variable:

Radas: Termometer, kelalang dasar bulat, bikar besar, termometer, tolok tekanan, penunu Bunsen, tungku
kaki tiga, pangacau, kaki retort, air dan ais.
Apparatus: Thermometer, round bottom flask, beaker, retort stand, Bunsen burner, tripod stand, wire gauze, stirrer,
pressure gauge, retort stand, water and ice.

Prosedur/ Procedure:
(i) Ais diletakkan ke dalam air dan dikacau berterusan.
Ice is placed into the water and these are continuously stirred.
(ii) Rekodkan suhu awal air, θ = 20°C.
Record the initial temperature of the water, θ = 20°C.
(iii) Rekodkan bacaan tolok tekanan.
Record the reading of pressure gauge.
(iv) Ulang eksperimen dengan θ = 30°C, 40°C, 50°C, 60°C dan 70°C.
The experiment is repeated with θ = 30°C, 40°C, 50°C, 60°and 70°C.
Termometer
Thermometer
Pengacau Tiub getah
Stirrer Tolok tekanan
Rubber tube
Pressure gauge

Kelalang dasar
bulat
Round bottom
flask
Penunu Bunsen
Bunsen burner Blok kayu
Wooden block

Keputusan/ Result:
(i) Jadualkan nilai-nilai berikut.
Tabulate the following values.

Suhu, θ / °C Tekanan gas, P / kPa


Temperature Gas pressure

20 109.0

30 112.5

40 116.0

50 119.5

60 122.5

70 126.5

© Penerbit Mahir Sdn. Bhd. (183897-P) 82

Fizik Tg 4 (Bab 4)6th 9/2/23.indd 82 09/02/2023 4:20 PM


38

38

38

38
38

38

37

37

37

37
37

37

36

36

36

36
36

36

35

35

35

35
35

35
Fizik Tingkatan 4 Bab 4
(ii) Berdasarkan jadual di atas, lakarkan graf P melawan θ (Kelvin).
Based on the table above, sketch the graph of P against θ (Kelvin). SB-19 SB-19 SB-19
P/kPa P/kPa
SB-18 SB-18 SB-18

SB-22
– 273 0 θ/°C 0 SB-21 θ/K SB-21 SB-21
SB-20 SB-20 SB-20
Tekanan gas berkadar terus dengan suhu mutlak
Kesimpulan:
Gas pressure is directly proportional to absolute temperature
Conclusion:

(a) Hukum Gay-Lussac menyatakan bahawa tekanan berkadar terus dengan suhu mutlak bagi suatu gas
berjisim tetap pada isi padu dimalarkan .
the pressure is directly proportional to its absolute temperature for a fixed mass of
Gay Lussac’s law states that
gas at constant volume
SB-24 SB-24 . SB-24

SB-23 SB

T1 T2

SB-25 SB-25 SB-25


SB-26 SB-26 SB-26
(b) Ungkapan matematik bagi hukum Gay-Lussac: P1 P2
The mathematical expression for Gay-Lussac’s law: =
T1 T2

(c) Satu tayar motosikal didapati mempunyai tekanan 3.2 atm apabia ia diuji dalam garaj pada suhu
27°C. Dengan andaian isi padu udara dalam tayar adalah malar, berapakah suhu sekeliling jika tayar itu
mempunyai tekanan 2.8 atm? TP 3 OP4
A motorcycle tyre has a pressure of 3.2 atm when it was tested in a garage at a temperature of 27 °C. By assuming that
the volume of the air in the tyre is fixed, what is the surrounding temperature if the tyre has a pressure of 2.8 atm?
T1 = 27 + 273 = 300 K
Mengikut hukum Gay-Lussac/ According to Gay-Lussac’s law:
P1 P2
=
T1 T2
3.2 2.8
=
300 T2
T2 = 262.5 K
= 262.5 – 273
= –10.5°C

83 © Penerbit Mahir Sdn. Bhd. (183897-P)

Fizik Tg 4 (Bab 4)6th 9/2/23.indd 83 09/02/2023 4:20 PM


Fizik Tingkatan 4 Bab 4

Praktis Sumatif
KERTAS 1

1 Apakah konsep yang digunakan dalam pengukuran D Aluminium memerlukan 900 J tenaga haba
OP2 suhu badan manusia menggunakan termometer? untuk menaikkan suhunya sebanyak 1°C.
What is the concept used in the measurement of human Aluminium requires 900 J of heat energy to
body temperature using a thermometer? increase its temperature by 1°C.
A Muatan haba tentu
Specific heat capacity 4 P, Q, R dan S adalah empat pepejal berlainan
B Haba pendam tentu OP4 dengan jisim yang sama. Jadual 1 menunjukkan
Specific latent heat KBAT muatan haba tentu dan takat lebur bagi setiap
C Keseimbangan terma pepejal.
Thermal equilibrium P, Q, R and S are four different solids with the same
D Perolakan haba mass. Table 1 shows the specific heat capacity and the
Heat convection melting point of each solid.

2 Panjang turus merkuri dalam termometer pada Pepejal Muatan haba tentu Takat lebur
OP3 takat ais dan takat stim adalah 5 cm dan 25 cm. Solid Specific heat capacity Melting point
Apabila termometer diletakkan di dalam satu
cecair, panjang turus merkuri ialah 12 cm. P 800 J kg–1 °C–1 200°C
Berapakah suhu cecair itu? Q 600 J kg °C
–1 –1
230°C
The lengths of the mercury column in a thermometer at
the ice point and the steam point are 5 cm and R 700 J kg–1 °C–1 400°C
25 cm respectively. When the thermometer is placed in
a liquid, the length of the mercury column is 12 cm. S 400 J kg–1 °C–1 300°C
What is the temperature of the liquid? Jadual 1
A 30°C C 55°C Table 1
B 35°C D 70°C
Jika haba dibekalkan dengan kadar yang sama,
3 Muatan haba tentu bagi aluminium ialah pepejal yang manakah akan lebur dahulu?
OP3 900 J kg–1 °C–1. If heat is supplied at the same rate which solid will be
Pernyataan manakah yang menerangkan tentang the first to melt?
muatan haba tentu bagi aluminium dengan betul? A P C R
The specific heat capacity of aluminium is 900 J kg–1 °C–1. B Q D S
Which statement explains the specific heat capacity of
aluminium correctly? 5 Satu blok logam dijatuhkan dari ketinggian 82 m.
A 1 kg aluminium menyerap 900 J tenaga haba OP3 Berapakah kenaikan suhu blok logam selepas ia
semasa pemanasan. mengenai tanah?
1 kg of aluminium absorbs 900 J of heat energy A metal block is dropped from a height of 82 m.
during heating. What is the rise in the temperature of the metal block
B Suhu 1 kg aluminium akan meningkat 1°C after it hits the ground?
apabila ia menyerap 900 J tenaga haba. [Muatan haba tentu blok logam = 410 J kg–1 °C–1]
The temperature of 1 kg of aluminium will rise by [Specific heat capacity of the metal block
1°C when it absorbs 900 J of heat energy. = 410 J kg–1 °C–1]
C Suhu 1 kg aluminium akan turun 1°C apabila A 0.5°C
ia menyerap 900 J tenaga haba. B 1.0°C
The temperature of 1 kg of aluminium will drop by C 2.0°C
1°C when it absorbs 900 J of heat energy.
D 4.0°C

© Penerbit Mahir Sdn. Bhd. (183897-P) 84

Fizik Tg 4 (Bab 4)6th 9/2/23.indd 84 09/02/2023 4:20 PM


Fizik Tingkatan 4 Bab 4
6 Rajah 1 menunjukkan graf suhu melawan masa 9 Rajah 2(a) menunjukkan satu silinder
OP5 bagi satu bahan Y yang dipanaskan. OP4 mengandungi gas pada suhu 30°C. Ketinggian
KBAT Diagram 1 shows a graph of temperature against time KBAT silinder ialah P. Rajah 2(b) menunjukkan gas
for a substance Y which is heated. di dalam silinder yang dipanaskan supaya ia
Suhu
mengembang ke sehingga ketinggian menjadi 3P.
Temperature Diagram 2 (a) shows a cylinder containing gas at
temperature 30°C. The height of the cylinder is P.
Diagram 2 (b) shows the gas in the cylinder is heated
U so that it expands to a height of 3P.
S
T
Q
R

P
Masa Gas
Time P
Gas
(30° C)
Rajah 1 (a)
Diagram 1

Apakah yang berlaku di QR?


What is happening at QR?
A Bahan Y sedang dipanaskan
Substance Y is being heated up
B Bahan Y mendidih pada takat didihnya
Gas 3P
Substance Y is boiling at its boiling point
Gas
C Bahan Y sedang cair pada takat leburnya
Substance Y is melting at its melting point
D Bahan Y mengalami perubahan kimia (b)
Substance Y undergoes chemical changes
Rajah 2
7 Haba pendam tentu pelakuran ialah haba yang Diagram 2
OP2 diperlukan untuk Berapakah suhu akhir gas itu?
The specific latent heat of fusion is the heat required to What is the final temperature of the gas?
A mengubah 1 kg ais kepada air pada 0°C. A 90°C
change 1 kg of ice into water at 0°C. B 180°C
B mengubah 1 kg air kepada stim pada 100°C. C 303°C
change 1 kg of water steam at 100°C.
D 636°C
C mengubah suhu 1 kg ais sebanyak 1°C.
change the temperature of 1 kg of ice by 1°C.
10 Suatu gas berjisim tetap di dalam sebuah bekas
D mengubah suhu 1 kg air sebanyak 1°C.
change the temperature of 1 kg of water by 1°C.
OP3berisi padu tetap dipanaskan.
Pernyataan manakah yang betul?
A fixed mass of gas in a container of fixed volume is
8 Berapa banyak haba yang perlu dibekalkan untuk
heated.
OP3 mengubah 500 g blok ais pada 0°C kepada air Which statement is correct?
pada 0°C? A Tekanan gas adalah malar.
How much heat is needed to be supplied to change The pressure of the gas is constant.
500 g of ice block at 0 °C to water at 0 °C?
B Molekul gas bergerak lebih jauh antara satu
[Haba pendam tentu pelakuran ais
sama lain.
= 3.36 × 105 J kg–1] The gas molecules move further apart.
[Specific latent heat of fusion of ice
C Tenaga kinetik molekul gas adalah malar.
= 3.36 × 105 J kg –1]
The kinetic energy of the gas molecules is
A 6.72 × 102 J constant.
B 1.68 × 105 J D Molekul gas bergerak dengan halaju yang
C 6.72 × 105 J lebih tinggi.
D 1.68 × 108 J The gas molecules move with higher velocity.

85 © Penerbit Mahir Sdn. Bhd. (183897-P)

Fizik Tg 4 (Bab 4)6th 9/2/23.indd 85 09/02/2023 4:20 PM


Fizik Tingkatan 4 Bab 4
11 Rajah 3 menunjukkan satu picagari Bacaan pada tolok tekanan ialah 1.0 × 105 Pa
OP3 disambungkan kepada tolok tekanan.
apabila isi padu udara di dalam picagari ialah
Diagram 3 shows a syringe connected to pressure gauge. 40 cm3.
Berapakah bacaan baharu apabila isi padu udara
Omboh
Piston
ialah 10 cm3?
The reading on the Pressure gauge is 1.0 × 105 Pa
Picagari
when the volume of air in the syringe is 40 cm3.
Syringe
What is the new reading on the pressure gauge when
Udara the volume of air is 10 cm3?
Tolok tekanan
Air Pressure gauge A 4.0 × 105 Pa
B 2.5 × 105 Pa
C 1.0 × 10-5 Pa
Rajah 3 D 4.0 × 10-5 Pa
Diagram 3

KERTAS 2
Bahagian A
1 Rajah 1.1 dan Rajah 1.2 menunjukkan satu eksperimen dijalankan untuk menyiasat satu hukum gas. Satu
turus udara diperangkap dalam tiub kapilari dan dipanaskan untuk beberapa ketika.
Diagram 1.1 and Diagram 1.2 show an experiment is carried out to investigate a law of gas. A column of air is trapped
in the capillary tube and is heated for a period of time.
Termometer
Thermometer

Tiub kapilari
Capillary tube

Air
Water
Asid sulfurik
Sulphuric acid
Udara terperangkap
Trapped air
Penunu bunsen
Bunsen burner

Rajah 1.1 Rajah 1.2


Diagram 1.1 Diagram 1.2

(a) Apakah maksud haba? OP2


What is the meaning of heat?
Satu bentuk tenaga/ A form of energy
[1 markah/mark]
(b) (i) Bandingkan panjang turus udara terperangkap di dalam tiub kapilari dalam Rajah 1.1 dan Rajah 1.2.
Compare the length of the air trapped in the capillary tube in Diagram 1.1 and Diagram 1.2. OP3

Panjang turus udara terperangkap di dalam tiub kapilari dalam Rajah 1.2 lebih panjang daripada

Rajah 1.1.

The length of air trapped in the capillary tube in Diagram 1.2 is longer than in Diagram 1.1.

[1 markah/mark]
© Penerbit Mahir Sdn. Bhd. (183897-P) 86

Fizik Tg 4 (Bab 4)6th 9/2/23.indd 86 09/02/2023 4:20 PM


Fizik Tingkatan 4 Bab 4
(ii) Bandingkan suhu udara terperangkap dalam tiub kapilari dalam Rajah 1.1 dan Rajah 1.2. OP3
Compare the temperature of air trapped in the capillary tube in Diagram 1.1 and Diagram 1.2.
Suhu udara terperangkap di dalam tiub kapilari dalam Rajah 1.2 lebih tinggi daripada Rajah 1.1

The temperature of air trapped in the capillary tube in Diagram 1.2 is higher than Diagram 1.1
[1 markah/mark]

(iii) Namakan kuantiti fizik yang diwakili oleh panjang turus udara terperangkap. OP2
Name physical quantity represented by the length of air trapped.
Isi padu/ Volume
[1 markah/mark]

(iv) Apakah hubungan antara kuantiti fizik di 1(b)(iii) dengan suhu? OP3
What is the relationship between the physical quantity in 1(b)(iii) with temperature?
Berkadar terus/ Directly proportional
[1 markah/mark]

(c) Namakan hukum fizik yang terlibat di 1(b)(iv). OP2


Name a physics law involved in 1(b)(iv).
Hukum Charles/ Charles’ law
[1 markah/mark]

(d) Rajah 1.3 menunjukkan graf yang didapati daripada eksperimen. Suhu, θ ialah –273°C apabila
diekstrapolasi dan memotong paksi x.
Diagram 1.3 shows a graph obtained from the experiment. The temperature, θ is –273°C when it is extrapolated
and intersect x-axis.

V/ cm3

– 273°C 0 θ/ °C

Rajah 1.3
Diagram 1.3

(i) Apakah nama yang diberikan kepada suhu θ = –273°C? OP2


What is the name given to the temperature θ = –273°C?
Sifar mutlak/ Absolute zero
[1 markah/mark]

(ii) Nyatakan satu ciri gas pada suhu θ = –273°C. OP2


State a characteristic of gas at temperature θ = –273°C.
Molekul gas pegun/ Gas molecules stationary
[1 markah/mark]

87 © Penerbit Mahir Sdn. Bhd. (183897-P)

Fizik Tg 4 (Bab 4)6th 9/2/23.indd 87 09/02/2023 4:20 PM


Fizik Tingkatan 4 Bab 4
Bahagian B

2 Rajah 2.1 menunjukkan satu bongkah logam, M berjisim 500 g dipanaskan dengan cepat ke dalam air yang
mendidih bersuhu 100°C dalam jangka masa yang lama. Bongkah logam M kemudiannya dipindahkan
dengan cepat ke dalam cawan polistirena yang berisi 200 g air bersuhu 30°C. Air itu dikacau sehingga
keseimbangan terma tercapai.
Diagram 2.1 shows a metal block, M, of mass 500 g heated in boiling water of temperature 100°C for a long time. The
metal block M is then quickly transferred into 200 g of water at 30°C in a polystyrene cup. The water is stirred until
thermal equilibrium is achieved.

Air mendidih
Boiling water M

Cawan polistirena
Polystyrene cup

Air pada 30°C


Water at 30°C

Rajah 2.1
Diagram 2.1

(a) (i) Apakah maksud keseimbangan terma? OP2


What is the meaning of thermal equilibrium?
[1 markah/mark]

(ii) Terangkan dalam konteks keseimbangan terma, bagaimana termometer yang diletakkan ke dalam
air mendidih memberikan bacaan suhu 100°C. OP3
Explain, in terms of thermal equilibrium, how a thermometer which is placed into the boiling water reads at
the temperature of 100 °C.
[4 markah/marks]

(b) (i) Hitung suhu akhir air di dalam cawan polistirena itu. OP3
Calculate the final temperature of the water in the polystyrene cup.
[Muatan haba tentu M = 800 J kg–1 °C–1 , muatan haba tentu air = 4 200 J kg–1 °C–1]
[Specific heat capacity of M = 800 J kg–1 °C–1, specific heat capacity of water = 4 200 J kg–1 °C–1]
[4 markah/marks]

(ii) Nyatakan satu andaian yang anda buat di 2(b)(i). OP4


State one assumption that you have made in 2(b)(i).
[1 markah/mark]

© Penerbit Mahir Sdn. Bhd. (183897-P) 88

Fizik Tg 4 (Bab 4)6th 9/2/23.indd 88 09/02/2023 4:20 PM


Fizik Tingkatan 4 Bab 4
(c) Rajah 2.2 menunjukkan sebuah kelalang termos digunakan unuk mengekalkan suhu minuman panas
untuk jangka masa yang lama. OP6 KBAT
Diagram 2.2 shows a thermos flask used to maintain the temperature of a hot drink for a long time.

Penutup
Stopper
Ruang P
Space P

Tiub dinding dua lapis


Double walled tube

Minuman panas
Hot drink

Rajah 2.2
Diagram 2.2

Jadual 1 menunjukkan ciri-ciri bagi empat kelalang termos yang berlainan.


Table 1 shows characteristics for four different thermos flasks.

Tiub dinding dua lapis


Kelalang Double walled tube
Penutup Ruang P
termos
Stopper Space P
Thermos flask Dibuat Disalut oleh
Made of Coated by

Plastik berongga Vakum Kaca Cat berkilat


W
Hollow plastic Vacuum Glass Shiny paint

Plastik berongga Vakum Kuprum Cat hitam


X
Hollow plastic Vacuum Copper Black paint

Plastik pepejal Udara Kaca Cat hitam


Y
Solid plastic Air Glass Black paint

Plastik pepejal Udara Kuprum Cat berkilat


Z
Solid plastic Air Copper Shiny paint

Maklumat tambahan/ Additional information:


Muatan haba tentu kaca/ Specific heat capacity of glass = 840 J kg–1 °C–1
Muatan haba tentu kuprum/ Specific heat capacity of copper = 387 J kg–1 °C–1

Jadual 1
Table 1

Terangkan kesesuaian setiap ciri kelalang termos dan tentukan kelalang termos yang paling sesuai untuk
digunakan bagi tujuan mengekalkan suhu minumann panas. Beri sebab bagi pilihan anda.
Explain the suitability of each characteristic of the thermos flasks and determine the most suitable thermos flask
to be used for the purpose of maintaining the temperature of the hot drink. Give reasons for your choice.
[10 markah/marks]

89 © Penerbit Mahir Sdn. Bhd. (183897-P)

Fizik Tg 4 (Bab 4)6th 9/2/23.indd 89 09/02/2023 4:20 PM


Fizik Tingkatan 4 Bab 5

BAB
5
Gelombang
Waves

Tema Gelombang, Cahaya dan Optik


Theme Waves, Light and Optics

NOTA EFEKTIF

1 Gelombang ialah satu proses memindahkan 4 Pelembapan/ Damping


tenaga daripada satu tempat ke tempat lain yang Pengurangan amplitud bagi satu sistem ayunan
dihasilkan oleh sistem bergetar atau berayun. di mana tenaga dilesapkan dalam bentuk haba.
Waves is a process of transferring energy from one location The decrease in the amplitude of an oscillating system
to another which is produced by an oscillating or vibrating. when its energy is drained out as heat.
2 Jenis gelombang/ Types of waves: Sesaran (cm)
(a) Gelombang melintang/ Transverse wave Displacement
Arah getaran zarah-zarah medium
gelombang berserenjang dengan arah
perambatan gelombang. 0 Masa (s)
The vibration of particles in the medium is Time
perpendicular to the direction of the wave.
(b) Gelombang membujur/ Longitudinal wave 5 Resonans/ Resonance
Arah getaran zarah-zarah medium Resonans berlaku apabila satu sistem ayunan
gelombang selari dengan arah perambatan dikenakan daya luar yang mempunyai frekuensi
gelombang. yang sama dengan frekuensi aslinya. Semasa
The vibration of particles in the medium is parallel resonans, sistem berayun dengan frekuensi aslinya
to the direction of propagation of the wave. dan amplitud maksimum.
3 Ciri-ciri gelombang/ Characteristics of waves: When a periodic force is applied to an oscillating system
(a) Amplitud/ Amplitude at its natural frequency. During resonance, the system
Sesaran masimum sesuatu objek dari oscillates with its natural frequency and it oscillates with
kedudukan keseimbangan maximum amplitude.
The maximum displacement of an object from the 6 Muka gelombang/ Wavefront
equilibrium position. Garisan yang menyambungkan titik-titik yang
(b) Tempoh/ Period (T) berada pada fasa yang sama.
Masa yang diambil bagi satu ayunan lengkap. Lines joining all the points of the same phase.
The time taken for one complete oscillation.
(c) Frekuensi/ Frequency (f)
λ λ
Bilangan ayunan lengkap dalam masa satu
saat.
The number of complete oscillation per second.
(d) Panjang gelombang/ Wavelength (λ)
Gelombang satah Gelombang membulat
Jarak antara dua puncak berturutan/ dua Plane wave Circular wave
titik sefasa yang berturutan/ dua kawasan
mampatan berturutan/ dua regangan 7 Pantulan gelombang/ Reflection of wave
berturutan. (a) Gelombang berubah arah perambatan
Distance between two consecutive points in selepas dipantulkan.
phase two successive crests/ two successive Direction of propagation of wave change after
compressions/ two successive rarefactions. reflection.
(e) Laju gelombang/ Speed of wave (v) (b) Frekuensi, panjang gelombang, dan laju
Jarak yang dilalui sesaat oleh profil gelombang. tidak berubah selepas pantulan.
Distance travelled per second by a wave profile. Frequency, wavelength and speed remain
v = fλ unchanged after reflection.

© Penerbit Mahir Sdn. Bhd. (183897-P) 90

Fizik Tg 4 (Bab 5)5th 9/2/23.indd 90 09/02/2023 5:56 PM


Fizik Tingkatan 4 Bab 5

8 Pembiasan gelombang/ Refraction of wave
Perubahan
(a) Perubahan arah perambatan gelombang Ciri gelombang
selepas belauan
apabila melalui dua medium berbeza Characteristics
Change after
disebabkan laju gelombang berubah. of waves
diffraction
A change in direction propagation as the
waves pass through two different mediums due Panjang Tiada perubahan
to the speed of the wave change. gelombang Unchanged
(b) Perubahan arah perambatan Wavelength
Change of direction of propagation Muka gelombang
Arah perambatan
• Kawasan dalam ke kawasan cetek: Direction of tersebar
Dibias mendekati garis normal propagation Wavefront spreads
Deep region to shallow region:
Refracts towards normal line Amplitud Berkurang
• Kawasan cetek ke kawasan dalam: Dibias Amplitude Decreases
menjauhi garis normal (b) Faktor-faktor yang mempengaruhi kesan
Shallow region to deep region: pembelauan.
Refracts away from normal line Factors that influences diffraction of waves.
• Celah kecil: Pembelauan lebih jelas
Air cetek
Kawasan Air dalam Narrow slit: Diffraction is more obvious
Shallow
Region Deep water • Panjang gelombang besar: Pembelauan
water
jelas
Long wavelength: Diffraction more obvious
Laju Laju Perlahan
10 Prinsip superposisi/ The principle of
Speed Faster Slower
superposition
Panjang Apabila dua gelombang bersuperposisi, sesaran
Panjang Pendek paduan ialah hasil tambah secara individu bagi
gelombang
Longer Shorter dua gelombang itu.
Wavelength
When two waves overlap, the resultant displacement is
Tiada Tiada the sum of the individual of the two waves.
Frekuensi 11 Interferens gelombang/ Interference of waves
perubahan perubahan
Frequency Superposisi dua gelombang berpunca daripada
Unchanged Unchanged
dua sumber koheren/ The superposition of two
waves from a coherent source of waves.
9 Pembelauan gelombang/ Diffraction of wave
12 Sumber koheren/ Coherent source
(a) Penyebaran gelombang apabila
Dua sumber gelombang yang mempunyai
gelombang itu merambat melalui suatu
frekuensi yang sama dan beza fasa yang sama.
celah atau tepi halangan.
When the frequency of both waves is the same and the
The spreading of waves when the waves pass
phase difference is the same.
through a gap or round a barrier.
ax
Perubahan λ=
Ciri gelombang D
selepas belauan
Characteristics λ = Panjang gelombang/ Wavelength
Change after
of waves a = Jarak di antara dua sumber koheren
diffraction
Distance between two coherent sources
Frekuensi Tiada perubahan x = Jarak di antara dua garis antinod// nod berturutan
Frequency Unchanged Distance between two consecutive antinodes/ nodes
D = Jarak di antara dua sumber koheren dengan
Laju Tiada perubahan
titik di mana x diukur/ Distance between two
Speed Unchanged
coherent sources and point where x is measured

91 © Penerbit Mahir Sdn. Bhd. (183897-P)

Fizik Tg 4 (Bab 5)5th 9/2/23.indd 91 09/02/2023 5:56 PM


Fizik Tingkatan 4 Bab 5

5.1 Asas Gelombang


Fundamental of Waves

Buku Teks: m.s 172-183

1 Apakah maksud gelombang? TP 1 OP2


What is the meaning of wave?

Satu proses memindahkan tenaga daripada satu tempat ke tempat lain yang dihasilkan oleh sistem
bergetar atau berayun.
A process of transferring energy from one location to another which is produced by an oscillating of vibrating
system.

2 Gelombang memindahkan tenaga tanpa memindahkan jirim.


Wave transfer energy without transferring matter.

Rajah menunjukkan satu reben diikat pada spring slinki. Seorang murid menggerakkan hujung A spring slinki
secara mengufuk dari sisi ke sisi.
Diagram shows a ribbon is tied to the slinky spring. A pupil moved the end A of the spring slinky from side to side
horizontally.

Reben
Ribbon

Gerakan
tangan
Hand
movement B
A
Arah pemindahan tenaga
Direction of energy transfer

Berdasarkan rajah: SB 26
Based on the diagram:

(a) Apakah yang dirasai oleh tangan murid di hujung B yang tetap setelah hujung A digerakkan dari sisi ke
sisi? TP 3 OP4
What is felt by the pupil at end B after the slinky spring is moved from side to side?
Getaran spring slinki/ Vibration of the slinky spring

(b) Apakah arah pemindahan tenaga sepanjang spring slinki? Tandakan pada rajah. TP 3 OP4
What is the direction of energy transfer along the slinky spring? Mark on the diagram.
Dari kiri ke kanan// Dari A ke B

From left to right// From A to B

(c) Huraikan gerakan reben yang diikat pada spring slinki. TP 3 OP4
Describe the movement of the ribbon on the slinky spring.
Reben cuma bergetar ke atas dan ke bawah di satu kedudukan yang tetap.

The ribbon only vibrates up and down at a fixed position.

© Penerbit Mahir Sdn. Bhd. (183897-P) 92 SB 28

Fizik Tg 4 (Bab 5)5th 9/2/23.indd 92 09/02/2023 5:56 PM


Fizik Tingkatan 4 Bab 5
3 Jenis-jenis gelombang
Types of waves

(a) Apakah profil gelombang? TP 1 OP2


What is wave profile?

Rupa bentuk spring slinki semasa gelombang merambat melaluinya.


The shape of the slinky spring as waves propagate through it.

(b) Lengkapkan carta pengelasan gelombang di bawah. TP 1 OP2


Complete the chart of wave classification below.

(i) Gelombang melintang


Transverse wave

Gelombang progresif
Profil gelombang

Progressive wave
Wave profile

(ii) Gelombang membujur


Longitudinal wave

Gelombang pegun
Stationary wave

(c) Perbandingan antara gelombang mekanikal dan gelombang elektromagnet. TP 2 OP3


Comparison between mechanical wave and electromagnetic wave.

Gelombang mekanikal Gelombang elektromagnet


Mechanical wave Electromagnetic wave

(i) Memerlukan medium untuk memindahkan Tidak memerlukan medium untuk memindahkan
tenaga dari satu titik ke titik yang lain tenaga
Requires a medium to transfer energy from one Does not require a medium to transfer energy
point to another

Terdiri daripada getaran zarah-zarah medium (ii) Terdiri daripada ayunan medan elektrik dan
Made up of vibrating particles of a medium medan magnet yang berserenjang antara satu
sama lain
Made up of oscillating electric and magnetic
fields perpendicular to one another

(iii) Gelombang air dan gelombang bunyi Gelombang radio, gelombang cahaya dan sinar
Water waves and sound waves gama
Radio waves, light waves and gamma ray

93 © Penerbit Mahir Sdn. Bhd. (183897-P)

Fizik Tg 4 (Bab 5)5th 9/2/23.indd 93 09/02/2023 5:56 PM


Fizik Tingkatan 4 Bab 5
ARTWORK BY FREELANCER M.ZAIDEE
(d) Rajah menunjukkan gelombang terhasil apabila spring slinki digerakkan ke depan dan ke belakang
secara berulang. TP 3 OP4
Diagram shows the waves produce when a slinky spring was moved forward and backward repeatedly.

C R C R C R

(i) Tandakan (✓) pada jawapan yang betul. SB 27


Tick (✓) the correct answer.

Gelombang yang dihasilkan oleh spring slinki dalam rajah di atas adalah
The waves produced by the slinky spring in the diagram above is

gelombang melintang gelombang membujur


transverse waves longitudinal waves ✓

(ii) Labelkan mampatan spring (C), regangan spring (R) dan panjang gelombang (λ) terhasil dalam
SUPERB SBP FIZIK TINGKATAN 4 BATCH 3
rajah di atas.
Label the compression (C), rarefaction (R) of the spring and the wavelength (λ) of the wave produced in the
diagram above.

(iii) Gariskan jawapan yang betul berdasarkan rajah di atas.


Underline the correct answer based on the diagram above.
SB 29
Arah perambatan gelombang spring (selari / berserenjang) dengan arah getaran zarah.

The direction of propagation of the spring waves is (parallel / perpendicular) to the vibration of the particle.

(iv) Nyatakan satu contoh gelombang bagi jawapan di 3(d)(i).


State one example of the wave for the answer in 3(d)(i).
Gelombang bunyi/ Sound wave
SB 26
(e) Rajah menunjukkan gelombang terhasil apabila spring slinki digerakkan ke kiri dan ke kanan secara
berulang. TP 3 OP4
Diagram shows the waves produce when a slinky spring was moved to the left and right repeatedly.

Arah getaran zarah


Direction of particles vibration

Gerakan
tangan
Hand
movement SB 31
λ Arah perambatan
Direction of propagation

© Penerbit Mahir Sdn. Bhd. (183897-P) 94 SB 28

Fizik Tg 4 (Bab 5)5th 9/2/23.indd 94 09/02/2023 5:56 PM


Fizik Tingkatan 4 Bab 5
(i) Tandakan (✓) pada jawapan yang betul.
Tick (✓) the correct answer.

Gelombang yang dihasilkan oleh spring slinki dalam rajah diberi adalah
The waves produced by the slinky spring in the given diagram is

gelombang melintang gelombang membujur


transverse waves ✓ longitudinal waves

(ii) Labelkan panjang gelombang terhasil dan tunjukkan arah getaran zarah dan arah perambatan
gelombang spring dalam rajah diberi.
Label the wavelength of the wave produced and show the direction of particles vibration and the direction of
propagation of the spring waves in the given diagram.

(iii) Gariskan jawapan yang betul berdasarkan rajah diberi.


Underline the correct answer based on the given diagram.

Arah perambatan gelombang spring (selari / berserenjang) dengan arah getaran zarah.

The direction of propagation of the spring waves is (parallel / perpendicular) to the vibration of the particle.

(iv) Nyatakan satu contoh gelombang bagi jawapan di 3(e)(i).


State one example of the wave for the answer in 3(e)(i).
Gelombang cahaya/ Light wave

4 (a) Lengkapkan jadual ciri-ciri gelombang di bawah. TP 1 OP2


Complete the table of the characteristics of wave below.

Istilah Maksud
Term Meaning

(i) Amplitud (A) Sesaran maksimum sesuatu objek dari kedudukan keseimbangan
Amplitude The maximum displacement of the object from its equilibrium position

(ii) Tempoh (T ) Masa bagi satu ayunan// getaran lengkap


Period The time taken for one complete oscillation// vibration

(iii) Frekuensi ( f ) Bilangan ayunan lengkap dalam masa satu saat


Frequency Number of complete oscillation per second

(iv) Panjang gelombang (λ) Jarak di antara dua titik sefasa yang berturutan
Wavelength The distance between two consecutive points in phase

(v) Laju gelombang (v) Jarak yang dilalui oleh gelombang per unit masa
Wave speed The distance travelled by a wave per unit of time

95 © Penerbit Mahir Sdn. Bhd. (183897-P)

Fizik Tg 4 (Bab 5)5th 9/2/23.indd 95 09/02/2023 5:56 PM


Fizik Tingkatan 4 Bab 5
(b) Rajah di bawah menunjukkan graf sesaran-masa
SB 28 SBair.
dan graf sesaran-jarak bagi suatu gelombang 28TP 4 OP5 SB
The diagram below shows a graph of displacement-time and a graph of displacement-distance for water waves.
d/cm d/cm

0.5 0.5

0 0
0.1 0.2 0.3 0.4 0.5 0.6 t/s 0.5 1.0 1.5 2.0 2.5 3.0 s/cm
– 0.5 – 0.5

Berdasarkan graf-graf tersebut, tentukan:


SB 30
Based on the graphs, determine: SB 30

(i) Amplitud (ii) Tempoh (iii) Frekuensi (iv) Panjang gelombang


Amplitude Period Frequency Wavelength SB
= 0.5 cm = 0.4 s = 2.5 Hz = 2.0 cm

λ
(c) Diberi persamaan v = di mana v = laju, λ = panjang gelombang dan T = tempoh. Frekuensi
T
1
gelombang adalah f = .
T
λ 1
The equation given v = where v = speed, λ = wavelength and T = period. The wave frequency is f = .
T T
Maka, laju gelombang adalah
Then, the wave speed is v=fλ

SB
SB 32 SB 32
5.2 Pelembapan dan Resonans
Damping and Resonance

Buku Teks: m.s 184-183


1 Apakah maksud pelembapan? TP 1 OP2
What is the meaning of damping?

Pengurangan amplitud suatu sistem ayunan akibat kehilangan tenaga.


The reduction in amplitude in an oscillating system due to loss of energy.

2 Huraikan dua jenis pelembapan berikut: TP 1 OP2


Describe two types of the following damping:

Jenis Penjelasan
Type Explanation

(a) Pelembapan luaran Sistem ayunan kehilangan tenaga bagi mengatasi daya geseran atau
External damping SB 34
rintangan udara SB 34 SB 35
Oscillating system loses energy to overcome friction or air resistance

(b) Pelembapan dalaman Sistem ayunan kehilangan tenaga kerana renggangan dan mampatan
Internal damping zarah-zarah yang bergetar dalam sistem tersebut
Oscillating system loses energy because of the stretching and compression of
the vibrating particles in the system

© Penerbit Mahir Sdn. Bhd. (183897-P) 96

Fizik Tg 4 (Bab 5)5th 9/2/23.indd 96 09/02/2023 5:56 PM


Fizik Tingkatan 4 Bab 5
SB 27
3 Lakar graf sesaran melawan masa untuk menunjukkan proses pelembapan. TP 2 OP3
Sketch a graph of displacement against time to show damping process.
Sesaran
Displacement

INFO
Masa Semasa pelembapan berlaku,
Time frekuensi ayunan adalah kekal
manakala amplitud ayunan
berkurang.
During damping, the oscillating frequency
remains constant while the oscillating
amplitude decreases.

4 Seorang ibu meletakkan bayinya untuk tidur di dalam buaian. Dia perlu menggerakkan buaian itu ke atas dan
ke bawah secara berterusan. TP 4 OP5
SB 29
A mother puts her baby to sleep in a cradle. She needs to continuously move the cradle upward and downward.

Gerakan
buaian
Cradle
movement

(a) Apakah yang berlaku kepada buaian itu apabila ibu memberhentikan gerakan itu?
What happen to the cradle when the mother stops moving the cradle?
SB 31
Amplitud ayunan buaian semakin berkurang dan berhenti.

Amplitude of oscillation of the cradle decreases and stops.

(b) Apakah nama fenomena yang berlaku di 4(a).


What is the name of phenomenon happened in 4(a).
Pelembapan/ Damping

(c) Apakah yang perlu dilakukan untuk meneruskan gerakan buaian? Jelaskan.
What must be done to maintain the movement of the cradle? Explain.
Mengenakan daya luar pada buaian. Daya luar yang dikenakan itu membekalkan tenaga tambahan ke

atas ayunan buaian untuk menggantikan tenaga yang hilang disebabkan daya geseran.

Apply external force to the cradle. The external force gives extra energy to the oscillating cradle as to compensate

SB 33
for the energy dissipated by frictional force.

97 © Penerbit Mahir Sdn. Bhd. (183897-P)

Fizik Tg 4 (Bab 5)5th 9/2/23.indd 97 09/02/2023 5:56 PM


Fizik Tingkatan 4 Bab 5
5 Sistem ayunan yang dikenakan daya luar berkala dikatakan sedang melakukan ayunan paksa .
forced oscillation.
The oscillating system that applied by the periodic external force is said to be in a .

6
SB 27
Sistem ayunan yang disesar dan kemudian dibiarkan berayun tanpa tindakan daya luar akan berayun dengan

satu frekuensi yang tertentu yang dinamakan frekuensi asli .


An oscillating system that is displaced and then left to oscillate without the action of external forces, will oscillate at its
natural frequency
.

7 Resonans berlaku apabila suatu sistem dikenakan daya luar yang mempunyai frekuensi yang
sama dengan frekuensi asli sistem ayunan tersebut.
Resonance
occurs when a force is applied to an oscillating system at its natural frequency.

8 Semasa resonans, sistem berayun dengan frekuensi aslinya dan amplitud maksimum .
natural frequency maximum amplitude
During resonance, the system oscillates with its and .

9
SB 29
Rajah menunjukkan susunan radas untuk mengkaji penghasilan resonans. TP 4 OP5
The diagram shows the apparatus arrangement to study the production of resonance.
Tali
Rope

Bandul Kaki retort


Pendulum A Restort stand
B
D E C

Pengapit G
G-clamp
Meja
Table

(a) Dua bandul yang manakah mempunyai frekuensi asli yang sama?
Which two pendulums will have the same natural frequency? SB 31
C dan D/ C and D

(b) Apabila bandul C ditolak, apakah yang berlaku kepada bandul-bandul yang lain? Mengapa?
When pendulum C is pushed, what happen to the other pendulums? Why?
Bandul-bandul yang lain turut berayun kerana tenaga dipindahkan melalui tali.

The other pendulums start to oscillate because energy is transferred through the rope.

(c) Bandul yang manakah akan berayun dengan amplitud yang paling besar?
Which pendulum oscillates with the largest amplitude?
D

SB 33

© Penerbit Mahir Sdn. Bhd. (183897-P) 98

Fizik Tg 4 (Bab 5)5th 9/2/23.indd 98 09/02/2023 5:56 PM


Fizik Tingkatan 4 Bab 5
(d) Dua bandul manakah yang akan terus berayun walaupun bandul-bandul yang lain sudah berhenti? Mengapa?
Which two pendulums will continue to oscillate even when the other pendulums have stopped? Why?
Bandul C dan D. Bandul C menyebabkan bandul D berayun dengan frekuensi aslinya.

Bandul D menerima tenaga yang lebih besar dan berayun dengan amplitud yang maksimun. Resonans

berlaku/ Pendulums C and D. Pendulum C causes pendulum D to oscillate at its natural frequency. Pendulum D
receives a bigger amount of energy and oscillate with the maximum amplitude. Resonance occurs.

5.3 Pantulan Gelombang


Reflection of Waves

Buku Teks: m.s 184-183

INFO
Pantulan gelombang akan berlaku apabila suatu gelombang tuju mengena dengan permukaan yang keras dan
dibalikkan. Permukaan itu memainkan peranan sebagai pemantul.
Reflection of waves occurs when an incidence wave strikes a hard surface and reflected. The surface acts as the reflector.

1 Apakah maksud muka gelombang? TP 1 OP2


What is the meaning of wavefront?

Garisan yang menyambungkan titik-titik pada fasa yang sama.


Lines joining all points of the same phase.

2 Rajah di bawah menunjukkan gelombang air menuju ke arah sebuah pemantul satah. TP 3 OP4
The diagram below shows water waves move towards a plane reflector.

λ1

Pemantul satah
Plane reflector

Gelombang pantulan
Reflection waves

(a) Lukis pantulan gelombang yang terhasil dalam rajah di atas.


Draw the reflection of waves produced in the diagram above.
SB 32
(b) Bandingkan sudut tuju, i dengan sudut pantulan, r.
Compare the angle of incidence, i with the angle of reflection, r.
i=r

99 © Penerbit Mahir Sdn. Bhd. (183897-P)

Fizik Tg 4 (Bab 5)5th 9/2/23.indd 99 09/02/2023 5:56 PM


Fizik Tingkatan 4 Bab 5
(c) Bandingkan antara gelombang tuju dan gelombang dipantulkan dari segi:
Compare between incident waves and reflected waves in terms of:

Aspek Berubah/ Kekal


Aspect Change/ Unchanged

(i) Laju gelombang Kekal


Wave speed Unchanged

(ii) Panjang gelombang Kekal


Wavelength Unchanged

(iii) Frekuensi Kekal


Frequency Unchanged

(iv) Arah perambatan Berubah


Direction propagation Change

3 Rajah menunjukkan sistem sonar digunakan untuk menentukan kedalaman dasar laut. TP 3 OP4
The diagram shows a sonar system used to determine the depth of the seabed.

Penerima Penghantar
Receiver Transmitter

Dasar laut
Seabed

INFO
Sonar adalah gelombang ultrasonik yang dipantulkan untuk navigasi,
komunikasi dan mengesan objek di bawah permukaan air.
Sonar is a reflected ultrasonic waves that used to navigate, communicate with or detect
objects under the surface of the water.

(a) Nyatakan fenomena gelombang bunyi bagi mengukur kedalaman dasar laut.
State the sound wave phenomenon to measure the depth of the seabed.
Pantulan/ Reflection

(b) Terangkan mengapa sonar menggunakan bunyi berfrekuensi tinggi.


Explain why sonar used a high frequency.
Tenaga tinggi dan boleh bergerak jauh/ High energy and can move further

© Penerbit Mahir Sdn. Bhd. (183897-P) 100

Fizik Tg 4 (Bab 5)5th 9/2/23.indd 100 09/02/2023 5:56 PM


Fizik Tingkatan 4 Bab 5
(c) Jelaskan bagaimana kedalaman laut diukur.
Explain how to measure the depth of the sea.
Pemancar mengeluarkan gelombang ultrasonik ke dasar laut. Gelombang itu dipantulkan oleh dasar laut

dan dikesan oleh penerima. Sela masa, t bagi gelombang bergerak pergi dan balik dasar laut diukur.

Kedalaman laut, s diukur dengan s = vt/2 di mana v ialah halaju gelombang ultrasonik dalam air.

A transmitter emits ultrasonic wave to the seabed. The ultrasonic wave is reflected by the transmitter and

detected by a receiver. Time interval, t, for the ultrasonic wave to propagate back and forth from the seabed is

measured. The depth of the seabed, s is measured by s = vt/2 where v is the speed of ultrasonic wave in water.

4 Menyelesaikan masalah melibatkan pantulan gelombang. TP 3 OP4


Solve problem involving reflection of wave.

(a) Ahmad menjerit di hadapan dinding tinggi. Dia mendengar bunyi gema suaranya 1.2 saat kemudian.
Laju bunyi di udara ialah 340 m s–1. Berapakah jarak di antara Ahmad dan dinding?
Ahmad shouts in front of a high wall. He hears the sound of his echoes 1.2 s later. The speed of sound in air is 340 m s–1.
What is the distance between Ahmad and the wall?
vt
s=
2
340 × 1.2
=
2
= 204 m

(b) Sebuah radar memancarkan isyarat ke arah sebuah kapal terbang. Halaju isyarat itu ialah 3.0 × 108 m s–1.
Selepas 4.0 × 10–3 s, radar mengesan isyarat yang dipantulkan. Berapakah jarak kapal terbang itu
daripada radar?
A radar transmit a signal to an aeroplane. The velocity of the signal is 3.0 × 108 m s-1. After 4.0 × 10–3 s, the radar
detects the reflected signal. What is the distance of the plane from the radar?
vt
s=
2
3 × 108 × 4 × 10–3
=
2
= 600 000 m
= 600 km

5.4 Pembiasan Gelombang


Refraction of Waves

Buku Teks: m.s 194-201


1 Apakah maksud pembiasan gelombang? TP 1 OP2
What is the meaning of refraction of wave?

Perubahan arah perambatan gelombang yang disebabkan oleh perubahan halaju gelombang apabila
gelombang itu merambat dari satu medium ke medium yang lain.
The change in direction of propagation waves caused by the change in the velocity of waves when the waves
propagate from one medium to another.

101 © Penerbit Mahir Sdn. Bhd. (183897-P)

Fizik Tg 4 (Bab 5)5th 9/2/23.indd 101 09/02/2023 5:56 PM


Fizik Tingkatan 4 Bab 5
2 Rajah menunjukkan sekeping papak kaca berbentuk segi empat direndamkan dalam tangki riak untuk
SB 31
mewujudkan perbezaan kedalaman air. TP 2 OP3
Diagram shows a square glass slab is immersed in the ripple tank so as to create different depths of water.

v1 > v2
v2 v1
λ1 λ2 λ1

Kawasan cetek
Shallow region
Kawasan
Kawasan dalam Papak kaca dalam
Deep region Glass slab Deep region

(a) Bandingkan frekuensi gelombang air di kawasan dalam dengan di kawasan cetek. Jelaskan.
Compare the frequency of water wave in deep region to the shallow region. Explain.
SB 33
Sama kerana menggunakan motor penggetar satah yang sama.

Same because using the same plane vibration motor.

(b) Bandingkan laju gelombang air di kawasan dalam dengan di kawasan cetek.
Compare the speed of water wave in deep region and in shallow region.
Laju gelombang air di kawasan dalam lebih tinggi daripada di kawasan cetek.

The speed of water wave in deep region is higher than in shallow region.

(c) Bandingkan panjang gelombang di kawasan dalam dengan di kawasan cetek.


Compare the wavelength in deep region and in shallow region.
Panjang gelombang di kawasan dalam lebih panjang daripada di kawasan cetek.

The wavelength in deep region is longer than in shallow region.

(d) Hubung kait laju gelombang air dengan panjang gelombang air.
Relate the speed of water wave and the wavelength of water wave.
Berkadar terus/ Directly proportional

3 Kesan pembiasan ke atas ciri-ciri gelombang. TP 4 OP5


The effects of refraction on the characteristics of waves.

Dari SB 35 air dalam ke


kawasan Dari kawasan air cetek ke
Ciri-ciri kawasan air cetek kawasan air dalam
Characteristics From deep water region to shallow From shallow water region to deep
water region water region

Sudut tuju dan sudut biasan


(a) i > r (b) i < r
Incident angle and refraction angle

Laju gelombang (c) Berkurang (d) Bertambah


Speed of wave Decreases Increases

© Penerbit Mahir Sdn. Bhd. (183897-P) 102

Fizik Tg 4 (Bab 5)5th 9/2/23.indd 102 09/02/2023 5:56 PM


Fizik Tingkatan 4 Bab 5

Panjang gelombang (e) Berkurang (f) Bertambah


Wavelength Decreases Increases

Frekuensi (g) Kekal (h) Kekal


Frequency Unchanged Unchanged

(i) Dibias mendekati garis


Arah perambatan (j) Dibias menjauhi garis normal
normal
Direction propagation Refract away from normal line
Refract towards normal line

4 Rajah di bawah menunjukkan susunan radas untuk mengkaji pembiasan gelombang. TP 4 OP5
The diagram below shows the arrangement to apparatus study the refraction of waves.

Lengkapkan rajah berikut dengan melukis pembiasan gelombang yang terhasil.


Complete the following diagrams by drawing the refraction of waves.
(a) Dalam Cetek Dalam
Deep Shallow Deep

(b) Dalam Cetek Dalam


Deep Shallow Deep

103 © Penerbit Mahir Sdn. Bhd. (183897-P)

Fizik Tg 4 (Bab 5)5th 9/2/23.indd 103 09/02/2023 5:56 PM


Fizik Tingkatan 4 Bab 5
5 Fenomena Pembiasan Gelombang dalam Kehidupan Harian TP 5 OP6
Phenomena of Refraction of Waves in Daily Life
SB 33SB 33
Berdasarkan rajah berikut, terangkan mengapa bunyi tidak boleh didengar pada jarak yang jauh pada waktu
siang SB
yang panas
32SB 32 tetapi boleh didengar dengan jelas pada waktu malam yang sejuk.
Based on the following diagrams, explain why sound cannot be heard over a longer distance during hot day but can be
heard clearly over a longer distance at cold night.

Udara yang sejuk (Lebih tumpat)


Cool air (Denser)
Udara suam (Kurang tumpat)
Warm air (Less dense)
Pada waktu malam
At cold night
Pada waktu siang
During hot day
Udara suam (Kurang tumpat) Udara sejuk (tumpat) Padang
Warm air (Less dense) Padang
Cool air (Denser) Field
Field

SB 34SB 34
(a) Pada waktu siang: SB 35SB 35
During hot day:
Lapisan udara berhampiran permukaan Bumi lebih panas. Bunyi bergerak daripada udara panas ke

udara sejuk. Laju bunyi berkurang. Bunyi dibias mendekati garis normal. Bunyi semakin menjauhi

Bumi dan tidak dapat didengar dengan jelas.

The air layer near the surface to be warmer. Sound travels from hot to cold air. The speed of sound decreases.

Sound is refracted towards normal line. Sound refracted away from the Earth and cannot be heard clearly.

(b) Pada waktu malam:


At night:
Lapisan udara berhampiran Bumi lebih sejuk. Bunyi bergerak daripada udara sejuk ke udara panas.

Laju bunyi bertambah. Bunyi dibias menjauhi garis normal. Bunyi semakin menghampiri Bumi dan

dapat didengar dengan jelas.

The air layer near the ground colder. Sound travels from cold to hot air. The speed of sound increases. Sound

refracted away from the normal line. Sound waves moving towards the Earth and can be heard clearly.

6 Menyelesaikan masalah melibatkan pembiasan gelombang. TP 4 OP5


Solve problem involving refraction of waves.

(a) Tulis persamaan yang melibatkan pembiasan gelombang.


Write an equation for refraction of waves.
v1 v2
=
λ1 λ2

© Penerbit Mahir Sdn. Bhd. (183897-P) 104

Fizik Tg 4 (Bab 5)5th 9/2/23.indd 104 09/02/2023 5:56 PM


Fizik Tingkatan 4 Bab 5
(b) Rajah menunjukkan pembiasan gelombang air.
The diagram shows a refraction of a water wave.
3.0 cm

1.5 cm

Air dalam
Deep water
Air cetek
Shallow water

Jika laju gelombang air ialah 5 cm s–1 di dalam kawasan air dalam, hitung laju gelombang air di dalam
kawasan air cetek.
If the speed of the water waves is 5 cm s–1 in the deep water region, calculate the speed of the water wave in the
shallow water region.
v1 = 5 cm s–1; λ1 = 3.0 cm; λ2 = 1.5 cm
v1 v2
=
λ1 λ2
5 v2
=
3.0 1.5
v2 = 2.5 cm s–1

5.5 Pembelauan Gelombang


Diffraction of Waves

Buku Teks: m.s 202-209


1 Apakah maksud pembelauan gelombang? TP 1 OP2
What is the meaning of diffraction of wave?

Fenomena gelombang tersebar atau bengkok apabila merambat melalui suatu celah.
A phenomenon in which waves spread out or bend as they pass through a gap.

2 Rajah di bawah menunjukkan susunan radas untuk mengkaji pembelauan gelombang. TP 3 OP4
The diagram below shows the arrangement of apparatus to study the diffraction of waves.

Lampu
Lamp

Tangki riak
Ripple tank

Skrin
Screen

105 © Penerbit Mahir Sdn. Bhd. (183897-P)

Fizik Tg 4 (Bab 5)5th 9/2/23.indd 105 09/02/2023 5:56 PM


Fizik Tingkatan 4 Bab 5
Lukis gelombang yang dibelaukan selepas melalui celah atau halangan.
Draw the diffraction waves after passing through a gap or obstacle.
(a) (b)

3 Nyatakan perubahan yang berlaku kepada gelombang tuju selepas melalui celah. TP 4 OP5
State the change occurs to the incident wave after passing through a gap.

(a) Frekuensi Sama


Frequency Equal

(b) Panjang gelombang Sama


Wavelength Equal

(c) Laju gelombang Sama


Speed of wave Equal

(d) Arah perambatan Disebarkan


Direction propagation Spread out

(e) Amplitud Berkurang


Amplitude Decrease

4 Huraikan corak gelombang yang berlainan panjang gelombang selepas melalui celah yang sama saiz. TP 4 OP5
Describe the pattern of the waves with different wavelength after passing through the gap of same size.

Panjang gelombang Pendek Panjang


Wavelength Short Long

3.0 cm 6.0 cm

Corak gelombang
Pattern of waves a a

Pembelauan Kurang ketara Lebih ketara


Diffraction Less significant More significant

5 Nyatakan dua faktor yang menyebabkan kesan pembelauan lebih ketara. TP 3 OP4
State the two factors that cause the diffraction more significant.
Saiz celah yang kecil/ Small size of gap

Panjang gelombang yang panjang/ Longer wavelength

© Penerbit Mahir Sdn. Bhd. (183897-P) 106

Fizik Tg 4 (Bab 5)5th 9/2/23.indd 106 09/02/2023 5:56 PM


Fizik Tingkatan 4 Bab 5
6 Pembelauan Gelombang dalam Kehidupan Harian TP 5 OP6
Diffraction of Waves in Daily Life

(a) Gelombang air


Water waves

Sepasang benteng dibina untuk melindungi kawasan perumahan di tepi pantai.


A pair of barriers is built to protect housing area near a beach.

(i) Bandingkan amplitud gelombang laut sebelum dan selepas melalui benteng itu.
Compare the amplitude of the sea wave before and after passing through the barriers.
Amplitud gelombang selepas melalui bukaan benteng lebih kecil daripada amplitud sebelumnya.

The amplitude after passing through the opening barrier smaller than the amplitude before

(ii) Dengan menghubungkan saiz ombak dengan tenaga gelombang, terangkan bagaimana kesan
hakisan pantai di sekitar kawasan perumahan dapat dikurangkan dengan kehadiran benteng ini.
By relating the size of the wave with the energy, explain how the erosion effect of the beach around the housing
area can be reduced by the barriers.
Membina benteng dengan celah kecil supaya pembelauan berlaku. Tenaga gelombang disebarkan

ke kawasan yang luas selepas melalui celah dan akan berkurang. Hal ini akan mengurangkan

amplitud gelombang.

Build barriers with small gap so that diffraction can occur. Energy of the waves is spread out to larger

area after passing the gap and will be reduced. Thus, the amplitude of the waves will be decreased.

(b) Gelombang bunyi


Sound waves
SUPERB SBP FIZIK TINGKATAN 4 BATCH 3 AR
Rajah menunjukkan Ahmad sedang berdiri di hadapan dinding dan sebuah radio diletakkan di sebalik
dinding. Radio itu dihidupkan.
Diagram shows Ahmad stand in front of a wall and a radio is placed behind the wall. The radio is turned on.

Dinding
Wall

Ahmad Radio

SB107
36 © Penerbit Mahir Sdn. Bhd. (183897-P)

Fizik Tg 4 (Bab 5)5th 9/2/23.indd 107 09/02/2023 5:56 PM


Fizik Tingkatan 4 Bab 5

(i) Adakah Ahmad dapat melihat radio itu? Berikan inferens anda.
Is Ahmad able to see the radio? Give your inference.
Tidak dapat melihat radio kerana gelombang cahaya tidak dibelaukan.

Cannot see the radio because light waves cannot be diffracted.

(ii) Bolehkah Ahmad mendengar bunyi radio itu? Berikan inferens anda.
Is Ahmad hear the sound of the radio? Give your inference.
Boleh mendengar radio kerana bunyi boleh dibelaukan.

Can hear the sound because sound waves can be diffracted.

(iii) Hubung kait antara panjang gelombang denganSUPERB SBP FIZIK


kesan pembelauan untukTINGKATAN 4 BATCH 3
menerangkan mengapa
siaran radio dari sebalik dinding boleh didengar tetapi tidak boleh melihatnya.
Relate between wavelengths with effect of diffraction to explain why we can hear the sound of a radio placed
nearby a corner of a wall but cannot see the radio.
Bunyi mudah dibelaukan kerana panjang gelombang bunyi lebih panjang daripada panjang

gelombang cahaya.

Sound waves are more easily diffracted because its wavelength is longer than the wavelength of light wave.

(c) Gelombang cahaya


Light waves

Rajah menunjukkan corak yang terbentuk apabila cahaya mentol diperhatikan melalui celah di antara
duaARTWORK
jari. BY FREELANCER M.ZAIDEE SB 36
The diagram shows the pattern formed when the light bulb is seen through a gap between two fingers.

Pemerhati
Cahaya
Observer
Light

SB 37 SB 38

(i) Namakan fenomena cahaya yang terlibat.


Name the phenomenon involved.
Pembelauan/ Diffraction

(ii) Lukis corak yang terbentuk apabila saiz celah di antara dua jari lebih sempit pada ruang disediakan.
Draw the pattern formed when the size of the gap between two fingers narrower in the provided space.

© Penerbit Mahir Sdn. Bhd. (183897-P) 108

Fizik Tg 4 (Bab 5)5th 9/2/23.indd 108 09/02/2023 5:56 PM


Fizik Tingkatan 4 Bab 5
(iii) Jelaskan mengapa fenomena di 6(c)(i) lebih sukar dilihat dengan gelombang cahaya berbanding
dengan gelombang bunyi.
Explain why the phenomena in 6(c)(i) is difficult to see for light wave compared to sound wave.
Panjang gelombang cahaya lebih pendek daripada gelombang bunyi. Cahaya sukar untuk

dibelaukan./ Wavelength of light waves is smaller than sound waves. Light is difficult to diffract.

5.6 Interferens Gelombang


Interference of Waves

Buku Teks: m.s 202-209

INFO

Rajah menunjukkan gelombang air apabila dua biji batu dibaling ke dalam air.
Dua gelombang membulat yang dihasilkan bertindih atau bersuperposisi.
Diagram shows a water waves when two stones are thrown in the water. Two circular waves that are
formed on the surface of water are in superposition.

1 Apakah prinsip superposisi? TP 1 OP2


What is the principle of superposition?

Apabila dua gelombang merambat serentak dan bertindih pada satu titik, hasil tambah sesaran pada titik
itu adalah sama dengan hasil tambah sesaran gelombang itu secara individu.
When two waves interfered, the resulting displacement of the medium at any location is the algebraic sum of the
displacements of the individual waves.

2 Apakah maksud interferens gelombang? TP 1 OP2


What is the meaning of interference of wave?

Superposisi dua gelombang yang berpunca daripada dua sumber koheren.


The superposition of two waves originating from two coherent sources.

3 Nyatakan jenis dan maksud interferens yang betul bagi setiap situasi berikut. TP 2 OP3
State the type and definition of interference correctly for each of the following situation.

(a)
Interferens membina : Superposisi antara dua puncak
Constructive crests
interference: Superposition between two

Amplitud terhasil
Resultant amplitude

2a
a a
x
O
109 © Penerbit Mahir Sdn. Bhd. (183897-P)

x
Fizik Tg 4 (Bab 5)5th 9/2/23.indd 109 a O a 10/02/2023 10:12 AM
a2 SUPERB SBP FIZIK TINGKATAN 4 BATCH 3 2a ARTWORK BY FR
aa aa
x x
Fizik Tingkatan 4 Bab 5 O O
(b)
membina 2a
lembangan
Interferens: a Superposisi
a antara dua
x
Constructive troughs
interference: O
Superposition between two
xx
aa OO aa
a2 2a
Amplitud terhasil
Resultant amplitude
x 2a
a a
a Ox a
2a
S
O

a
SB 36 a
x x x x
(c)
O memusnah
a OO a puncak
O
Interferens x : Superposisi antara dan
lembangan a O a
x
2a
x
Destructive O a crest
O trough
interference: Superposition between and

Amplitud terhasil
SBamplitude
Resultant 39
SB 38a x x
O a O

4 Rajah di bawah menunjukkan corak interferens gelombang air yang dihasilkan oleh dua sumber yang koheren. SB
The diagram below shows interference pattern of water waves produced by two coherent sources.

A Garis antinod
Antinodal lines
C Garis nod
Nodal lines
B

S1 S2
SB2 41OP3
(a) Lengkapkan jadual di bawah. TP SB 42
Complete the table below.

Keadaan Jenis interferens


State Type of interference

Titik A: Dua puncak bertemu Interferens membina


Point A: Two crests meet Constructive interference

Titik B: Dua lembangan bertemu Interferens membina


Point B: Two troughs meet Constructive interference
SB 44
Titik C: Puncak bertemuSB 43
lembangan Interferens memusnah
Point C: Crests meet troughs Destructive interference

© Penerbit Mahir Sdn. Bhd. (183897-P) 110

Fizik Tg 4 (Bab 5)5th 9/2/23.indd 110 09/02/2023 5:57 PM


Fizik Tingkatan 4 Bab 5
(b) Pada rajah diberi, TP 2 OP3
On the diagram given,
(i) tandakan titik-titik antinod dan nod.
mark points of antinodes and nodes.
(ii) lukis dan label garis nod dan garis antinod.
draw and label nodal lines and antinodal lines.
ax
(c) Panjang gelombang, λ bagi satu gelombang boleh dihitung dengan menggunakan rumus: λ =
ax D
The wavelength, λ of the wave can be calculated using formula: λ =
D

SUPERB SBP FIZIK TINGKATAN 4 BATCH 3


Jarak di antara dua sumber koheren
ARTWORK BY FREELANCER M.ZAIDEE
a
Distance between two coherent sources

Jarak di antara dua garis antinod/ nod berturutan


x
Distance between two consecutive antinodal lines/ nodal lines

Jarak di antara dua sumber koheren dengan tempat di mana x diukur


D
Distance between two coherent sources and point x is measured
SB 47 SB 48 SB 49
5 Rajah di bawah menunjukkan corak interferens suatu gelombang cahaya. TP 3 OP4
The diagram below shows interference pattern for light wave.
Skrin
Dwicelah Screen
Double-slit

Corak pinggir
Sumber cahaya
interferens
monokromatik
SB 50 Interference
Monochromatic
fringe pattern
light source
SB 51 SB 52
(a) Apakah maksud cahaya monokromatik?
What is meant by monochromatic light?
Cahaya dengan satu warna atau panjang gelombang./ Light with one colour or wavelength.

(b) Huraikan corak interferens yang diperhatikan apabila cahaya melalui dwicelah.
Describe the interference pattern observed when light passes through the double-slit.
Pinggir cerah dan gelap berselang seli dan seragam./ Bright and dark fringes alternately and uniformly.

(c) Hitung panjang gelombang, λ cahaya merah berdasarkan maklumat di bawah.


Calculate the wavelength, λ of red light based on the information below.
a = 0.5 mm, D = 2.0 m, x = 1.8 mm
ax SB 53 SB 54
λ=
D
(0.5 × 10–3) × (1.8 × 10–3)
=
2
= 0.45 × 10–6 m

111 © Penerbit Mahir Sdn. Bhd. (183897-P)

SB 55 SB 56

Fizik Tg 4 (Bab 5)5th 9/2/23.indd 111 09/02/2023 5:57 PM


Fizik Tingkatan 4 Bab 5
(d) Apakah yang terjadi kepada pinggir-pinggir yang terbentuk pada skrin itu apabila
What will happen to the fringes on the screen when

(i) jarak di antara dwicelah dan skrin dikecilkan.


the distance between double-slit and screen is reduced.
Berkurang/ Decreases

(ii) cahaya merah digantikan dengan cahaya biru.


the red light replaced by blue light.
Berkurang/ Decreases

(iii) jarak pemisahan celah itu diperbesarkan.


the distance of separation between slit is enlarged.
Berkurang/ Decreases

6 Rajah di bawah menunjukkan satu eksperimen dilakukan di sebuah kawasan lapang untuk mengkaji interferens
gelombang bunyi. TP 4 OP5
The diagram below shows an experiment was done in an open area to study the interference of sound wave.

X
X: Bunyi kuat
Y Loud sound
Penjana audio
Audio generator X Y: Bunyi lemah
Soft sound
Y
Pembesar suara
Loudspeakers

(a) Dua pembesar suara merupakan sumber koheren gelombang bunyi. Mengapa?
Two loudspeakers are coherent sources of sound wave. Why?
Kedua-dua pembesar suara disambung ke penjana audio yang sama.

Both loudspeakers are connected to the same audio generator.

(a) Berdasarkan rajah di atas, apakah yang anda boleh perhatikan apabila dua gelombang bunyi bertindih?
Based on the diagram obove, what can be observed when two sound waves overlap?
Menghasilkan bunyi kuat dan bunyi lemah secara berselang-seli/ Produce loud sound and soft sound
alternately.

(c) Terangkan pemerhatian anda.


Explain your observation.
Bunyi kuat terhasil di kawasan interferens membina manakalan bunyi lemah terhasil di kawasan

interferens memusnah.

Loud sound produced at constructive interference region while soft sound produced at destructive interference

region.

(d) Mengapakah eksperimen ini mesti dibuat di kawasan lapang?


Why the experiment must be done in an open area?
Mengelakkan pantulan bunyi/ To avoid the reflection of sound

© Penerbit Mahir Sdn. Bhd. (183897-P) 112

Fizik Tg 4 (Bab 5)5th 9/2/23.indd 112 09/02/2023 5:57 PM


Fizik Tingkatan 4 Bab 5
7 Menyelesaikan masalah melibatkan gelombang interferens. TP 3 OP4
Solve problem involving interference wave.

(a) Dalam satu eksperimen interferens gelombang bunyi, dua pembesar


suara diletakkan pada jarak 2.0 m antara satu sama lain. Jarak di
ax
antara dua bunyi kuat berturutan ialah 1.0 m dan pendengar berada λ=
pada jarak 3.0 m dari pembesar suara. D
Berapakah panjang gelombang bunyi yang digunakan? (2.0) × (1.0)
=
In an experiment on the interference of sound waves, two loudspeakers 3.0
are placed at a distance of 2.0 m from each other. = 0.67 m
The distance between two consecutive loud sounds is 1.0 m and the listener
is at a distance of 3.0 m from the loudspeakers.
What is the wavelength of the sound wave used?

(b) Dalam satu eksperimen interferens gelombang air menggunakan


tangki air yang besar, jarak di antara dua penggetar ialah 5.0 cm dan
jarak dari penggetar ke kedudukan di mana pemerhatian garis ax
λ=
antinod dibuat ialah 50.0 cm. Jika jarak di antara garis antinod yang D
bersebelahan ialah 16.0 cm, hitungkan panjang gelombang bagi (5.0) × (16.0)
gelombang yang dihasilkan oleh penggetar. =
50.0
In an experiment of interference of water waves by using a big water tank,
= 1.6 cm
the distance between two vibrators is 5.0 cm and the distance from the
vibrator to the position where antinodal line is observed is 50.0 cm. If the
distance between two consecutive antinodal lines is 16.0 cm, calculate the
wavelength of the waves produced by the vibrator.

(c) Dalam eksperimen dwicelah Young, jarak di antara dwicelah


dan skrin ialah 4.0 m dan jarak pemisahan di antara dua celah ialah ax
λ=
0.5 mm. Hitung di jarak antara dua pinggir cerah berturutan untuk D
cahaya merah yang mempunyai panjang gelombang 7.0 × 10–7 m. λD
x= a
In a Young’s double slit experiment, the distance between the double slit
and the screen is 4.0 m and the separation the two slits is 0.5 mm. (7.0 × 10–7) × (4.0)
=
Calculate the distance between two consecutive bright fringes for red light 0.5 × 10–3
with a wavelength of 7.0 × 10–7 m. = 0.0056 m
= 5.6 m

5.7 Gelombang Elektromagnet


Electromagnetic Waves

Buku Teks: m.s 220-223


1 Rajah di bawah menunjukkan kumpulan gelombang dalam spektrum elektromagnet berdasarkan frekuensi dan
panjang gelombangnya. TP 2 OP3
The diagram below shows a group of waves in the electromagnetic spectrum based on their frequencies and wavelengths.
Sinar ultraungu Cahaya Sinar Gelombang Gelombang
Sinar gama Sinar X Ultraviolet nampak inframerah mikro radio
Gamma rays X-rays rays Visible light Infrared rays Microwaves Radio waves

Panjang gelombang semakin bertambah


Wavelength increasing

113 © Penerbit Mahir Sdn. Bhd. (183897-P)

Fizik Tg 4 (Bab 5)5th 9/2/23.indd 113 09/02/2023 5:57 PM


Fizik Tingkatan 4 Bab 5
(a) Gelombang yang manakah mempunyai frekuensi yang paling rendah?
Which wave has the lowest frequency?
Gelombang radio/ Radio waves

(b) Gelombang yang manakah mempunyai tenaga yang paling tinggi?


Which wave has the highest energy?
Sinar gama/ Gamma rays

(c) Susun gelombang-gelombang dalam rajah diberi mengikut susunan frekuensi menurun.
Arrange the waves in the diagram given in descending order of frequency.
Sinar gama, sinar-X, sinar ultraungu, cahaya nampak, sinar inframerah, gelombang mikro dan

gelombang radio

Gamma ray, X-ray, ultraviolet ray, visible light, infrared ray, microwave and radio wave

2 SB 41
Rajah di bawah menunjukkan komponen-komponen bagi gelombang elektromagnet. TP 2 OP3
The diagram below shows components for electromagnetic waves.

Medan elektrik
Electric field

Arah perambatan
Direction of propagation

Medan magnet
Magnetic field

SB 43
(a) Namakan dua komponen bagi gelombang elektromagnet.
Name two components of electromagnetic wave.
Medan elektrik dan medan magnet

Electric field and magnetic field

(b) Apakah yang boleh anda katakan mengenai arah getaran kedua-dua komponen itu?
What can you say about the direction of vibration of both components?
Berserenjang/ Perpendicular

(c) Apakah jenis gelombang elektromagnet?


What is the type of electromagnetic wave?
Gelombang melintang/ Transverse waves

SB 45

© Penerbit Mahir Sdn. Bhd. (183897-P) 114

Fizik Tg 4 (Bab 5)5th 9/2/23.indd 114 09/02/2023 5:57 PM


Fizik Tingkatan 4 Bab 5
3 Aplikasi gelombang elektromagnet. TP 5 OP6
Applications of electromagnetic waves.

Sumber Aplikasi
Source Application

(a) Sinar gama • Membunuh sel kanser dalam radioterapi


Gamma ray Kills cancer cells in radiotherapy
• Digunakan dalam industri pemprosesan makanan supaya makanan tahan
lebih lama
Used in food processing industry so that food can last longer

(b) Sinar-X • Mengesan retakan atau patah pada tulang dan pemeriksaan organ
X-ray
dalaman
Detects fractures or broken and examines internal organs
• Pengimbas bagasi di lapangan terbang
Baggage scanning at airport

(c) Sinar ultraungu • Mengeraskan bahan tampalan gigi


Ultraviolet ray Hardens tooth filling material
• Menentukan kesahihan wang kertas
Determines authenticity of currency notes

(d) Cahaya nampak • Membolehkan benda hidup untuk melihat


Visile light Enables living things to see
• Fotografi
Photography

(e) Inframerah
• Untuk memasak (ketuhar, pemanggang dan pembakar)
Infrared
For cooking (oven, grill and toaster)
• Alat kawalan jauh untuk televisyen dan pemain DVD
Remote control device for television and DVD player

(f) Gelombang mikro • Komunikasi antara alat elektronik (wifi, bluetooth, zigbee)
Microwave Communication between electronis devices (wifi, bluetooth, zigbee)
• Pengesanan radar pesawat dan pemerangkap laju
Detection of plane radar and speed trap

(g) Gelombang radio • Komunikasi radio jarak jauh


Radio wave Long distance radio communication
• Mesin gelombang millimeter untuk mengimbas badan penumpang di
lapangan terbang
Millimeter-wave machine to scan body of passengers at airport

VIDEO
bit.ly/3Qq0o9Z
Kegunaan Gelombang Elektromagnet
Uses of Electromagnetic Waves

115 © Penerbit Mahir Sdn. Bhd. (183897-P)

Fizik Tg 4 (Bab 5)5th 9/2/23.indd 115 09/02/2023 5:57 PM


SB 40
SB 41

Fizik Tingkatan 4 Bab 5

Praktis Sumatif
SB 40

KERTAS 1

1 Rajah 1 menunjukkan dua keadaan di mana sebiji 2


SB 43
Rajah 2 menunjukkan corak muka gelombang
bola terapung dan bergerak di atas permukaan air. terhasil oleh pencelup bergetar dengan frekuensi
OP3
SB 42
Diagram 1 shows two situations where a ball floats and
OP3
12 Hz di dalam tangki riak.
moves on the surface of the water. Diagram 2 shows a wavefront pattern produces by a
dipper vibrating at a frequency of 12 Hz in a ripple tank.
Muka gelombang
Bola Arah perambatan gelombang Wavefronts
Ball Direction of wave propagation
6 cm

Air
SB 42 Water

Rajah 2
Diagram 2
Arah perambatan gelombang
Direction of wave propagation
SB 45
Berapakah laju gelombang?
What is the speed of the waves?
A 2 cm s–1 C 12 cm s–1
Bola SB Air
44 B 8 cm s–1 D 18 cm s–1
Ball Water
3 Antara berikut, kuantiti fizik yang manakah
OP4 berkurang apabila sistem mengalami pelembapan?
Which of the following physical quantity decreases
Rajah 1 when a system undergoes damping?
Diagram 1 A Amplitud/ Amplitude
B Halaju/ Velocity
Arah pergerakan bola adalah C Panjang gelombang/ Wavelength
SB 44 of the movement of the ball is
The direction
D Frekuensi/ Frequency
A selari dengan arah perambatan gelombang
parallel to the direction of the wave propagation 4 Satu sistem mempunyai frekuensi asli bersamaan
B berserenjang dengan arah perambatan
gelombang
OP3
SB 46
f. Satu daya luar memberikan tenaga kepada sistem
itu untuk meneruskan ayunan.
perpendicular to the direction of the wave Resonans akan berlaku dalam sistem itu jika
propagation tenaga diberikan kepada sistem itu pada frekuensi
C sama arah dengan arah perambatan yang sama dengan
gelombang A system has a natural frequency of oscillation equals to
the same as the direction of the wave propagation f. An external force is supplying energy to the system to
D bertentangan dengan arah perambatan continue the oscillations of the system.
gelombang Resonance will occur in this system if energy is supplied
SB 46
the opposite to the direction of the wave
propagation.
to the system at a frequency equal to
f
A f C
2
f
B 2f D
2

© Penerbit Mahir Sdn. Bhd. (183897-P) 116

Fizik Tg 4 (Bab 5)5th 9/2/23.indd 116 09/02/2023 5:57 PM


Fizik Tingkatan 4 Bab 5
5
Rajah 3 menunjukkan muka gelombang satah Hitung jarak kapal selam dari batu besar itu.
OP53 ditujukan ke pemantul satah.
TINGKATAN 4 BATCH ARTWORK BY FREELANCER Calculate
M.ZAIDEE the distance of the submarine from the big
KBAT Diagram 3 shows the wavefronts of a plane wave rock.
incident on a plane reflector. [Halaju gelombang ultrasonik = 1 560 m s–1]
[Velocity of ultrasonic wave = 1 560 m s–1]
A 3.9 km
B 7.8 km
Gelombang tuju
Incident wave
C 15.6 km
D 31.2 km
Pemantul satah SB 37
Plane reflector 7 Antara berikut, yang manakah menunjukkan kesan
SB 36
OP4 gelombang air yang bergerak dari kawasan dalam
Rajah 3 KBAT ke kawasan cetek?
Diagram 3 Which of the following shows the effect of water waves
travelled from deep region to shallow region?
Pernyataan yang manakah betul tentang
gelombang pantulan dan gelombang tuju? v λ
Which statement is correct about the reflected waves
and incident waves? Bertambah Malar
A Laju gelombang pantulan sama dengan laju A
SB 39 Increasing Constant
gelombang tuju.
The speed of the reflected waves is the same as the Malar Berkurang
speed of the incident waves. B
Constant Decreasing
B Panjang gelombang bagi gelombang pantulan
SB 40
lebih pendek daripada panjang gelombang Berkurang Berkurang
bagi gelombang tuju. C
Decreasing Decreasing
The wavelength of the reflected waves is shorter
than that of the incident waves. Bertambah Bertambah
C Frekuensi gelombang pantulan lebih kecil D
Increasing Increasing
daripada frekuensi gelombang tuju.
The frequency of the reflected waves is lower than
8 Rajah 5 menunjukkan pembiasan gelombang air.
that of the incident waves.
OP3 Diagram 5 shows a refraction of a water wave.
D Arah gelombang pantulan sentiasa bersudut
tegak dengan arah gelombang tuju. 2.0 cm
SB 41 SB 42
The directions of the reflected waves are always at
1.5 cm
right angles to the incident waves.

6 Rajah 4 menunjukkan sebuah kapal selam


OP4 memancarkan gelombang ultrasonik ke arah batu
besar di dasar laut. Selepas 10 saat, kapal selam
itu mengesan gelombang yang dipantulkan.
Diagram 4 shows a submarine transmitting ultrasonic Air dalam
waves directed at a big rock on the seabed. Deep water
SB 44
B 43 After 10 seconds, the submarine detects the reflected wave. Air cetek
Shallow water
Batu besar
Big rock Rajah 5
Gelombang ultrasonik Diagram 5
Ultrasonic waves
Jika laju gelombang air ialah 4 cm s–1 di kawasan
air dalam, hitung laju gelombang air di kawasan
SB 46
air cetek.
If the speed of the water wave is 4 cm s–1 in the deep
Kapal selam water region, calculate the speed of the water wave in
Submarine
the shallow water region.
Rajah 4 A 0.8 cm s–1 C 3.0 cm s–1
Diagram 4 B 2.5 cm s–1 D 5.3 cm s–1

117 © Penerbit Mahir Sdn. Bhd. (183897-P)

Fizik Tg 4 (Bab 5)5th 9/2/23.indd 117 09/02/2023 5:57 PM


SUPERB SBP FIZIK TINGKATAN 4 BATCH 3 ARTWORK BY FRE
SUPERB
Fizik Tingkatan 4 Bab 5SBP FIZIK TINGKATAN 4 BATCH 3 ARTWO
9 Rajah manakah menunjukkan gelombang air yang 12 Rajah 8 menunjukkan satu susunan radas eksperimen
OP4 melalui satu celah kecil dengan betul? OP3dwicelah Young. Panjang gelombang cahaya
N 4 BATCH 3 KBAT Which diagram
SUPERB shows
SBP FIZIKARTWORKthe
TINGKATAN water3 waves
BY4FREELANCER
BATCH that passes
M.ZAIDEE monokromatik ialah 6.5
ARTWORK BY FREELANCER × 10–7 m. Jarak di antara
M.ZAIDEE
through a small gap correctly? dua celah ialah 0.5 × 10–3 m dan jarak di antara
A C dwicelah dengan skrin ialah 3 m.
Diagram 8 shows the apparatus set-up of Young’s double-
slit experiment. The wavelength
SB 48of the monochromatic
IK TINGKATAN
PERB SBP FIZIK4TINGKATAN
BATCH 3 4 BATCH 3 ARTWORKSB 47
BYARTWORK
FREELANCER M.ZAIDEE M.ZAIDEE
BY FREELANCER
light is 6.5 × 10–7 m. The distance between the two slits
is 0.5 × 10–3 m and the distance between the double-slit
B
SB 48 SB 47 D SB 49 SB 48 SB 49
and the screen is 3 m.
Skrin
Dwicelah Screen
SB 47 Double-slit SB 48
SB 47 SB 48 SB 48 SB 49 SB 49
10 Rajah 6 menunjukkan gelombang air melepasi
OP2 sebongkah batu dalam sungai.
Diagram 6 shows water passing a block of rock in a
Corak pinggir
river. SB 50 Sumber cahaya
interferens
SB 51 SB 51 monokromatik SB 52
SB
SB 52 50 Interference
Monochromatic
ERB SBP FIZIK TINGKATAN 4 BATCH 3 Batu ARTWORK BY FREELANCER M.ZAIDEE fringe pattern
light source
Rock SB 51
Rajah 8
SB 50 Diagram 8
SB 51 SBX51 SB 52 SB 52
Berapakah jarak di antara dua pinggir cerah?
What is the distance between two bright fringes?
Rajah 6 A 3.9 × 10–3 m C 1.1 × 10–10 m
Diagram 6
B 7.8 × 10–3 m D 1.6 × 10–10 m
Fenomena SBSB
yang 53SB 50
manakah
54 menerangkan SB 54

SB 47 perambatan gelombang di X? SB 48 13 Rajah 9 menunjukkanSBalat


49 mengesan bagasi yang
Which phenomenom explains the propagation of waves SB 51
OP4 digunakan di lapangan terbang untuk keselamatan
at X? KBAT mengesan bagasi penumpang.
A Pantulan/ Reflection Diagram 9 shows a luggage scanning device used in an
SB 53 SB 53 SB 54 SB 54 airport for security inspection of passengers’ luggage.
B Pembiasan/ Refraction
C Pembelauan/ Diffraction SB 53 SB 54
D Interferens/ Interference
SB 55 SB 56 SB 56
Alat mengesan
11 Rajah 7 menunjukkan corak interferens bagi dua bagasi
Bagasi
OP2 sumber koheren bagi gelombang air, P dan Q. Luggage scanner
Luggage
Diagram 7 shows interference pattern of two coherent
SB 55 SB 55 sources for water waves, P and Q.
SB 56 SB 56
SB 50
Puncak Lembangan
Crest SB 51
Trough Rajah 9
SB 52
SB 57 Diagram 9
A Sinar manakah yang sesuai untuk mengesan
D B
SB 55 kandungan dalam bagasi? SB 56
Which ray is suitable for scanning the contents of the
C
SB 57 luggage?
P Q
SB 53 A Sinar inframerah
Infrared ray
Rajah 7 B Gelombang ultrasonik
Diagram 7
Ultrasonic waves
Antara kedudukan A, B, C dan D, yang manakah C Sinar gamma
menunjukkan amplitud gelombang adalah sifar? Gamma rays
At which of the positions A, B, C and D shows the D Sinar X
SB 53amplitude of the wave is zero? X-ray
SB 54
SB 57
© Penerbit Mahir Sdn. Bhd. (183897-P) 118

Fizik Tg 4 (Bab 5)5th 9/2/23.indd 118 10/02/2023 10:12 AM


SB 50
Fizik Tingkatan 4 Bab 5
SB 51
KERTAS 2

SB 51
Bahagian A
1 Rajah 1.1 menunjukkan sebuah penjana audio disambungkan kepada pembesar suara. SB 52
B 51 Diagram 1.1 shows an audio generator connected to a loudspeaker.
SB 52
Pembesar suara
Penjana audio Speaker
Audio generator

Rajah 1.1
Diagram 1.1
SB 53 SB
Rajah 1.2 dan Rajah 1.3 masing-masing menunjukkan pembelauan gelombang bunyi daripada pembesar suara
apabila bunyi dengan frekuensi rendah dan frekuensi tinggi dihasilkan.
Diagram 1.2 and Diagram 1.3 show the diffraction of sound wave from the loudspeaker when the sound of low and high
frequency are produced respectively.

Bunyi berfrekuensi rendah Bunyi berfrekuensi tinggi


Sound of low frequency Sound of high frequency

SB 55 SB 5

Rajah 1.2 Rajah 1.3


Diagram 1.2 Diagram 1.3

(a) Apakah yang dimaksudkan dengan pembelauan?


SB 54
OP2
What is the meaning of diffraction?
SB 57
SB 54
Penyebaran gelombang apabila gelombang itu merambat melalui satu celah atau tepi suatu penghalang

The spreading of waves when the waves propagate through a slit or side of a barrier.
[1 markah/ mark]

(b) Pada Rajah 1.2, tanda dan labelkan jarak gelombang dengan λ. OP3
In Diagram 1.2, mark and label the wavelength with λ.
[1 markah/ mark]

(c) Perhatikan Rajah 1.2 dan Rajah 1.3. OP4


Observe Diagram 1.2 and Diagram 1.3.

(i) Bandingkan jarak gelombang.


Compare the wavelengths.
Panjang gelombang dalam Rajah 1.2 lebih panjang daripada dalam Rajah 1.3.

The wavelength in Diagram 1.2 is longer than in Diagram 1.3.


[1 markah/ mark]

119 © Penerbit Mahir Sdn. Bhd. (183897-P)

SB 56
Fizik Tg 4 (Bab 5)5th 9/2/23.indd 119 09/02/2023 5:57 PM
SB 47 SB 47 SB 48 SB 48 SB 49
Fizik Tingkatan 4 Bab 5
(ii) Bandingkan bentuk pembelauan gelombang bunyi.
Compare the patterns of the diffraction of sound waves.
Bentuk pembelauan gelombang bunyi dalam Rajah 1.2 lebih membulat daripada dalam Rajah 1.3.

The pattern of the diffraction of sound wave in Diagram 1.2 is more circular than in Diagram 1.3.
[1 markah/ mark]

(d) Apakah yang akan terjadi kepada amplitud gelombang bunyi yang terbelau? OP3
What will be happen to the amplitude of the diffracted sound waves?
Berkurang/Decreases
SB 50 SB 50 [1 markah/ mark]
(e) Terangkan jawapan anda di 1(d). OP4
Explain your answer in 1(d). SB 51 SB 51 SB 52
Tenaga telah berkurang kerana disebarkan ke kawasan lebih luas selepas melalui celah.

Energy decreases. It spreads out to larger area after passing through the gap.
[1 markah/ mark]

(f) Nyatakan hubungan antara panjang gelombang dan amplitud gelombang yang terbelau. OP2
State the relationship between wavelength and amplitude of the diffracted wave.
Semakin panjang panjang gelombang, semakin rendah amplitud gelombang

The longer the wavelength, the smaller the amplitude of the wave
[1 markah/ mark]

Bahagian B

2 Rajah 2.1SB 53
menunjukkan SBmerah
pinggir-pinggir 53 yang terbentuk dalam eksperimen dwicelah SBYoung.
54 Rajah 2.2 SB 54
menunjukkan pinggir-pinggir hijau yang terbentuk apabila cahaya merah diganti dengan cahaya hijau. Panjang
gelombang cahaya merah dan hijau masing-masing adalah 7.0 × 10–7 m dan 5.0 × 10–7 m.
Diagram 2.1 shows the red fringes formed in a Young’s double-slit experiment. Diagram 2.2 shows the green fringes
formed when the red light is replaced by the green light. The wavelengths of red light and green light are 7.0 × 10–7 m and
5.0 × 10–7 m respectively.

Pinggir merah Pinggir hijau


Red fringes Green fringes

Rajah 2.1 Rajah 2.2


SB 55 Diagram 2.1 SB 55 Diagram 2.2SB 56 SB 56
(a) Apakah maksud panjang gelombang? OP2
What is the meaning of wavelength?
[1 markah/ mark]

(b) Berdasarkan Rajah 2.1 dan Rajah 2.2, bandingkan panjang gelombang cahaya, jarak di antara dua pinggir
berturutan yang terbentuk dan jarak di antara dua pinggir berturutan. Hubungkaitkan panjang gelombang
cahaya dengan jarak di antara dua pinggir berturutan. Namakan fenomena gelombang yang terlibat.
Based on Diagram 2.1 and Diagram 2.2, compare the wavelength of the light, the distance between two consecutive
fringes formed and the distance between two consecutive fringes. Relate the wavelength of the light with the distance
between two consecutive fringes. Name the wave phenomenon involved. OP4
[5 markah/ marks]
SB 57 SB 57

© Penerbit Mahir Sdn. Bhd. (183897-P) 120

Fizik Tg 4 (Bab 5)5th 9/2/23.indd 120 09/02/2023 5:57 PM


SB 55
Fizik Tingkatan 4 Bab 5
(c) Rajah 2.3 menunjukkan reka bentuk sebuah piring yang dibuat oleh seorang murid untuk mendidihkan
air di dalam cerek menggunakan tenaga suria. OP7 KBAT
Diagram 2.3 shows the design of a dish made by a student to boil water in a kettle using solar energy.

Cerek
Kettle

Piring
Dish

Tapak
Base

Rajah 2.3
Diagram 2.3
SB 57
Anda dikehendaki untuk mengubahsuaikan reka bentuk dalam Rajah 2.3 supaya air mendidih dalam
masa yang lebih singkat. Nyatakan dan terangkan pengubahsuaian berdasarkan aspek-aspek berikut:
You are required to modify the design in Diagram 2.3 so that the water boils in a shorter time. State and explain the
modificication based on the following aspects:
(i) Bentuk piring
The shape of the dish
(ii) Jenis permukaan piring
The type of surface of the dish
(iii) Saiz piring
The size of the dish
(iv) Kedudukan cerek
The position of the kettle
(v) Orientasi piring
The orientation of the dish
[10 markah/ marks]

121 © Penerbit Mahir Sdn. Bhd. (183897-P)

Fizik Tg 4 (Bab 5)5th 9/2/23.indd 121 09/02/2023 5:57 PM


Fizik Tingkatan 4 Bab 6

BAB
6
Cahaya dan Optik
Light and Optics

Tema Gelombang, Cahaya dan Optik


Theme Waves, Light and Optics

NOTA EFEKTIF

1 Pembiasan cahaya 3 Dalam nyata & dalam ketara


Refraction of light Real depth & apparent depth
Fenomena cahaya bertukar arah apabila ia Indeks biasan/ Refractive index,
merambat melalui dua medium berlainan dalam sebenar/ real depth H
ketumpatan disebabkan halaju cahaya yang n= =
dalam ketara/ apparent depth h
berubah.
A phenomenon when light changes direction when 4 Pantulan dalam penuh/ Total internal reflection
it travels from one medium to another medium of Semua cahaya dipantulkan dalam medium
different density due to the change of velocity of light. apabila sudut tuju, i dalam medium yang
2 Indeks biasan tumpat melebihi sudut genting, c.
Refractive index All light are reflected in the medium when incidence
Nisbah laju cahaya di dalam vakum kepada laju angle, i in the denser medium is larger than the critical
cahaya di dalam medium angle, c.
The ratio of speed of light in vacuum to the speed of • Sinar cahaya merambat dari suatu medium
light in medium yang lebih tumpat ke medium yang kurang
laju cahaya dalam vakum/ udara tumpat.
n= All light propagate from denser medium to less dense
laju cahaya dalam medium
c medium.
speed of light in vacuum/ air n=
n= • i>c
speed of light in medium v
sin i sin 90° 1
Sinar tuju Garis normal n= = =
Normal line sin r sin c sin c
Incident ray
1 1
Udara/ Air sin c = =
i
(n = 1)
n 1.5
c = 42°
Medium (n)
r
sin i Kaca
Kaca r=c
n= Glass
Glass sin r
Sinar biasan
Refracted ray i Udara
Air
sin i
n= 5 Kanta cembung
sin r Convex lens
n : indeks biasan/ refractive index Titik fokus, F
i : sudut kawasan kurang tempat Focal point
angle of less dense region
Paksi utama
r : sudut kawasan lebih tumpat Principal axis F
angle of denser region

Pusat optik, O Panjang fokus, f


Optical centre Focal length

© Penerbit Mahir Sdn. Bhd. (183897-P) 122

Fizik Tg 4 (Bab 6)6th 10/2/23.indd 122 10/02/2023 9:54 AM


Fizik Tingkatan 4 Bab 6

Jarak objek Ciri-ciri imej v hi
m= =
Object distance, u Characteristics of image u ho

Nyata, songsang, diperkecil Persamaan kanta nipis/ Thin-lens equation:


u=∞
Real, inverted, diminished
1=1+1
Nyata, songsang, diperkecil f u v
u > 2f
Real, inverted, diminished
7 Alatan optik
Nyata, songsang, sama saiz Optical instruments
u = 2f
Real, inverted, same size (a) Kanta pembesar
Magnifying glass
Nyata, songsang, diperbesar
f < u < 2f (b) Mikroskop majmuk
Real, inverted, magnified
Compound microscope
Imej berada di infiniti (c) Teleskop astronomi
u=f
Image at infiniti Astronomical telescope
8 Cermin cekung
Maya, tegak, diperbesar Concave mirror
u<f
Virtual, upright, magnified
Jarak objek Ciri-ciri imej
Object distance, u Characteristics of image
6 Kanta cekung/ Concave lens
Pusat optik, O Nyata, songsang,
Optical centre u=∞ diperkecil
Real, inverted, diminished

Titik fokus, F Nyata, songsang,


Focal point Paksi utama u > 2f diperkecil
F Principal axis Real, inverted, diminished
Nyata, songsang, sama
u = 2f saiz
Real, inverted, same size
Panjang fokus, f
Focal length Nyata, songsang,
f < u < 2f diperbesar
Real, inverted, magnified
Jarak objek Ciri-ciri imej
Object distance, u Characteristics of image Imej berada di infiniti
u=f Image at infiniti
u pada sebarang
Maya, tegak, diperkecil Maya, tegak, diperbesar
kedudukan u<f
Virtual, upright, diminished Virtual, upright, magnified
u at any position
9 Cermin cembung
Pembesaran linear, Convex mirror
Linear magnification,
jarak imej/ image distance Jarak objek Ciri-ciri imej
m= Object distance, u Characteristics of image
jarak objek/ object distance
tinggi imej/ height of image Maya, tegak, diperkecil
m= u>f
Virtual, upright, diminished
tinggi objek/ height of object
Maya, tegak, diperkecil
u<f
Virtual, upright, diminished

123 © Penerbit Mahir Sdn. Bhd. (183897-P)

Fizik Tg 4 (Bab 6)6th 10/2/23.indd 123 10/02/2023 9:54 AM


Fizik Tingkatan 4 Bab 6

6.1 Pembiasan Cahaya


Refraction of Light

Buku Teks: m.s 232-241

INFO
Pemerhati
Rajah menunjukkan ikan kelihatan lebih dekat dengan permukaan air. Observer
Cahaya bergerak lurus apabila melalui bahan lut sinar (udara, air dan kaca).
Imej ikan
Jika cahaya bergerak melalui dua medium yang berlainan ketumpatan, ia
Fish image
akan berubah arah. Fenomena ini dikenali sebagai pembiasan cahaya.
Diagram shows the fish appears closer to the surface of water. Light travels in a
straight line through transparent materials (air, water and glass). If light passes
Air
through two materials of different optical densities, it will change direction. The light
Ikan Water
bent. This phenomenon is called the refraction of light.
Fish

1 Apakah maksud pembiasan cahaya? TP 1 OP2


What is the meaning of refraction of light?

Fenomena di mana arah cahaya berubah apabila bergerak melalui dua bahan berlainan ketumpatan optik.
Phenomenon where the direction of light changes when it travels through two materials of different optical densities.

2 Lukis arah sinar pembiasan. Tandakan sudut tuju, i dan sudut biasan, r. TP 2 OP3
Draw the direction of refracted ray. Mark the angle of incidence, i and the angle of refraction, r.

Sinar tuju Garis normal


Incident Normal line
Sinar tuju Garis normal
ray i Udara Incident Normal line
Air ray Kaca
i
Kaca Glass
r Glass Udara
r Air

Rajah (a) Rajah (b)


Diagram (a) Diagram (b)

3 Nyatakan perubahan pada laju cahaya setelah pembiasan. TP 2 OP3 State the change in velocity of light
after refraction.

Rajah (a): Berkurang Rajah (b): Bertambah

Diagram (a): Decreases Diagram (b): Increases

4 Apakah indeks biasan? TP 1 OP2


What is refractive index?

Nisbah laju cahaya di dalam vakum kepada laju cahaya di dalam medium.
The ratio of speed of light in vacuum to the speed of light in medium.

© Penerbit Mahir Sdn. Bhd. (183897-P) 124

Fizik Tg 4 (Bab 6)6th 10/2/23.indd 124 10/02/2023 9:54 AM


Fizik Tingkatan 4 Bab 6
5 Nyatakan persamaan Hukum Snell. TP 1 OP2
State the equation of Snell’s law.

n1 sin θ1 = n2 sin θ2

di mana n1
where indeks biasan dalam medium 1/ refractive index in medium 1
n2 indeks biasan dalam medium 2/ refractive index in medium 2
θ1 sudut tuju dalam medium 1/ angle of incidence in medium 1
θ2 sudut biasan dalam medium 2/ angle of refraction in medium 2

EKSPERIMEN 6.1
Rajah 1.1 dan Rajah 1.2 menunjukkan rajah sinar daripada ikan ke mata pemerhati. Didapati pembengkokan
cahaya adalah pada sudut yang berbeza apabila kedudukan mata pemerhati berubah dari X ke Y.
Diagram 1.1 and Diagram 1.2 shows the ray diagram from fish to observer’s eyes. It is found that the bending of light
ray at different angles when the position of eyes change from X to Y.

Pemerhati Pemerhati
Pemerhati Pemerhati
Observer Observer
X X Y Observer Y Observer

Rajah 1.1 Rajah 1.2


Diagram 1.1 Diagram 1.2

Inferens: Sudut biasan, r bergantung kepada sudut tuju, i.


Angle of refraction, r depends on angle of incidence, i.
Inference:

Hipotesis: Semakin bertambah sudut tuju, semakin bertambah sudut biasan.


The larger the angle of incidence, the larger the angle of refraction.
Hypothesis:

Tujuan: Mengkaji hubungan antara sudut tuju dengan sudut biasan dan menentukan indeks
biasan bagi blok kaca.
To investigate the relationship between angle of incidence and angle of refraction and to determine the refractive
Aim:
index of glass block.

Pemboleh ubah dimanipulasi: Sudut tuju, i


Angle of incidence, i
Manipulated variable:

Pemboleh ubah bergerak balas: Sudut biasan, r


Angle of refraction, r
Responding variable:

Pemboleh ubah dimalarkan: Ketumpatan kaca


Density of glass
Constant variable:

125 © Penerbit Mahir Sdn. Bhd. (183897-P)

Fizik Tg 4 (Bab 6)6th 10/2/23.indd 125 10/02/2023 9:54 AM


Fizik Tingkatan 4 Bab 6

Radas: Kotak sinar dengan plat celah tunggal, jangka sudut dan pembaris, blok kaca, kertas putih dan pensel.
Ray box with single slit, protractor, ruler, glass block, white paper and pencil.
Apparatus:

Prosedur/ Procedure:
(i) Lukis garis luar blok kaca itu pada kertas putih dan garis
normal melalui titik O. Lukis garis pada sudut tuju, i = 20°
dengan menggunakan jangka sudut. Tujukan sinar cahaya dari Kotak sinar
Ray box
kotak sinar pada sudut tuju itu. Lukis sinar pada PQ.
Bongkah kaca
Trace the outline of the glass block on the white paper and draw a i Glass block
normal line at point O. Draw a lines at angles of incidence, i = 20°
O
using a protractor. Direct the light ray from the ray box at the
angle of incidence. Draw the emergent ray at PQ.
(ii) Alihkan bongkah kaca dan lukis sinar biasan OP. Ukur r
sudut biasan, r.
Remove the glass block and draw the refracted ray OP. Measure
the angle of refraction, r. P
(iii) Letakkan blok kaca semula. Ulangi langkah eksperimen
dengan sudut tuju, i = 30°, 40°, 50° dan 60°. Hitungkan
nilai sin i dan sin r. Q
Put the glass block back in place. Repeat the experiment steps
with angle of incidence, i = 30°, 40°, 50° and 60°. Calculate
sin i and sin r.

Keputusan/ Result:
(i) Jadualkan data bagi nilai i, r, sin i, sin r. (ii) Graf sin i melawan sin r diplot.
Tabulate data for values i, r, sin i and sin r. Graph of sin i against sin r is plotted.

sin i sin i
i r sin i sin r
sin r
1.0
20° 13° 0.34 0.22 1.55
0.9
30° 20° 0.50 0.34 1.47
0.8
40° 25° 0.64 0.42 1.52
0.7
50° 30° 0.77 0.50 1.54
0.6
60° 25° 0.87 0.57 1.53
0.5

0.4

0.3

0.2

0.1

sin r
0.1 0.2 0.3 0.4 0.5 0.6 0.7

© Penerbit Mahir Sdn. Bhd. (183897-P) 126

Fizik Tg 4 (Bab 6)6th 10/2/23.indd 126 10/02/2023 9:54 AM


Fizik Tingkatan 4 Bab 6

Kesimpulan/ Conclusion:
sin i
Sin i berkadar terus dengan sin r. Hukum Snell: = malar.
sin r
sin i
Sin i directly proportional to sin r. Snell’s law: = constant.
sin r
Perbincangan/ Discussion:
(i) Apakah yang berlaku kepada sudut biasan, r apabila sudut tuju, i bertambah?
What happens to the angle of refraction, r when the angle of incidence, i increases?
Bertambah/ Increases

(ii) Hitung kecerunan graf. Tunjukkan pada graf.


Calculate the gradient of graph. Show on the graph.
0.91 – 0
m= = 1.52
0.60 – 0
(iii) Apakah nilai indeks biasan, n bagi bongkah kaca?
What is the value of refractive index, n of of the glass block?
n = m = 1.52

6 Rajah menunjukkan sebuah objek di dalam sebuah bikar berisi air dan diperhatikan oleh seorang pemerhati.
The diagram shows an object in a beaker filled with water and observed by an observer. TP 2 OP3

Pemerhati
Observer

Dalam ketara, h
Apparent depth, h
Imej Dalam nyata, H
Image Real depth, H

O
Objek
Object

(a) Lukis rajah sinar dari titik O ke mata pemerhati untuk menunjukkan bagaimana objek kelihatan lebih
dekat dengan permukaan air.
Draw a ray diagram from point O to the observer’s eye to show how the object appears nearer to the surface
of water.

(b) Tanda dan labelkan dalam nyata, H dan dalam ketara, h pada rajah di atas.
Mark and label real depth, H and apparent depth, h on the diagram above.

(c) Tulis persamaan bagi indeks biasan, n yang melibatkan dalam nyata, H dan dalam ketara, h.
Write an equation for refractive index which involved real depth, H and apparent depth, h
H
n=
h

127 © Penerbit Mahir Sdn. Bhd. (183897-P)

Fizik Tg 4 (Bab 6)6th 10/2/23.indd 127 10/02/2023 9:54 AM


Fizik Tingkatan 4 Bab 6

EKSPERIMEN 6.2

Inferens: Kedudukan imej mempengaruhi kedudukan objek// Dalam nyata mempengaruhi dalam ketara.
Position of an image is influenced by position of object// Real depth influences the apparent depth.
Inference:

Hipotesis: Semakin bertambah dalam nyata, H, semakin bertambah dalam ketara, h.


The greater the real depth, H, the greater the apparent depth, h.
Hypothesis:

Tujuan: Mengkaji hubungan antara dalam nyata, H dengan dalam ketara, h.


To investigate the relationship between real depth, H and apparent depth, h.
Aim:

Pemboleh ubah dimanipulasi: Dalam nyata, H


Real depth, H
Manipulated variable:

Pemboleh ubah bergerak balas: Dalam ketara, h


Apparent depth, h
Responding variable:

Pemboleh ubah dimalarkan: Ketumpatan cecair


Density of liquid
Constant variable:

Radas: Bikar 1 000 ml, pembaris, kaki retort dan dua pengapit, gabus, dua batang pin, pita selofan dan air.
1 000 ml beaker, ruler, retort stand with two clamps, cork, two pins, cellophane tape and water.
Apparatus:

Prosedur/ Procedure:
1 Letakkan sebatang pin PO dengan pita selofan pada dasar bikar itu.
Stick a pin PO with cellophane tape to the base of the beaker.
30
29

2 Isi air ke dalam bikar sehingga kedalaman


28
27
26

6.0 cm. Kedalaman ini ialah dalam nyata, H. Pembaris


25
24

Fill the beaker with water to a depth of 6.0 cm. Ruler


23
22

This depth is real depth, H.


21
20

3 Perhatikan imej pin PO dari bahagian atas Kaki retort


19
18

Retort stand
17

permukaan air.
16
15

Observe the image of pin PO from above the Bikar


14
13

water surface. Beaker


12
11

4 Laraskan kedudukan pin P1 secara menegak


10

Air
9
8

sehingga kelihatan segaris dengan imej pin Water


7
6

PO. Pada masa ini, kedudukan pin P1 adalah


5

Imej pin PO
4
3

sama aras dengan imej pin PO. Image of pin PO y


2
0 cm 1

Adjust the position of pin P1 vertically until it


appears in line with the image of pin PO. At this Pin PO Pin P1
level, position of pin P1 is at the same level as the Gabus
image of pin PO. Cork
5 Ukur jarak, y iaitu antara pin P1 dengan dasar bikar. Rekodkan bacaan dalam jadual.
Measure the distance, y which is between pin P1 and the base of the beaker. Record the reading in table.
6 Ulangi eksperimen ini dengan mengubah dalam nyata, H = 7.0 cm, 8.0 cm, 9.0 cm dan 10.0 cm.
Repeat the experiment with real depth, H = 7.0 cm, 8.0 cm, 9.0 cm and 10.0 cm.

© Penerbit Mahir Sdn. Bhd. (183897-P) 128

Fizik Tg 4 (Bab 6)6th 10/2/23.indd 128 10/02/2023 9:54 AM


Fizik Tingkatan 4 Bab 6
Keputusan/ Result:
(i) Jadualkan data bagi nilai H, y dan h.
Tabulate data for values H, y and h.

Dalam nyata, H / cm Dalam ketara, h / cm


y / cm
Real depth Apparent depth

6.0 1.6 4.4

7.0 1.5 5.5

8.0 2.0 6.0

9.0 2.0 7.0

10.0 2.6 7.4

(ii) Lakarkan graf H melawan h.


Plot graph of H against h.

H/ cm

10

0 1 2 3 4 5 6 7 8 h/ cm

Kesimpulan/ Conclusion:
Dalam nyata, H berkadar terus dengan dalam ketara, h.

The real depth, H is directly proportional to the apparent depth, h.

Perbincangan/ Discussion:
(i) Hitung kecerunan graf. Tunjukkan pada graf.
Calculate the gradient of graph. Show on the graph.
9.3 – 0
m= = 1.34
7.0 – 0

129 © Penerbit Mahir Sdn. Bhd. (183897-P)

Fizik Tg 4 (Bab 6)6th 10/2/23.indd 129 10/02/2023 9:54 AM


Fizik Tingkatan 4 Bab 6

(ii) Hubung kait dalam nyata (H), dalam ketara (h) dan indeks biasan air (n).
Relate the real depth (H), apparent depth (h) and refractive index of water (n).
H
n=
h
(iii) Apakah nilai indeks biasan air, n?
What is the value of refractive index of water, n?
n = m = 1.34

7 Menyelesaikan masalah melibatkan pembiasan cahaya. TP 3 OP4


Solve problem involving refraction of light.

(a) Satu sinar cahaya merambat daripada kaca ke udara. Indeks biasan kaca ialah 1.54 dan laju cahaya
dalam udara ialah 3 × 108 m s–1. Hitung
A light ray travels from glass to air. The refractive index of glass is 1.54 and the speed of light in air is
3 × 108 m s–1. Calculate

(i) sudut biasan, θ.


the angle of refraction, θ. Udara
θ
sin θ Air
n= = 1.54
sin 33°
sin θ = 1.54 × sin 33°
= 0.838 Kaca
33° Glass
θ = 57°

(ii) laju cahaya dalam kaca.


the speed of light in glass. Sinar cahaya
Light ray
3 × 108
n= = 1.54
v
3 × 108
v=
1.54
= 1.94 × 108 m s–1

(b) Satu sinar cahaya tidak lengkap memasuki sebuah prisma kaca yang mempunyai indek biasan 1.50.
An incomplete light ray entering a glass prism which has a refractive index of 1.50.

Prisma kaca
Glass prism

Sinar cahaya
Light ray 30°

(i) Pada rajah di atas, lengkapkan rajah sinar.


On the diagram above, complete the ray diagram.

(ii) Cari sudut biasan.


Find the angle of refraction.
sin r
n= = 1.50
sin 30°
sin r = 1.50 × sin 30°
= 0.75
r = 48.6°

© Penerbit Mahir Sdn. Bhd. (183897-P) 130

Fizik Tg 4 (Bab 6)6th 10/2/23.indd 130 10/02/2023 9:54 AM


Fizik Tingkatan 4 Bab 6

6.2 Pantulan Dalam Penuh


Total Internal Reflection

Buku Teks: m.s 242-250

INFO
Sebutir intan terkenal dengan rupanya yang berkilau.
Fenomena disebabkan oleh sifat cahaya iaitu pantulan dalam penuh.
Diamonds are famous for their sparkle. This phenomenon is due to the properties of light which is total internal reflection.

1 Rajah menunjukkan susunan radas untuk memerhati fenomena pantulan dalam penuh dan menentukan sudut
genting kaca.
The diagram shows the apparatus arrangement to observe the total internal reflection and to determine the critical angle
of glass.

Kotak sinar
Ray box
Kertas putih
N White paper
Blok kaca semibulatan
Semicircular glass block
i

O r

(i) Mengapakah sinar cahaya mesti ditujukan pada pusat bongkah kaca? TP 1 OP2
Why the light ray must be directed to the center of the glass block?
Sinar cahaya selari dengan garis normal/ Light ray parallel to normal line

(ii) Lengkapkan jadual di bawah dengan melukis dan menerangkan lintasan sinar cahaya bagi tiga keadaan.
Complete the table below by drawing and explain the path of light ray for three states. TP 2 OP3

Sudut tuju, i dan


sudut genting, c i <c i=c i>c
Incident angle, i and
critical angle, c

N N N

Lintasan sinar
cahaya i i i
Path of light ray

Dibias di sepanjang Pantulan dalam penuh


Dibias menjauhi garis
Penjelasan sempadan kaca-udara berlaku
normal
Explanation Refracted ray is at the Total internal reflection
Refracted away from normal
boundary of glass-air occured

131 © Penerbit Mahir Sdn. Bhd. (183897-P)

Fizik Tg 4 (Bab 6)6th 10/2/23.indd 131 10/02/2023 9:54 AM


Fizik Tingkatan 4 Bab 6
2 Hubung kait antara sudut genting dengan sudut biasan.
Relate between critical angle and angle of refraction.
Rajah menunjukkan sinar cahaya masuk ke dalam cecair Y. TP 3 OP4
The diagram shows a light ray entering liquid Y.

Udara
Air 90°

Cecair Y
Liquid Y
c
Sinar cahaya
Light ray

(a) Menggunakan hukum Snell, tulis satu persamaan yang menghubungkan sudut genting, c, sudut biasan
dan indeks pembiasan cecair Y.
Using Snell’s law, write an equation that relates between the critical angle, c, and the refractive index, n of liquid Y.
sin r sin 90°
n = sin i = sin c
1
n = sin c

(b) Nyatakan hubungan antara indeks biasan, n dengan sudut genting, c.


State the relationship between the refractive index, n and the critical angle, c.
n berkadar songsang dengan sin c/ n inversely proportional to sin c

3 Fenomena semula jadi dan aplikasi pantulan dalam penuh. TP 5 OP6


Natural phenomenon and application of total internal reflection.
Cahaya dari langit
Light from the sky
(a) Seorang pemandu melihat lopak air di hadapannya Pantulan dalam penuh berlaku
semasa memandu pada waktu siang yang panas. Total internal reflection occurs
Udara sejuk
Namun, lopak air tersebut hilang apabila dia sampai Cold air
di hadapan. Fenomena ini dikenali sebagai logamaya. Udara panas
Hot air
Terangkan bagaimana logamaya berlaku.
A driver see a puddle of water on the road ahead while
driving on a hot sunny day. However, the puddle of Imej awan
water went missing when he arrived at the road ahead. Image of the cloud
This phenomenon known as mirage.
Explain how mirage occurs.

(i) Pada hari yang panas, suhu udara di bahagian atas lebih sejuk berbanding suhu udara di
bahagian bawah.
On hot day, the temperature of air at upper layer is colder than the temperature of air at lower layer.

(ii) Udara yang sejuk lebih tumpat dan menyebabkan sinar cahaya dibias menjauhi garis
normal.
The cold air denser and causes the light rays refracted away from normal.
besar pantulan dalam penuh
(iii) Apabila sudut tuju lebih daripada sudut genting, maka
berlaku. Cahaya (imej awan) dipantulkan ke mata pemerhati. Pemerhati melihat imej awan sebagai
lopak air di permukaan jalan.
larger total internal reflection
When the incident angle than the critical angle, then occurs.
Light (cloud image) reflected to observer’s eye. The observer see the cloud image as puddle of water on the
road surface.

© Penerbit Mahir Sdn. Bhd. (183897-P) 132

Fizik Tg 4 (Bab 6)6th 10/2/23.indd 132 10/02/2023 9:54 AM


Fizik Tingkatan 4 Bab 6
(b) Rajah menunjukkan satu sinar cahaya memasuki sebuah gentian optik.
The diagram shows a light ray entering an optical fibre.
P

Sinar cahaya
Light ray Q

(i) Nyatakan dua bidang yang menggunakan gentian optik secara meluas.
State two fields that widely used optical fibre.
Komunikasi dan perubatan/ Communication and medical

(ii) Namakan bahan untuk gentian optik.


Name the material used for optical fiber.
Kaca/ Glass

(iii) Pada rajah di atas, lengkapkan lintasan sinar cahaya dalam gentian optik.
On the diagram above, complete the light ray path in the optical fibre.

(iv) Bandingkan antara teras dalam, Q, dengan pembalut luar, P dari aspek berikut:
Compare between the inner core, Q and the outer cladding, P in the following aspects:

• Ketumpatan: Q lebih tumpat daripada P

Density: Q denser than P


• Indeks biasan: Q lebih besar daripada P

Refractive index: Q larger than P

(v) Terangkan mengapa sinar cahaya mengalami pantulan dalam penuh apabila mengenai dinding di
dalam gentian optik.
Explain why the light ray experiences total internal reflection when it hits the inner wall of the optical fibre.
Sudut tuju lebih besar daripada sudut genting // i > c

Incident angle larger than critical angle

(vi) Berikan dua kelebihan kegunaan gentian optik.


Give two advantages of the optical fibre.
Lebih nipis dan ringan/ Thinner and lighter

Lebih banyak maklumat boleh dihantar pada satu masa/ A large number of signals can be sent at

one time

(vii) Nyatakan dua aplikasi lain bagi pantulan dalam penuh.


State two other applications for total internal reflection.
Periskop prisma/ Prism periscope

Pemantul mata kucing/ Cat’s eye reflector

133 © Penerbit Mahir Sdn. Bhd. (183897-P)

Fizik Tg 4 (Bab 6)6th 10/2/23.indd 133 10/02/2023 9:54 AM


Fizik Tingkatan 4 Bab 6

6.3 Pembentukan Imej oleh Kanta


Image Formation by Lenses

Buku Teks: m.s 242-250


1 Rajah menunjukkan susunan radas untuk menunjukkan kanta cembung sebagai kanta penumpu dan kanta
cekung sebagai kanta pencapah.
The diagram shows the apparatus arrangement to show convex lens as converging lens and concave lens as diverging lens.
Plat 5 celah
5-slit plate

Kanta cembung
Kotak sinar
silinder
Ray box
Cylindrical
convex lens

(a) Lukis rajah sinar apabila sinar cahaya selari melalui kanta cembung dan kanta cekung. Tandakan titik
fokus, F, pusat kanta, O dan panjang fokus, f. TP 2 OP3
Draw the rays diagram when the parallel lights passes through a convex lens and concave lens. Mark the focal
point, F, centre of lens, O and focal point, f.

Paksi utama F O Paksi utama


Principal axis F Principal axis
O

f f

(b) Apakah berlaku kepada sinar cahaya selari selepas melalui: TP 2 OP3
What happens to the parallel light rays after passing through:

• Kanta cembung: Sinar cahaya selari ditumpukan ke titik fokus.

Convex lens: The parallel light rays is converged to a focal point.

• Kanta cekung: Sinar cahaya selari dicapahkan seperti datang dari titik fokus.

Concave lens: The parallel light rays is diverged as they come from a point F in front of the lens.

2 Nyatakan definisi istilah berikut: TP 1 OP2


State the definition of the following terms.

Titik fokus
(a) Titik di mana semua sinar cahaya selari ditumpukan selepas dibiaskan.
Focal point
A point where all parallel light rays are converged after refraction.
(F)

Panjang fokus
(b) Jarak di antara pusat kanta, O dengan titik fokus, F.
Focal length
Distance between the optical centre, O with focal point, F.
(f)

© Penerbit Mahir Sdn. Bhd. (183897-P) 134

Fizik Tg 4 (Bab 6)6th 10/2/23.indd 134 10/02/2023 9:54 AM


Fizik Tingkatan 4 Bab 6

Jarak objek
(c) Jarak di antara pusat kanta, O dengan objek.
Object distance
Distance between the optical centre, O with object.
(u)

Jarak imej
(d) Jarak di antara pusat kanta, O dengan imej.
Image distance
Distance between the optical centre, O with image.
(v)

Pusat optik
(e) Titik di pusat kanta. Sinar cahaya yang melalui pusat optik tidak dibiaskan.
Optical centre
Point at the centre of the lens. Light rays passing through it are not refracted.
(O)

3 Rajah menunjukkan aktiviti menentukan kedudukan dan ciri-ciri imej yang terhasil. TP 4 OP5
The diagram shows the activity to determine the position and characteristics of the formed image.
Pemegang kanta
Lens holder
Kanta cembung
Convex lens
Kertas lut sinar dengan
anak panah sebagai objek
Transparent paper with
Skrin putih
u arrow as object
White screen
v

Pembaris meter
Metre rule
(a) Lukis rajah sinar bagi setiap jarak objek, u berikut dan nyatakan ciri imej untuk kanta cembung.
Draw the ray diagram for each of the following object distances, u and state the characteristics of image for
convex lens.

Rajah sinar Ciri-ciri imej


u
Ray diagram Characteristics of image

I
• Maya/ Virtual
u<f
o F • Tegak/ Upright
F • Diperbesar/ Magnified
• Imej di/ Image at: v > u

• Maya/ Virtual
o F 2F • Tegak/ Upright
u=f
2F F • Diperbesar/ Magnified
• Imej di infiniti/ Image at infinity

o • Nyata/ Real
F 2F • Songsang/ Inverted
2f > u > f
2F F • Diperbesar/ Magnified
• Imej di/ Image at: v > 2f
I

135 © Penerbit Mahir Sdn. Bhd. (183897-P)

Fizik Tg 4 (Bab 6)6th 10/2/23.indd 135 10/02/2023 9:54 AM


Fizik Tingkatan 4 Bab 6

Rajah sinar Ciri-ciri imej


u
Ray diagram Characteristics of image

O • Nyata/ Real
F 2F • Songsang/ Inverted
u = 2f
2F F • Sama saiz/ Same size
I • Imej di/ Image at: v = 2f

o • Nyata/ Real
F 2F • Songsang/ Inverted
u > 2f
2F F I • Diperkecil/ Diminished
• Imej di/ Image at: f < v < 2f

• Nyata/ Real
F F • Songsang/ Inverted
u=∞
• Diperkecil/ Diminished
I • Imej di F/ Image at F

(b) Lukis rajah sinar bagi setiap jarak objek, u berikut dan nyatakan ciri imej untuk kanta cekung.
Draw the ray diagram for each of the following object distances, u and state the characteristic of image for concave lens.

Rajah sinar Ciri-ciri imej


u
Ray diagram Characteristics of image

O • Maya/ Virtual
• Tegak/ Upright
u < 2f
2F F I • Diperkecil/ Diminished
• Imej di/ Image at: v < f

O • Maya/ Virtual
I • Tegak/ Upright
u > 2f
2F F • Diperkecil/ Diminished
• Imej di/ Image at: v < f

4 Apabila suatu objek dilihat melalui kanta pembesar pada jarak yang kurang daripada panjang fokusnya, imej
yang terbentuk diperbesar. TP 1 OP2
When an object is seen through a magnifying glass at a distance less than its focal length, the image formed is magnified.
hi v
Rumus bagi pembesaran linear, m = = di mana
ho u
hi v TIP
The formula for linear magnification, m = = where
ho u
Pintar
hi: ketinggian imej/ height of image • m < 1 : Imej diperkecil
Diminished image
h : ketinggian objek/ height of object
o • m = 1 : Imej sama saiz
Diminished image
v: jarak imej/ image distance • m > 1 : Imej diperbesar
Magnified image
u: jarak objek/ object distance

© Penerbit Mahir Sdn. Bhd. (183897-P) 136

Fizik Tg 4 (Bab 6)6th 10/2/23.indd 136 10/02/2023 9:54 AM


Fizik Tingkatan 4 Bab 6

6.4 Formula Kanta Nipis


Thin Lens Formula

Buku Teks: m.s 242-250


1 Formula kanta nipis: TP 1 OP2
Thin lens formula:

1 1 1
+ = u: jarak objek/ object distance
u v f
v: jarak imej/ image distance

f: panjang fokus/ focal length

2 Peraturan menggunakan formula kanta: TP 2 OP3


Rules using lens formula:

Positif/ Negatif (a) Positif (b) Negatif


Positive/ Negative Positive Negative

Objek nyata Objek maya


u
Real object Virtual object

Imej nyata Imej maya


v
Real image Virtual image

Kanta cembung Kanta cekung


f
Convex lens Concave lens

EKSPERIMEN 6.3

Inferens: Jarak objek, u mempengaruhi jarak imej, v


Image distance, u is influenced by object distance, v
Inference:

Hipotesis: Semakin bertambah jarak objek, semakin berkurang jarak imej.


When the object distance increases, the image distance decreases.
Hypothesis:

Tujuan: Mengkaji hubungan antara jarak objek, u dengan jarak imej, v bagi kanta cembung.
To investigate the relationship between object distance, u and image distance, v for a convex lens.
Aim:

Pemboleh ubah dimanipulasi: Jarak objek, u


Object distance, u
Manipulated variable:

Pemboleh ubah bergerak balas: Jarak imej, v


Image distance, v
Responding variable:

137 © Penerbit Mahir Sdn. Bhd. (183897-P)

Fizik Tg 4 (Bab 6)6th 10/2/23.indd 137 10/02/2023 9:54 AM


Fizik Tingkatan 4 Bab 6

Pemboleh ubah dimalarkan: Panjang fokus, f


Focal length, f
Constant variable:

Radas: Kanta cembung (f = 10.0 cm), pemegang kanta, lilin, blok kayu, skrin putih, dan pembaris meter
Convex lens (f = 10.0 cm), lens holder, candle, wooden block, white screen, and metre rule
Apparatus:

Prosedur/ Procedure:
Pemegang kanta
Lens holder
Kanta cembung Skrin putih
Convex lens White screen

Kertas lut sinar dengan v


anak panah sebagai objek
Transparent paper with u
arrow as object Pembaris meter
Metre rule

1 Laraskan jarak objek sehingga u = 15.0 cm.


Adjust the object distance until, u = 15.0 cm.
2 Laraskan skrin untuk mendapatkan imej yang tajam di atasnya.
Adjust the position of the screen until a sharp image is formed.
3 Ukur dan rekodkan jarak imej, v.
Measure anad record the image distance, v.
4 Ulang langkah eksperimen untuk u = 20.0 cm, 25.0 cm, 30.0 cm, 35.0 cm and 40.0 cm.
Repeat the experiment with object distance, u = 20.0 cm, 25.0 cm, 30.0 cm and 40.0 cm.

Keputusan/ Result:
(i) Jadualkan data bagi nilai i, r, sin i, sin r.
Tabulate data for values i, r, sin i and sin r.

1 1
u /cm v /cm /cm–1 /cm–1
u v

40.0 13.3 0.025 0.075

35.0 14.0 0.029 0.071

30.0 15.0 0.033 0.067

25.0 16.7 0.040 0.060

20.0 20.0 0.050 0.050

15.0 30.0 0.067 0.033

1 1
(ii) Lakarkan graf melawan .
v u
1 1
Plot graph of against .
v u

© Penerbit Mahir Sdn. Bhd. (183897-P) 138

Fizik Tg 4 (Bab 6)6th 10/2/23.indd 138 10/02/2023 9:54 AM


Fizik Tingkatan 4 Bab 6

1
/ cm–1
v

0.10

0.09

0.08

0.07

0.06

0.05

0.04

0.03

0.02

0.01

0.01 0.02 0.03 0.04 0.05 0.06 0.07 1


/ cm–1
u

Kesimpulan/ Conclusion:
Jarak objek, u berkadar songsang dengan jarak imej, v.

Object distance, u is inversely proportional to image distance, v.

Perbincangan/ Discussion:
(i) Tentukan kecerunan graf.
Determine the gradient of the graph.
0.10 – 0.045
m= =1
0 – 0.055
1
(ii) Tentukan pintasan paksi . Kemudian, deduksikan bagaimana nilai pintasan berhubung dengan
panjang fokus, f kanta. v
1
Determine the intercept of axis. Then, deduce how the values of the intercepts relate to the focal length, f of the
lens. v
1 1
c = 0.10 cm–1 = =
10 f

(ii) Tulis persamaan menghubungkan antara u, v dan f sebuah kanta.


Write the equation that relates between u, v and f of a lens.
1 1 1
y = mx + c → = (–1) +
v u f
1 1 1
= +
f u v

139 © Penerbit Mahir Sdn. Bhd. (183897-P)

Fizik Tg 4 (Bab 6)6th 10/2/23.indd 139 10/02/2023 9:54 AM


Fizik Tingkatan 4 Bab 6
3 Menyelesaikan masalah melibatkan formula kanta nipis. TP 3 OP4
Solve problem involving this lens formula.

(a) Satu objek diletakkan 30 cm dari kanta cembung dengan panjang fokus 20 cm.
An object is placed 30 cm from a convex lens with a focal length of 20 cm.
Objek
Object

u F

Imej
(i) Berapakah jarak imej kanta? Image
What is the image distance of the lens?
1 1 1 1 1 1
= + → = +
f u v 20 30 v
v = 60 cm

(ii) Berapakah pembesaran linear yang dihasilkan?


What is the linear magnification formed?
v 60
m= → m=
u 30
= 20 cm

(iii) Apakah ciri-ciri imej yang terhasil?


What is the characteristics of the image formed?
Nyata, songsang, diperbesar/ Real, inverted, magnified

(b) Satu objek diletakkan pada 30 cm dari kanta cekung dengan panjang fokus 20 cm.
An object is placed 30 cm from a concave lens with a focal length of 20 cm.

Objek
Object

Imej
Image

(i) Berapakah jarak imej kanta?


What is the image distance of the lens?
1 1 1 1 1 1
= + →− = +
f u v 20 30 v
v = –12 cm

(ii) Berapakah pembesaran linear kanta?


What is the linear magnification of the lens?
v 12
m= → m=
u 30
= 0.4 cm

(iii) Apakah ciri-ciri imej yang terhasil?


What is the characteristics of the image formed?
Maya, tegak, diperkecil/ Virtual, upright and diminished

© Penerbit Mahir Sdn. Bhd. (183897-P) 140

Fizik Tg 4 (Bab 6)6th 10/2/23.indd 140 10/02/2023 9:54 AM


Fizik Tingkatan 4 Bab 6

6.5 Peralatan Optik


Optical Instruments

Buku Teks: m.s 263-269


1 Kanta pembesar TP 4 OP5
Magnifying glass

Kegunaan Membesarkan imej


Application Enlarge the image

Ia mempunyai 1 kanta cembung


It consists of 1 convex lens

Jarak objek
u<f
Object distance

Ciri-ciri imej Maya, tegak, diperbesar


Characteristics of image Virtual, upright, magnified

Rajah sinar o
Ray diagram F F

2 Mikroskop majmuk TP 4 OP5


Compound microscope

Kegunaan Melihat objek seni


Application To see small object

Ia mempunyai 2 kanta cembung


It consists of 2 convex lenses

Banding fo dan fe
fo < fe
Compare fo and fe

Jarak objek
fo < u < 2fo
Object distance

Ciri-ciri imej melalui kanta


objek Nyata, songsang, diperbesar
Characteristics of image through Real, inverted, magnified
objective lens

141 © Penerbit Mahir Sdn. Bhd. (183897-P)

Fizik Tg 4 (Bab 6)6th 10/2/23.indd 141 10/02/2023 9:54 AM


Fizik Tingkatan 4 Bab 6

Kedudukan kanta mata


u < fe
Position of eyepiece lens

Ciri-ciri imej akhir Maya, tegak, diperbesar


Characteristics of final image Virtual, upright, magnified

Kanta mata
Eyepiece lense
Kanta objektif
Objek Imej
Objective lens
Object Image
Rajah sinar F1 F2 (I1) F2
Ray diagram
F1
Imej akhir Pemerhati
Final Observer
image
(I2)

3 Teleskop astronomi TP 4 OP5


Astronomical telescope

Kegunaan Melihat objek jauh


Application To see distant object

Ia mempunyai 2 kanta cembung


It consists of 2 convex lenses

Banding fO dan fe
fo > fe
Compare fo and fe

Jarak objek Infiniti// Jauh


Object distance Infinity// Distant

Bagaimana sinar cahaya melalui


kanta objek? Sinar selari
How light rays passes through Parallel rays
objective lens

Kedudukan imej melalui kanta


objek Pada titik fokus, fo
Position of image through objective At focus point, fo
lens

Ciri-ciri imej melalui kanta


objek Nyata, songsang, diperkecil
Characteristics of image through Real, inverted, diminished
objective lens

© Penerbit Mahir Sdn. Bhd. (183897-P) 142

Fizik Tg 4 (Bab 6)6th 10/2/23.indd 142 10/02/2023 9:54 AM


Fizik Tingkatan 4 Bab 6

Kedudukan kanta mata


u = fe
Position of eyepiece lens

Ciri-ciri imej akhir Maya, songsang, diperbesar


Characteristics of final image Virtual, inverted, magnified

Pelarasan normal: Jarak di


antara fo dan fe
fo + fe
Normal adjustment: The distance
between fo and fe

Pembesaran linear fo
Linear magnification fe

Kanta objek
Objective lens Kanta mata
Eyepiece lens

fo fe

Rajah sinar Fo Fe Fe
Ray diagram Pemerhati
Observer
Imej/ Image
(I1)
Imej akhir
Final image
(I2)

INFO
Kanta objektif: VIDEO
The objective lens:
• Diameter besar supaya lebih cahaya masuk dan imej lebih cerah bit.ly/3RJKGXM
Bigger diameter so that more light can enter to get a brighter image Melukis gambar rajah sinar
Drawing ray diagrams
• Pembesaran linear besar supaya imej besar
Large linear magnification so that it will produce bigger image

6.6 Pembentukan Imej oleh Cermin Sfera


Image Formation by Spherical Mirrors

Buku Teks: m.s 263-269


1 Terdapat dua jenis cermin:
There are two type of curved mirrors:

(a) Cermin cekung melengkung ke dalam .


Concave mirror curves inwards .

(b) Cermin cembung melengkung ke luar .


Convex mirror curves outwards .

143 © Penerbit Mahir Sdn. Bhd. (183897-P)

Fizik Tg 4 (Bab 6)6th 10/2/23.indd 143 10/02/2023 9:54 AM


Fizik Tingkatan 4 Bab 6
2 Lengkapkan rajah sinar untuk cermin cekung dan cermin cembung. Label panjang fokus, f dan jejari
kelengkungan, r. TP 3 OP4
Complete the ray diagram for concave mirror and convex mirror. Label the focal length, f and the radius of curvature, r.

(a) Cermin cekung (b) Cermin cembung


Concave mirror Convex mirror

Paksi utama
Principal axis C P Paksi utama
F
Principal axis P F C

f
f
r
r
3 Nyatakan definisi istilah berikut: 1
TP OP2
State the definition of the following terms.

Paksi utama (a) Garis lurus menerusi pusat kelengkungan, C dan kutub cermin, P.
Principal axis Straight line passing through the centre of curvature, C and pole of the mirror, P.

Titik fokus
(b) Titik di mana semua sinar selari menumpu selepas dipantulkan.
Focus point
A point where all parallel lights converge after reflection.
(F)

Panjang fokus
(c) Jarak di antara titik fokus, F dengan kutub cermin, P.
Focal length
Distance between focal point, F and the pole of the mirror, P.
(f)

Jarak objek
(d) Jarak dari objek ke kutub cermin.
Object distance
Distance between object and the pole of the mirror.
(u)

Jarak imej
(e) Jarak dari imej ke kutub cermin.
Image distance
Distance between image and the pole of the mirror.
(v)

Pusat kelengkungan
(f) Pusat sfera yang menghasilkan cermin cekung atau cermin cembung.
Center of curvature
Centre of sphere which produces a concave or convex mirror.
(O)

Jejari kelengkungan
(g) Jarak di antara kutub cermin, P dengan pusat kelengkungan, C.
Radius of curvature
Distance between the pole of the mirror, P and the centre of curvature, C.
(r)

© Penerbit Mahir Sdn. Bhd. (183897-P) 144

Fizik Tg 4 (Bab 6)6th 10/2/23.indd 144 10/02/2023 10:15 AM


Fizik Tingkatan 4 Bab 6
4 Lukis rajah sinar bagi setiap jarak objek, u berikut dan nyatakan ciri imej untuk cermin cekung. TP 4 OP5
Draw the ray diagram for each of the following object distances, u and state the characteristics of image for concave mirror.

Rajah sinar Ciri-ciri imej


u
Ray diagram Characteristics of image

I
O • Maya/ Virtual
u<f • Tegak/ Upright
P • Diperbesar/ Magnified
C F
• Imej di/ Image at: v > u
I

O • Maya/ Virtual
• Tegak/ Upright
u=f
C • Diperbesar/ Magnified
F P • Imej di infiniti/ Image at
infinity

O • Nyata/ Real
C • Songsang/ Inverted
2f > u > f
F P • Diperbesar/ Magnified
• Imej di/ Image at: v > 2f
I

O • Nyata/ Real
P • Songsang/ Inverted
u = 2f
C F • Sama saiz/ Same size
I • Imej di/ Image at: v = 2f

O • Nyata/ Real
F • Songsang/ Inverted
u > 2f P
• Diperkecil/ Diminished
C
I • Imej di/ Image at:
f < v < 2f

• Nyata/ Real
C F P • Songsang/ Inverted
u=∞
• Diperkecil/Diminished
I • Imej di F/ Image at F

145 © Penerbit Mahir Sdn. Bhd. (183897-P)

Fizik Tg 4 (Bab 6)6th 10/2/23.indd 145 10/02/2023 9:54 AM


Fizik Tingkatan 4 Bab 6
5 Lukis rajah sinar bagi setiap jarak objek, u berikut dan nyatakan ciri imej untuk cermin cembung.. TP 4 OP5
Draw the ray diagram for each of the following object distances, u and state the characteristics of image for convex
mirror.

Rajah sinar Ciri-ciri imej


u
Ray diagram Characteristics of image

I • Maya/ Virtual
O • Tegak/ Upright
u<f
F’ P F C • Diperkecil/ Diminished
• Imej di/ Image at: v < f

O • Maya/ Virtual
I
• Tegak/ Upright
u>f
F’ P F C • Diperkecil/ Diminished
• Imej di/ Image at: v < f

6 Rajah menunjukkan rajah sinar sebuah pemantul lampu suluh dan sebuah cermin selekoh.. TP 5 OP6
Diagrams show a ray diagram of a reflector of torchlight and a sharp corner mirror.
Sinar cahaya selari
Parallel light rays

A B
(a) Namakan jenis cermin yang digunakan dalam
Name the type of mirror used in

A: Cermin cekung/ Concave mirror

B: Cermin cembung/ Convex mirror

(b) Terangkan bagaimana cermin selekoh di B digunakan untuk melihat kereta di hadapan.
Explain how the corner mirror in B is used to see the incoming car.
Cahaya dari pemandu kereta hadapan dipantulkan melalui cermin ke mata pemerhati. Imej yang

dihasilkan adalah maya, tegak dan kecil.

The light from the incoming car will be reflected by the convex mirror to the eye’s of the observer in the car. The

image is seen by the observer as virtual, upright and diminished.

© Penerbit Mahir Sdn. Bhd. (183897-P) 146

Fizik Tg 4 (Bab 6)6th 10/2/23.indd 146 10/02/2023 9:54 AM


Fizik Tingkatan 4 Bab 6

Praktis Sumatif
KERTAS 1

1 Antara berikut, situasi manakah yang tidak 3 Rajah 2 menunjukkan sinar cahaya merambat
OP2 menunjukkan pembiasan cahaya? OP3 daripada kaca ke udara pada sudut tuju yang
Which of the following situations does not show the berbeza.
reflection of light? Diagram 2 shows light ray propagates from glass to
A air at different angles of incident.

Udara Udara
Air P Air
SM 18

B Kaca Kaca S R
Glass Q Glass

Sinar cahaya
Sinar cahaya Light ray
Light ray
C SM 18 Rajah 2
Diagram 2

Sudut manakah mewakili sudut genting kaca?


Which angle represents the critical angle of the glass?
A P
B Q
SM 19 C R
D D S

4 Instrumen manakah menggunakan konsep


OP2 pantulan dalam penuh?
2 Rajah 1 menunjukkan sinar cahaya merambat dari Which instrument uses the concept of total internal
OP3 udara ke kaca. reflection?
Diagram 1 shows
SM 19 a light ray propagates from air to A Mikroskop
glass. Microscope
B Kanta pembesar
Miagnifying glass
C Binokular berprisma
118°
Prism binocular
Sinar cahaya Kaca D Teleskop astronomi
Light ray 135° Glass Astronomical telescope
SM 20

Rajah 1 5 Rajah 3 menunjukkan objek diletakkan pada u cm


Diagram 1 OP3 dari kanta cembung. Panjang fokus kanta ialah
20 cm.
Berapakah indeks pembiasan bagi kaca itu? Diagram 3 shows an object placed at u cm from the
What is the refractive index for the glass? concave lens. The focal length of the lens is 20 cm.
A 1.38 C 1.45
B 1.51 SM 20 D 1.62

147 © Penerbit Mahir Sdn. Bhd. (183897-P)

Fizik Tg 4 (Bab 6)6th 10/2/23.indd 147 10/02/2023 9:54 AM


SM 18

Fizik Tingkatan 4 Bab 6


7 Ciri manakah sesuai untuk dijadikan kanta
u
OP5 pembesar?
KBAT Which characteristic suitable for magnifying glass?

Kanta Jarak objek


Objek Lens Object distance
Object
Cembung Kurang daripada f
A
Convex Less than f
Kanta
Lens
Cembung Di antara f dan 2f
B
Rajah 3 Convex Between f and 2 f
Diagram 3
Cekung Kurang daripada f
C
Antara berikut, ciri manakah tidak benar apabila Concave Less than f
SM 19
u ialah 10 cm, 15 cm, 35 cm dan 45 cm dari
kanta? Cekung Di antara f dan 2f
D
Which of the following characteristics of image is not Concave Between f and 2 f
true when u are 10 cm, 15 cm, 35 cm and 5 cm from
the lens? 8 Rajah 5 menunjukkan dua kereta, P dan Q, dari
OP4 arah yang bertentangan melalui selekoh tajam.
u Imej/ Image KBAT Diagram 5 shows two cars, P and Q, from opposite
direction through a sharp corner.
Maya dan diperbesar
A 10 cm
Virtual and magnified

X
Maya dan diperbesar
B 15 cm
Virtual and magnified
P
Q
Nyata dan diperkecil
C 35 cm
Real and diminished
Rajah 5
Diagram 5
SM 20
Nyata dan diperkecil Cermin manakah paling sesuai diletakkan di X
D 45 cm
Real and diminished supaya pemandu kereta P dapat melihat kereta Q?
Which mirror is the most suitable to be placed at X so
thet the driver of car P can see car Q?
6 Rajah 4 menunjukkan sebuah objek di hadapan A
OP3 sebuah kanta cembung. Cermin satah
Diagram 4 shows an object in front of a convex lens. Plane mirror

Objek
B
Object Cermin cembung
Convex mirror
f = 10 cm
C
Cermin cekung
2F F F 2F
Concave mirror
15 cm
D
Rajah 4
Diagram 4 60°
Berapakah jarak imej? Dua cermin satah
What is the image distance? bersudut 60°
A 15 cm C 25 cm Two plane mirrors
B 20 cm D 30 cm at an angle of 60°

© Penerbit Mahir Sdn. Bhd. (183897-P) 148

Fizik Tg 4 (Bab 6)6th 10/2/23.indd 148 10/02/2023 9:54 AM


Fizik Tingkatan 4 Bab 6

KERTAS 2
Bahagian A
1 Rajah 1.1 menunjukkan cermin sisi sebuah kereta yang membantu pemandu melihat objek di belakang kereta
dengan lebih berkesan.
Diagram 1.1 shows the side mirrors of a car which helps the driver to see objects behind the car more effectively.

Cermin sisi
Side mirror

Rajah 1.1
Diagram 1.1

(a) Nyatakan jenis cermin yang digunakan sebagai cermin sisi. OP2
State the type of mirror used as side mirror.
Cermin cembung/ Convex mirror
[1 markah/ mark]

(b) Berikan satu sebab mengapa cermin di 1(a) yang digunakan dan bukan cermin satah. OP3
Give one reason why the type of mirror in 1(a) is used instead of a plane mirror.
Sudut penglihatan lebih besar dan imej tegak/ Larger view of angle and upright image
[1 markah/ mark]

(c) Rajah 1.2 menunjukkan kedudukan cermin cembung. F ialah titik fokus dan C ialah pusat kelengkungan
bagi cermin cembung.
Diagram 1.2 shows a position of a convex mirror. F is the focal point and C is the centre of curvature of the convex
mirror.

Objek Imej
Object Image

C F F C

Rajah 1.2
Diagram 1.2

(i) Pada Rajah 1.2, lengkapkan rajah sinar untuk menunjukkan kedudukan imej terbentuk. OP3
In Diagram 1.2, complete the ray diagram to show the position of the image formed.
[2 markah/ marks]

(ii) Nyatakan ciri-ciri imej terbentuk. OP4


State the characteristics of image formed.
Maya, tegak dan diperkecil/ Virtual, upright and diminished
[1 markah/ mark]

149 © Penerbit Mahir Sdn. Bhd. (183897-P)

Fizik Tg 4 (Bab 6)6th 10/2/23.indd 149 10/02/2023 9:54 AM


Fizik Tingkatan 4 Bab 6
2 Rajah 2.1 menunjukkan lintasan sinar cahaya yang tidak lengkap memasuki suatu prisma kaca. Indeks biasan
prisma kaca itu adalah 1.5.
Diagram 2.1 shows an incomplete light ray path entering a glass prism. The refractive index of the glass prism is 1.5.

Prisma kaca
Glass prism

Sinar cahaya O
Light ray

45°

Rajah 2.1
Diagram 2.1

(b) Hitung sudut genting prisma kaca itu. OP4


Calculate the critical angle of the glass prism.
1 1
n = sin c → 1.5 = sin c
1
sin c =
1.5
c = 41.81°
[2 markah/ marks]

(b) Pada Rajah 2.1, lengkapkan lintasan sinar cahaya dari titik O. OP4
On Diagram 2.1, complete the light ray path from point O.
[2 markah/ marks]

(c) Berdasarkan jawapan di 2(b), namakan fenomena cahaya yang terlibat. OP2
Based on the answer in 2(b), name the light phenomenon involved.
Pantulan dalam penuh/ Total internal reflection
[2 markah/ marks]

(d) Rajah 2.2 menunjukkan binokular berprisma dengan kedudukan dua prisma pada satu sisi binokular.
Rajah 2.3 adalah rajah skematik bagi Rajah 2.2.
Diagram 2.2 shows prism binoculars with two prisms on one side of the binoculars. Diagram 2.3 is a schematic
diagram of Diagram 2.2.

Pemerhati
Prisma Observer
Prism

Rajah 2.2 Rajah 2.3


Diagram 2.2 Diagram 2.3

Pada Rajah 2.3, lukis lintasan sinar cahaya yang memasuki kedua-dua prisma itu. Dalam lukisan anda,
tunjukkan arah lintasan sinar cahaya itu. OP4
On Diagram 2.3, draw the light ray path entering both prisms. In your drawing, indicate the direction of the light
ray path.
[2 markah/ marks]
© Penerbit Mahir Sdn. Bhd. (183897-P) 150

Fizik Tg 4 (Bab 6)6th 10/2/23.indd 150 10/02/2023 9:54 AM


Fizik Tingkatan 4 Bab 6
(e) Kanta objek digunakan dalam binokular itu supaya objek boleh dilihat dengan lebih jelas. Jadual 1
menunjukkan ciri-ciri tiga kanta.
Objective lens is used in the binoculars so that the object can be seen clearer. Table 1 shows the characteristics of
three lenses.

Kanta Panjang fokus Diameter kanta


Lens Focal length Diameter of lens

P 1.0 cm Kecil/ Small

Q 10.0 cm Besar/ Big

R 100.0 cm Besar/ Big

Jadual 1
Table 1

Berdasarkan Jadual 1, nyatakan ciri-ciri kesesuaian kanta untuk digunakan sebagai kanta objek. Beri
sebab untuk kesesuaian ciri itu. OP5 KBAT
Based on Table 1, state the suitable characteristics of the lens to be used as the objective lens. Give reason for
the suitable characteristics.

(i) Panjang fokus kanta: Panjang fokus yang panjang

Focal length of lens: Longer focal length

Sebab: Pembesaran linear yang tinggi

Reason: Higher linear magnification


[2 markah/ marks]

(ii) Diameter kanta: Diameter yang besar

Diameter of lens: Bigger diameter

Sebab: Lebih banyak cahaya dibiaskan

Reason: More light to be refracted


[2 markah/ marks]

(f) Berdasarkan jawapan di 2 (e), tentukan kanta yang paling sesuai digunakan sebagai kanta objek.
Based on the answers in 2 (e), determine the most suitable lens to be used as the objective lens. OP6 KBAT

R
[1 markah/ mark]

151 © Penerbit Mahir Sdn. Bhd. (183897-P)

Fizik Tg 4 (Bab 6)6th 10/2/23.indd 151 10/02/2023 9:54 AM


F2-31

Fizik Tingkatan 4 Penilaian Akhir Tahun

Penilaian Akhir Tahun


KERTAS 1
Jawab semua soalan.
Answer all questions.

1 Antara pasangan berikut, yang manakah kedua- 4 Rajah 1 menunjukkan satu carta pita.
duanya adalah kuantiti skalar? Diagram 1 shows a tape chart.
Which of the following pairs are both scalar
quantities? SBP FIZIK SET 3 Panjang/ cm AW B
Length/ cm
A Jarak dan kerja
Distance and work
8.0
B Laju dan berat
Speed and weight
C Daya dan momentum 6.0
Force and momentum
D Sesaran dan luas
Displacement and area 4.0 F2-32 F2-33
2 Kuantiti yang manakah adalah kuantiti terbitan?
Which quantity is a derived quantity?F2-31 2.0
A Masa C Jisim
Time Mass Masa/s
B Daya D Arus elektrik 0
Time/ s
Force Electric current
F3-01
Rajah 1
3 Rumus bagi ayunan bandul ringkas adalah seperti DiagramF2-34
1 F2-35
berikut:
The formula for a simple pendulum oscillation is as Berapakah jumlah sesaran?
follows: What is the total displacement?
A 8 cm C 20 cm
4π2l B 12 cm D 28 cm F2-36
T =
g
5 Didapati minyak tertitis pada kadar yang malar
di mana T = tempoh ayunan, l = panjang bandul daripada sebuah kereta yang bergerak. Rajah 2
dan g = pecutan graviti. menunjukkan corak titisan minyak tersebut di atas
where T = oscillation period, l = length of pendulum jalan raya.
and g = gravitational acceleration. It was found that oil dripped at a constant rate from
a moving car. Diagram 2 shows the pattern of the oil
Berdasarkan rumus yang diberi, yang manakah
drips on the road.
adalah betul?
Based on the given formula, which is correct? Titisan minyak F
A T berkadar terus dengan l OilF3-04
drips
T is directly proportional to l
B T berkadar songsang dengan g
T is inversely proportional to g Arah gerakan kereta
C Graf T melawan l mewakili graf garis lurus F3-02 Direction of car's movement
Graph T against l represents linear graph
D Graf T2 melawan l mewakili graf garis lurus Rajah 2
Graph T2 against l represents linear graph Diagram 2

© Penerbit Mahir Sdn. Bhd. (183897-P) 152

Fizik Tg 4 (KM)4th 10/2/23.indd 152 10/02/2023 5:01 PM


Fizik Tingkatan 4 Penilaian Akhir Tahun
Pernyataan yang manakah menerangkan pergerakan 7 Antara graf sesaran-masa berikut, yang manakah
kereta tersebut? mewakili nyahpecutan seragam suatu objek?
Which statement describes the motion of the car? Which of following displacement-time graphs represents
A Memecut dan kemudian bergerak dengan laju uniform deceleration of an object?
seragam A Sesaran/ m
It accelerated and then moved at a uniform speed Displacement/ m
B Memecut dan kemudian nyahpecut
It accelerated and then decelerated
C Bergerak dengan laju seragam dan kemudian
nyahpecut
It moved at a uniform speed and then decelerated
D Bergerak dengan laju seragam dan kemudian Masa/ s
0 Time/ s
memecut
It moved at a uniform speed and then accelerated
B Sesaran/ m
Displacement/ m
6 Rajah 3 menunjukkan graf halaju sebuah kereta y

berubah dengan masa.


Diagram 3 shows a graph of the velocity of a car changes
with time.

Halaju/ m s-1 Masa/ s


x
Velocity/ m s-1 00 Time/ s

12 Sesaran/ m
C
Displacement/
y m
10

4
xMasa/ s
2 00 Time/ s
Masa/ s
0 1 2 3 4 5 Time/ s
D Sesaran/ m
v
Displacement/ m
Rajah 3
Diagram 3

Pernyataan yang manakah betul mengenai


pergerakan kereta itu?
Which of the following statements is correct about the
Masa/ s
motion of the car? 00 t
Time/ s
A Kereta itu pegun dari 0 s hingga 2 s
The car is stationary from 0 s to 2 s 8 Apakah yang dimaksudkan dengan jatuh bebas?
B Jumlah sesaran kereta itu ialah 28 m What is meant by free fall?
The total displacement of the car is 28 m A Jatuh dengan pecutan sifar
C Kereta itu memecut dari 2 s hingga 5 s Falling with zero acceleration
The car is accelerating from 2 s to 5 s B Jatuh dengan daya paduan sifar
D Pecutan kereta itu adalah malar dari 0 s hingga Falling with zero resultant force
2s C Jatuh disebabkan oleh graviti
The acceleration of the car is constant from 0 s to Falling due to gravity
2s D Jatuh dalam persekitaran yang mempunyai
rintangan udara
Falling in an environment of air resistance

153 © Penerbit Mahir Sdn. Bhd. (183897-P)

Fizik Tg 4 (KM)4th 10/2/23.indd 153 10/02/2023 5:01 PM


F2-36

F3-02
Fizik Tingkatan 4 Penilaian Akhir Tahun
9 Situasi yang manakah menunjukkan kesan Kuantiti fizik yang manakah menjadi lebih kecil
inersia? F3-01 selepas perlanggaran tersebut?
Which situation shows the effect of inertia? Which physical quantity becomes smaller after the
A C collision?
A Jisim
Mass
B Halaju
F3-01 Velocity
C Momentum
B D Momentum
F3-03 F3-02 D Pecutan
Acceleration

F3-03 menunjukkan kesan daya?


12 Situasi yang manakah
Which situation shows the effect of force?
A
F3-05
10 Rajah 4 menunjukkan sebutir peluruF3-04
yang berjisim
15 g ditembak pada halaju 200 m s-1 daripada
01 sepucuk pistol berjisim 2.0 kg. SBP FIZIK SET 3
Diagram 4 shows a bullet of mass 15 g is fired at
velocity of 200 m s-1 from a pistol of mass 2.0 kg.
F3-05
F3-04 B
200 m s-1

Peluru
Bullet C F3-07
F3-08

Rajah 4
Diagram 4
F3-06
SBP FIZIK SET 3 AW
Berapakah halaju sentakan pistol itu?
What is the recoil velocity of the pistol?
A –1.5 m s–1 C 3.0 m s–1
F3-05 D F3-06
B 1.5 m s–1 D 5.0 m s–1
38.0

11 Rajah 5 menunjukkan satu objek bulat A bergerak


mendekati lalu melanggar satu objek bulat B yang
pegun di atas suatu permukaan licin. Selepas
F3-07 bulat itu bergerak
perlanggaran, kedua-dua objek F3-08
bersama-sama. 13 Antara situasi berikut,
F3-10 yang manakah tidak
F3
Diagram 5 shows a circular object A moving towards menunjukkan daya impuls digunakan secara
bermanfaat? F3-
a stationary circular object B and then collides on a
smooth surface. After the collision, the two circular Which of the following situations does not show impulsive
objects move together. force is being used beneficially?
A Memaku sebatang paku pada papan kayu
Hammering a nail into a wooden plank
A B A B B Perlanggaran antara dua buah kenderaan
Collision between two vehicles
Sebelum perlanggaran Selepas perlanggaran C Menendang sebiji bola
Before collision After collision Kicking a ball
Rajah 5 D Memukul bola menggunakan raket
Diagram 5 Hitting a ball using a racket

F3-10
© Penerbit Mahir Sdn. Bhd. (183897-P) 154
F3-11
F3-13 F3-14

Fizik Tg 4 (KM)4th 10/2/23.indd 154 10/02/2023 5:01 PM


Fizik Tingkatan 4 Penilaian Akhir Tahun
14 Mengapakah bahagian hadapan dan bahagian C Planet
belakang kereta direka bentuk supaya mudah Planet
remuk? Matahari D
Why do the front and rear sections of a car are designed B Sun
to crumple easily?
A Untuk mengurangkan impuls
To decrease the impulse
B Untuk mengurangkan momentum A
To decrease the momentum
Rajah 6
C Untuk menambahkan geseran Diagram 6
To increase the friction
D Untuk menambahkan masa hentaman Antara kedudukan A, B, C dan D, yang manakah
To increase the impact time halaju planet tersebut adalah maksimum?
At which position A, B, C or D, the velocity of the
15 Apakah unit S.I. bagi pecutan graviti yang setara planet is maximum?
dengan m s-2? 19 Sebuah satelit sentiasa berada di permukaan
What is the S.I. unit for gravitational acceleration that Bumi pada tempat yang sama sepanjang tahun.
equivalent to m s-2? Berapakah tempoh orbit untuk satelit itu bergerak
A N-1 kg-1 dalam orbitnya?
B N kg A satellite is always on the surface of the Earth in the
C N kg-1 same place throughout the year.
D N kg-2 What is the orbital period for the satellite to move in
its orbit?
16 Bumi menarik Bulan dengan daya graviti 1020 N. A 3 jam/ hours C 100 hari/ days
Berapakah daya graviti Bulan menarik Bumi? B 24 jam/ hours D 365 hari/ days
The Earth is pulling the Moon with a gravitational
force of 1020 N. 20 Rajah 7 menunjukkan sebuah termometer
What is the gravitational force of the Moon that inframerah digunakan untuk mengukur suhu badan
pulling the Earth? pesakit.
A 10−20 N Diagram 7 shows an infrared thermometer is used to
SBP FIZIK SET 3 B 102 N AW of
measure the body temperature BYpatient.
M ZAIDEE
C 1010 N Termometer inframerah
D 1020 N Infrared thermometer

17 Apakah faktor yang mempengaruhi halaju lepas 38.0

bagi suatu objek dari permukaan Bumi?


What is the factor that affects the escape velocity of an
object from the surface of the Earth?
F3-07 A Jisim Bumi
F3-08
Mass of the Earth Rajah 7
F3-09
Diagram 7
B Jisim objek
Mass of the object Konsep yang manakah menerangkan prinsip kerja
C Daya graviti satelit termometer inframerah itu?
Gravitational force of the satellite Which concept explains the working principle of the
D Pecutan graviti infrared thermometer?
Gravitational acceleration A Muatan haba tentu
Specific heat capacity
18 Rajah 6 menunjukkan empat kedudukan bagi B Keseimbangan terma
sebuah planet yang bergerak mengelilingi Matahari Thermal equilibrium
mengikut orbit yang berbentuk elips. C Sentuhan terma
Diagram 6 shows four positions of a planet that is Thermal contact
moving around the Sun in an elliptical orbit. D Pemindahan haba
F3-10 Heat transfer
F3-12
F3-11
155 © Penerbit Mahir Sdn. Bhd. (183897-P)

Fizik Tg 4 (KM)4th 10/2/23.indd 155 10/02/2023 5:01 PM


F3-07
F3-08

Fizik Tingkatan 4 Penilaian Akhir Tahun


21 Rajah 8 menunjukkan lengkung pemanasan suatu Apakah yang berlaku semasa pengukusan makanan?
bahan pepejal. What happens during the food steaming?
Diagram 8 shows a heating curve of a solid substance. A Suhu stim berkurang ketika kondensasi
The temperature of steam decreases during
Suhu
Temperature condensation
B Tenaga kinetik molekul air bertambah ketika

pendidihan
AW BY M ZAIDEE
The kinetic energy of water molecules increases
P Q R S during boiling
Masa C Haba yang dibebaskan oleh stim terkondensasi
0 Time digunakan untuk memasak
Heat released by the condensed steam is used for
38.0

cooking
Rajah 8
F3-10 8
Diagram D Daya ikatan antara molekul-molekul air
bertambah ketika kondensasi
Antara pernyataan berikut, yang manakah benar? The intermolecular
F3-11binding force of water molecules
Which of the following
F3-08
statements is true? increases during condensation
I Pada peringkat P, tenaga kinetik molekul bahan F3-09
pepejal tidak berubah 23 Rajah 10 menunjukkan 100 g air pada 20°C dituang
At stage P, the kinetic energy of molecules of the ke dalam bikar yang mengandungi 200 g air
solid substance is unchanged pada 80°C.
II Pada peringkat Q, bahan pepejal melebur Diagram 10 shows 100 g of water at 20°C is poured
menjadi cecair into a beaker containing 200 g of water at 80°C.
At stage Q, the solid substance melts to liquid
III Pada peringkat R, bahan pepejal berubah
menjadi stim
At stage R, the solid substance changes to 100 g
steam 20°C
IV Pada peringkat S, haba pendam pengewapan AW BY M ZAIDEE
diserap oleh bahan itu
200 g
At stage S, latent heat of vaporisation is absorbed 80°C
by the substance
A I dan II sahaja 38.0

I and II only F3-13 F3-14 F3-12


Rajah 10
B I dan III sahaja F3-11 Diagram 10
I and III only
Berapakah suhu akhir campuran air itu?
C II dan IV sahaja
What is the final temperature of the mixture of water?
II and IVF3-08
only
A 30°C
F3-09 C 60°C
D III dan IV sahaja
B 50°C D 70°C
III and IV only
24 Antara graf berikut, yang manakah menunjukkan
22 Rajah 9 menunjukkan air yang mendidih di dalam
hubungan antara isi padu dan suhu mutlak bagi
sebuah pengukus.
suatu gas berjisim tetap pada tekanan malar?
Diagram 9 shows the water is boiling in a steamer.
Which of the following graphs shows the relationship
Kondensasi stim between the volume and absolute temperature of a fixed
berlaku
Makanan Condensation of mass of gas at constant pressure?
Food steam occur A V
Air mendidih
Boiling water

F3-16
F3-12
Rajah
F3-14 9 F3-15
F3-11 9 0 T
Diagram
F3-17

© Penerbit Mahir Sdn. Bhd. (183897-P) 156

Fizik Tg 4 (KM)4th 10/2/23.indd 156 10/02/2023 5:01 PM


F3-10

Fizik Tingkatan 4 Penilaian Akhir Tahun


B Vy 26 Rajah 12 menunjukkan dua tala bunyi yang serupa,
X dan Y, diletakkan bersebelahan. Tala bunyi X
diketuk supaya bergetar. Kemudian, tala bunyi Y
turut bergetar.
Diagram 12 shows two identical tuning forks, X and
Y, are placed side by side. Tuning fork X is struck to
vibrate. Then, tuning fork Y also vibrates.
xT
00
X Y
C Vy

Tala bunyi
Tuning fork

xT Rajah 12
00 Diagram 12
F3-13
Kesan ini dikenali sebagai
D Vy
This effect is known as
A kelangsingan C resonans
pitch resonance
B pelembapan D ayunan
damping oscillation

00 xT 27 Apabila gelombang air satah dipantulkan oleh


pemantul satah,
When the plane water waves are reflected by a plane
25 Rajah 11 menunjukkan gelombang bunyi yang
reflector,
dihasilkan oleh sebuah pembesar suara.
I frekuensinya tidak berubah
The diagram 11 shows the sound waves produced by a
their frequency does not change
loudspeaker.
II panjang gelombangnya berubah
Arah perambatan their wavelength changes
Direction of propagation III arah perambatannya berubah
F3-16 changes
their direction of propagation
A I dan II sahaja C II dan III sahaja
I and II only II and III only
B I dan III sahaja
Pembesar suara 15 m
I and III only
Loudspeaker
28 Rajah 13 menunjukkan corak pembiasan bagi
Rajah 11 gelombang air dari S ke T.
Diagram 11
Diagram 13 shows the refraction pattern of water
Jika halaju gelombang bunyi itu ialah 250 m s-1, waves from S to T.
hitung frekuensi pembesar suara tersebut? Daratan
If the velocity of the sound wave is 250 m s-1, calculate Laut
Land
Sea
the frequency of the loudspeaker?
A 15 Hz
B 25 Hz T S
C 50 Hz
D 75 Hz

Rajah 13
Diagram 13

157 © Penerbit Mahir Sdn. Bhd. (183897-P)

Fizik Tg 4 (KM)4th 10/2/23.indd 157 10/02/2023 5:01 PM


Fizik Tingkatan 4 Penilaian Akhir Tahun
Perbandingan manakah yang betul tentang ciri-ciri B pembiasan gelombang cahaya
gelombang di S dan T? refraction of light waves
Which comparison is correct about the characteristics C pembelauan gelombang cahaya
of waves at S and T? diffraction of light waves
D interferens gelombang cahaya
Frekuensi Laju interference of light waves
Frequency Speed
A T lebih rendah T lebih perlahan 31 Antara susunan berikut, yang manakah menunjukkan
daripada S daripada S jenis gelombang elektromagnet disusun dalam
T is lower than S T is slower than S urutan frekuensi menurun?
Which of the following arrangements shows the types
B T lebih rendah T lebih cepat of electromagnetic waves are arranged in descending
daripada S daripada S order of frequency?
T is lower than S T is faster than S
A Sinar-X, gelombang radio, gelombang mikro
C T sama dengan S T lebih perlahan X-rays, radio waves, microwaves
T is same as S daripada S B Gelombang mikro, cahaya nampak, sinar-X
T is slower than S Microwaves, visible light, X-rays
D T sama dengan S T lebih cepat C Sinar gama, sinar-X, gelombang radio
T is same as S daripada S Gamma rays, X-rays, radio waves
T is faster than S D Sinar-X, sinar ultraungu, sinar gama
X-rays, ultraviolet rays, gamma rays
29 Rajah 14 menunjukkan gelombang air melepasi
celah di antara penghalang konkrit berhampiran 32 Rajah 15 menunjukkan satu sinar cahaya bergerak
pantai. melalui tiga medium P, Q dan R yang berbeza
Diagram 14 shows water waves passing through a gap ketumpatan optik.
between concrete barriers near the beach. Diagram 15 shows a light ray travel through three
mediums P, Q and R of different optical densities.

Sinar cahaya 70°


Light ray P
Penghalang
konkrit
22°
Beach
Pantai Concrete
barrier Q

Rajah 14 Rajah 15
Diagram 14 Diagram 15

Fenomena gelombang yang berlaku adalah Antara pasangan berikut, yang manakah adalah
The wave phenomenon occurs is betul?
A pantulan C pembelauan Which of the following pairs is correct?
reflection diffraction
B pembiasan D interferens Ketumpatan optik Sudut genting
refraction interference Optical density Critical angle

30 Satu cahaya monokromatik melalui dwicelah. A P<Q<R 70°


Pinggir-pinggir cerah dan gelap terbentuk di atas
skrin disebabkan oleh B P>Q>R 22°
A monochromatic light passes through a double-slit.
Bright and dark fringes are formed on the screen due to C P>Q>R 70°
A pantulan gelombang cahaya
reflection of light waves D P<Q>R 22°

© Penerbit Mahir Sdn. Bhd. (183897-P) 158

Fizik Tg 4 (KM)4th 10/2/23.indd 158 10/02/2023 5:01 PM


Fizik Tingkatan 4 Penilaian Akhir Tahun
33 Halaju cahaya dalam vakum ialah 3 × 10 m s . 8 -1 36 Rajah 17 menunjukkan sinar cahaya bergerak dari
Indeks biasan bagi air ialah 1.30. objek jauh ke sebuah kanta cembung.
Berapakah halaju cahaya di dalam air? Diagram 17 shows light rays travel from a distant object
The velocity of light in vacuum is 3 × 108 m s-1. to a convex lens.
The refractive index of water is 1.30.
What is the velocity of light in water?
A 2.11 × 108 m s-1
A B C D
B 2.31 × 108 m s-1
F3-12
C 3.11 × 108 m s-1 F O F 2F
D 4.26 × 108 m s-1

34 Rajah 16 menunjukkan sebuah kapal persiaran Rajah 17


dan imejnya di atas permukaan laut yang sejuk. Diagram 17
Imej itu terbentuk disebabkan oleh logamaya
superior. Antara kedudukan A, B, C dan D, kedudukan
Diagram 16 shows a cruise and its image on a cold manakah imej terbentuk?
sea surface. The image is formed due to superior At which position A, B, C or D, the image is formed?
mirage.
37 Rajah 18 menunjukkan satu graf sin i melawan sin
Imej kapal persiaran r bagi dua jenis bongkah kaca yang berlainan, P
Image of cruise
dan Q.
Diagram 18 shows a graph of sin i against sin r for
two different glass blocks, P and Q.
Kapal persiaran sin i
Cruise

P
Rajah 16 Q
F3-15 Diagram 16

Fenomena cahaya manakah yang membentuk imej


kapal persiaran itu?
sin r
Which light phenomenon forms the image of cruise?
A Pembiasan
Refraction Rajah 18
B Pembelauan Diagram 18
Diffraction
Kecerunan graf bagi bongkah kaca P lebih besar
C Pantulan cahaya
daripada Q. Hal ini menunjukkan
Reflection of light
Gradient of the graph for glass block P is greater than
D Pantulan dalam penuh Q. This shows that
Total internal reflection A Q lebih tumpat dari P
Q is denser than P
35 Peralatan yang manakah mengaplikasikan konsep
pantulan dalam penuh? B indeks biasan P lebih besar daripada Q
the refractive index for P is greater than Q
Which equipment applies concept of total internal
reflection?
C kelajuan cahaya dalam P lebih laju daripada
A Binokular dalam Q
the speed of light in P is higher than in Q
Binocular
B Teleskop D frekuensi cahaya dalam P lebih tinggi daripada
Telescope
dalam Q
F3-17 the frequency of light in P is higher than in Q
C Mikroskop
Microscope
D Projektor slaid
Slide projector

159 © Penerbit Mahir Sdn. Bhd. (183897-P)

Fizik Tg 4 (KM)4th 10/2/23.indd 159 10/02/2023 5:01 PM


Fizik Tingkatan 4 Penilaian Akhir Tahun
38 Rajah 19 menunjukkan sebuah teleskop astronomi Apakah jenis cermin X?
dengan 100 cm dalam pelarasan normal. What is the type of mirror X?
Diagram 19 shows an astronomical telescope with A Cermin satah
100 cm in normal adjusment. Plane mirror
B Cermin cembung
Convex mirror
C Cermin cekung
Concave mirror

40 Rajah 20 menunjukkan sinar cahaya melalui satu


gentian optik. Sudut genting teras dalam ialah 46°
Rajah 19 dan θ ialah 48°.
Diagram 19 Diagram 20 shows the light ray passes through an
Diberi panjang fokus kanta mata ialah 40 cm, optical fibre. The critical angle of the inner core is 46°
and θ is 48°.
hitung pembesaran teleskop tersebut.
Given that the focal length of the eyepiece is 40 cm, A Pembalut luar
calculate the magnification of the telescope. Teras dalam Outer cladding
Inner core
B
A 2.5 C 1.5
B 2.4 D 1.3
O C
39 Rajah 21 menunjukkan imej dalam cermin X.
Udara 48°
Diagram 21 shows an image in mirror X.
Air
D
Cermin X
Mirror X Sinar cahaya
Light ray

Rajah 20
Diagram 20
Imej
Image Antara sinar cahaya, A, B, C dan D, sinar cahaya
manakah yang betul selepas melalui titik O?
Which light ray, A, B, C or D, is correct after passing
Rajah 21 through point O?
Diagram 21

© Penerbit Mahir Sdn. Bhd. (183897-P) 160

Fizik Tg 4 (KM)4th 10/2/23.indd 160 10/02/2023 5:02 PM


Fizik Tingkatan 4 Penilaian Akhir Tahun
F3-13 F3-14
KERTAS 2
Bahagian A
Jawab semua soalan dalam bahagian ini.
Answer all questions in this section.

1 Rajah 1 menunjukkan graf sesaran melawan jarak bagi gelombang air dengan frekuensi 5 Hz.
Diagram 1 shows a graph of displacement against distance for a water wave with frequency of 5 Hz.

Sesaran, s/ m
Displacement, s/ m
16 m

Jarak, d/ m
Distance, d/ m

Rajah 1
Diagram 1
F3-16
(a) Apakah yang dimaksudkan dengan gelombang melintang?
What is meant by transverse wave?
Arah getaran zarah berserenjang dengan arah perambatan gelombang.

Direction of vibration of particles is perpendicular to the direction of propagation of wave.


[1 markah/ mark]

(b) Pada Rajah 1, tanda dan label amplitud dengan simbol ʻAʼ.
On Diagram 1, mark and label amplitude with symbol ʻAʼ.
[1 markah/ mark]

(c) Berdasarkan Rajah 1, hitung halaju gelombang tersebut.


Based on Diagram 1, calculate the velocity of the wave.
λ = 8 m
v= fλ
=5×8
= 40 m s–1
[2 markah/ marks]

161 © Penerbit Mahir Sdn. Bhd. (183897-P)

Fizik Tg 4 (KM)4th 10/2/23.indd 161 10/02/2023 5:02 PM


Fizik Tingkatan 4 Penilaian Akhir Tahun
2 Rajah 2.1 menunjukkan satu cermin yang digunakan untuk memeriksa gigi pesakit dan Rajah 2.2 menunjukkan
satu cermin yang diletakkan di selekoh yang tajam di sebatang jalan.
Diagram 2.1 shows a mirror used to inspect a patient’s teeth and Diagram 2.2 shows a mirror is placed at a sharp bend
of a road.

Cermin
Mirror

Cermin
Mirror

Rajah 2.1 Rajah 2.2


Diagram 2.1 Diagram 2.2

(a) Namakan jenis cermin yang digunakan dalam


Name the type of mirror used in
(i) Rajah 2.1.
Diagram 2.1.
Cermin cekung/ Concave mirror
[1 markah/ mark]
(ii) Rajah 2.2.
Diagram 2.2.
Cermin cembung/ Convex mirror
[1 markah/ mark]

(b) (i) Berdasarkan Rajah 2.1 dan Rajah 2.2, bandingkan ciri-ciri imej yang terbentuk.
Based on Diagram 2.1 and Diagram 2.2, compare the characteristics of the images formed.
Saiz imej dalam Rajah 2.1 lebih besar daripada dalam Rajah 2.2.

Size of image in Diagram 2.1 is larger than in Diagram 2.2.

Luas medan penglihatan dalam Rajah 2.2 lebih luas daripada dalam Rajah 2.1.

Field of view in Diagram 2.2 is wider than in Diagram 2.1.


[2 markah/ marks]

(ii) Berikan kegunaan lain bagi cermin dalam Rajah 2.2.


Give other application of the mirror in Diagram 2.2.
Cermin pandang belakang // Cermin kawalan

Rear view mirror // Surveillance mirror


[1 markah/ mark]

© Penerbit Mahir Sdn. Bhd. (183897-P) 162

Fizik Tg 4 (KM)4th 10/2/23.indd 162 10/02/2023 5:02 PM


F3-14 F3-14 F3-15 F3-15
Fizik Tingkatan 4 Penilaian Akhir Tahun
3 Rajah 3.1 dan Rajah 3.2 menunjukkan dua pemanas rendam yang serupa digunakan untuk memanaskan air
dan minyak yang sama jisim. Kedua-dua bekas yang digunakan untuk mengisi air dan minyak adalah daripada
jenis yang sama.
Diagrams 3.1 and Diagram 3.2 show two similar immersion heaters that are used to heat the same mass of water and oil.
Both containers used to fill the water and the oil are of the same type of material.

q = 60°C q = 70°C

Pemanas rendam Pemanas rendam


Immersion heater Immersion heater

Air Minyak
F3-16 Water Oil

Rajah 3.1 F3-17 F3-17 Rajah 3.2


Diagram 3.1 Diagram 3.2

(a) Apakah yang dimaksudkan dengan muatan haba tentu?


What is the meaning of specific heat capacity?
Tenaga haba yang diperlukan untuk menaikkan suhu 1 kg bahan itu sebanyak 1°C.

The quantity of heat required to raise the temperature of 1 kg of substance by 1°C.


[1 markah/ mark]

(b) Bandingkan bacaan termometer dalam Rajah 3.1 dan Rajah 3.2.
Compare the thermometer readings in Diagram 3.1 and Diagram 3.2.
Bacaan termometer dalam Rajah 3.1 lebih kecil berbanding dalam Rajah 3.2.

Thermometer reading in Diagram 3.1 is lower than in Diagram 3.2.


[1 markah/ 1 mark]

(c) Terangkan bagaimana perbezaan di 3(b) berlaku.


Explain how the difference in 3(b) occurs.
Muatan haba tentu air adalah besar. Maka, kadar kenaikan suhu adalah perlahan.

The specific heat capacity of water is greater. Hence, the rate of temperature rise is slower.
[2 markah/ marks]

(d) Jika 1 kg air digunakan, perubahan suhu adalah sebanyak 10°C.


Hitung kuantiti haba yang digunakan untuk memanaskan air tersebut.
If 1 kg of water is used, the change in temperature is 10°C.
Calculate the heat quantity used to heat the water.
[Muatan haba tentu air = 4 200 J kg °C–1]
[Specific heat capacity of water = 4 200 J kg °C–1]
Q = mcq
= 1 × 4 200 × 10
= 42 000 J

[2 markah/ 2 marks]

163 © Penerbit Mahir Sdn. Bhd. (183897-P)

Fizik Tg 4 (KM)4th 10/2/23.indd 163 10/02/2023 5:02 PM


Fizik Tingkatan 4 Penilaian Akhir Tahun
4 Rajah 4.1 menunjukkan seorang lelaki sedang memandu sebuah kereta di sepanjang jalan lurus dengan halaju
malar 20 m s-1. Jumlah jisim lelaki dan kereta itu ialah 1 500 kg.
Diagram 4.1 shows a man driving a car along a straight road with a constant velocity of 20 m s-1. The total mass of the
driver and the car is 1 500 kg. SBP FIZIK SET 3
20 m s-1

Rajah 4.1
F3-18
Diagram 4.1
F3-19

(a) Apakah maksud momentum?


What is the meaning of momentum?
Hasil darab jisim dan halaju

Product of mass and velocity


[1 markah/ mark]
(b) Berapakah momentum kereta yang sedang bergerak itu?
What is the momentum of the moving car?
p = mv
= 1 500 × 20
= 3.0 × 104 kg m s–1 F3-21

[2 markah/ marks]
(c) Rajah 4.2 menunjukkan kereta yang sama dalam Rajah 4.1 diisi sehingga muatan maksimumnya
dan bergerak di sepanjang jalan lurus yang sama dengan halaju malar 20 m s–1. Jumlah
jisim kereta dan semua penumpangnya ialah 2 000 kg.
Diagram 4.2 shows the same car in Diagram 4.1 is then loaded to its maximum capacity and travels
along the same straight road with the constant velocity of 20 m s–1. The total mass of the car and all
SBP FIZIK SET 3 passengers is 2 000 kg. AW BY M ZAIDEE
20 m s-1

Rajah 4.2
F3-18 F3-19
Diagram 4.2

(i) Berapakah momentum kereta dalam Rajah 4.2?


What is the momentum of the car in Diagram 4.2?
F3-20
p = mv
= 2 000 × 20 F3-23
= 4.0 × 104 kg m s–1

[2 markah/ marks]

© Penerbit Mahir Sdn. Bhd. (183897-P) 164


F3-21
F3-22
Fizik Tg 4 (KM)4th 10/2/23.indd 164 10/02/2023 5:02 PM
Fizik Tingkatan 4 Penilaian Akhir Tahun
(ii) Apakah yang berlaku kepada momentum dan inersia kereta tersebut apabila bilangan
penumpang bertambah?
What happens to the momentum and inertia of the car when the number of passengers increases?
Momentum bertambah. Inersia bertambah.

The momentum increases. The inertia increases.


[2 markah/ marks]

(d) Jika kereta dalam Rajah 4.1 dan Rajah 4.2 mengalami keadaan kecemasan, keadaan kereta yang manakah
lebih berbahaya? Terangkan jawapan anda.
If the car in Diagram 4.1 and Diagram 4.2 is in an emergency, which car is more dangerous condition? Explain your
answer.
Keadaan kereta dalam Rajah 4.2 adalah lebih berbahaya kerana daya yang lebih besar diperlukan untuk

memberhentikan kereta itu. Jika daya membrek adalah sama, kereta itu akan bergerak lebih jauh

sebelum ia berhenti.
The car situation in Diagram 4.2 is more dangerous because a greater force is needed to stop the car. If the car

braking force is the same, the car will move further before it comes to stop.
[2 markah/ marks]

5 Rajah 5.1 dan Rajah 5.2 menunjukkan rajah sinar bagi dua cermin cekung yang sama. C ialah pusat
kelengkungan dan F ialah titik fokus.
Diagram 5.1 and Diagram 5.2 show the ray diagrams for two identical concave mirrors. C is centre of curvature and F is
focal point.
Objek
Object

C F

Cermin cekung
Imej Concave mirror
Image

Rajah 5.1
Diagram 5.1

Objek
Object

C F
Imej
Image
Cermin cekung
Concave mirror
Rajah 5.2
Diagram 5.2

165 © Penerbit Mahir Sdn. Bhd. (183897-P)

Fizik Tg 4 (KM)4th 10/2/23.indd 165 10/02/2023 5:02 PM


Fizik Tingkatan 4 Penilaian Akhir Tahun
(a) Apakah yang dimaksudkan dengan titik fokus?
What is the meaning of focal point?
Satu titik pada paksi utama iaitu semua cahaya selari tertumpu selepas dipantulkan oleh cermin

A point at principal axis where all the parallel light are focused//converged after reflected by a mirror
[1 markah/ mark]

(b) Berdasarkan Rajah 5.1 dan Rajah 5.2,


Based on Diagram 5.1 and Diagram 5.2,
(i) bandingkan jarak objek.
compare the object distance.
Jarak objek dalam Rajah 5.1 lebih dekat daripada dalam Rajah 5.2.

Object distance in Diagram 5.1 is nearer than in Diagram 5.2.


[1 markah/ mark]

(ii) bandingkan jarak imej.


compare the image distance.
Jarak imej dalam Rajah 5.1 lebih jauh daripada dalam Rajah 5.2.

Image distance in Diagram 5.1 is further than in Diagram 5.2.


[1 markah/ mark]

(iii) hubung kait jarak objek dengan jarak imej.


relate the object distance to the image distance.
Semakin dekat jarak objek, semakin jauh jarak imej

The nearer the object distance, the further the image distance
[1 markah/ mark]

(iv) nyatakan hubungan antara jarak objek dengan jarak imej.


state the relationship between the object distance and the image distance.
Berkadar songsang/ Inversely proportional
[1 markah/ mark]

(v) namakan fenomena cahaya yang terlibat.


name the light phenomenon involved.
Pantulan/ Reflection
[1 markah/ mark]

© Penerbit Mahir Sdn. Bhd. (183897-P) 166

Fizik Tg 4 (KM)4th 10/2/23.indd 166 10/02/2023 5:02 PM


Fizik Tingkatan 4 Penilaian Akhir Tahun
(c) Rajah 5.3 menunjukkan rajah sinar yang tidak lengkap.
Diagram 5.3 shows an incomplete ray diagram.

Imej
Objek
Image
Object

2F F

Rajah 5.3
Diagram 5.3
Pada Rajah 5.3, lengkapkan rajah sinar itu untuk menunjukkan bagaimana imej terbentuk.
On Diagram 5.3, complete the ray diagram to show how the image is formed.
[3 markah/ marks]

6 Rajah 6.1 menunjukkan sebuah tangki riak dengan dasar condong. Satu penggetar satah digetarkan dengan
sebuah motor elektrik untuk menghasilkan corak gelombang melintang. Rajah 6.2 menunjukkan corak
gelombang yang terbentuk di atas skrin.
Diagram 6.1 shows a ripple tank with a sloping base. A plane vibrator is vibrated with an electric motor to produce a
AW BY
transverse waves pattern. Diagram 6.2 shows the waves pattern M ZAIDEE
formed on the screen.
AW BY M ZAIDEE
Lampu
Lamp

Penggetar
Vibrator

Kawasan B
Air Region B
Water
Kawasan A
F3-19 Region A
F3-19

Skrin
F3-20
Screen
F3-20
Rajah 6.1
Diagram 6.1

-21
-21 Kawasan A Kawasan B
Region A
F3-22
RegionF3-22
B

Rajah 6.2
Diagram 6.2

(a) Apakah yang dimaksudkan dengan gelombang melintang?


What is the meaning of transverse wave?
Zarah-zarah medium bergetar pada arah yang berserenjang dengan arah perambatan gelombang

Particles of the medium vibrate in the direction perpendicular to the direction of propagation of the wave
[1 markah/ mark]

167 © Penerbit Mahir Sdn. Bhd. (183897-P)

Fizik Tg 4 (KM)4th 10/2/23.indd 167 10/02/2023 5:02 PM


Fizik Tingkatan 4 Penilaian Akhir Tahun
(b) Dengan menggunakan Rajah 6.1 dan Rajah 6.2, bandingkan
Using Diagram 6.1 and Diagram 6.2, compare
(i) kedalaman air di kawasan A dan kawasan B.
the depths of water in region A and region B.
Kedalaman air di kawasan A lebih dalam berbanding di kawasan B

The depth of water in region A is deeper than region B


[1 markah/ mark]

(ii) panjang gelombang air di kawasan A dan kawasan B.


the wavelengths of the water wave in region A and region B.
Panjang gelombang air di kawasan A lebih besar berbanding di kawasan B

The wavelength of water wave in region A is larger than region B


[1 markah/ mark]

(iii) hubung kait kedalaman air dengan panjang gelombang bagi gelombang air itu.
relate the depth of water to the wavelength of the water wave.
Apabila kedalaman air bertambah, panjang gelombong air turut bertambah.

When the depth of water increases, the wavelength of water wave also increases.
[1 markah/ mark]

(c) (i) Namakan kuantiti fizik lain yang berubah semasa perambatan gelombang air itu dari
kawasan A ke kawasan B.
Name other physical quantity that changed during the propagation of the water wave from region A to region B.
Laju gelombang/ Speed of wave
[1 markah/ mark]

(ii) Berdasarkan kuantiti fizik di 6(c)(i), terangkan corak gelombang dalam Rajah 6.2.
Based on the physical quantity in 6(c)(i), explain the waves pattern in Diagram 6.2.
Kedalaman air berkurang, laju gelombang air berkurang.

As the depth of water decreases, the speed of water wave decreases.

Berdasarkan rumus v = f λ, jika laju gelombang berkurang, maka panjang gelombang berkurang.

According to v = f λ, if the speed of wave decreases, the wavelength decreases.


[2 markah/ marks]

(d) Apakah yang berlaku kepada panjang gelombang di kawasan A jika frekuensi penggetar satah itu
bertambah?
What happens to the wavelength in region A if the frequency of the plane vibrator increases?
Bertambah/ Increases
[1 markah/ mark]

(e) Hubung kait panjang gelombang dengan frekuensi jika kedalaman air di dalam tangki riak
adalah sama.
Relate the wavelength to the frequency if the depths of water in the ripple tank are the same.
Berkadar songsang/ Inversely proportional
[1 markah/ mark]

© Penerbit Mahir Sdn. Bhd. (183897-P) 168

Fizik Tg 4 (KM)4th 10/2/23.indd 168 10/02/2023 5:02 PM


Fizik Tingkatan 4 Penilaian Akhir Tahun
7 Jadual 1 menunjukkan tiga jenis kuali dan ciri-cirinya.
Table 1 shows three types of pans and their characteristics.

Jenis kuali Jisim Muatan haba tentu


Type of pan Mass Specific heat capacity

Tembaga 387 J kg-1 °C-1


Copper 2.2 kg

Besi 500 J kg-1 °C-1


Iron 2.5 kg

Aluminium
2.9 kg 900 J kg-1 °C-1
Aluminium

Jadual 1
Table 1

(a) Berdasarkan Jadual 1, nyatakan ciri-ciri sesuai bagi kuali yang digunakan untuk menggoreng keropok
lekor dengan cepat.
Based on Table 1, state the suitable characteristics of the pan that used to fry ‘keropok lekor’ quickly.
(i) Bahan kuali.
Material of the pan.
Tembaga/ Copper
[1 markah/ mark]

(ii) Jisim kuali.


Mass of the pan.
Jisim kecil/ Small mass
[1 markah/ mark]

(b) Ketiga-tiga kuali itu dipanaskan dengan menggunakan plat pemanas elektrik berkuasa 500 W.
Berdasarkan maklumat dalam Jadual 1, hitung masa yang diambil untuk meningkatkan
suhu sebanyak 50°C bagi
The three pans is heated by using an electric hot plate of power 500 W. Based on the information in
Table 1, calculate the time taken to achieve the temperature rise of 50°C for

(i) kuali tembaga/ copper pan,

t = mcq

P
2.2 × 387 × 50
=
500
= 85.14 s
[2 markah/ marks]

169 © Penerbit Mahir Sdn. Bhd. (183897-P)

Fizik Tg 4 (KM)4th 10/2/23.indd 169 10/02/2023 5:02 PM


Fizik Tingkatan 4 Penilaian Akhir Tahun
(ii) kuali besi/ iron pan.
t = mcq
P
2.5 × 500 × 50
=
500
= 125 s
[2 markah/ marks]
SBP FIZIK SET 3
(iii) kuali aluminium/ aluminium pan.
t = mcq

P
2.9 × 900 × 50
=
500
= 261 s

[2 markah/ marks]

(c) Antara kuali-kuali yang diberi, yang manakah lebih sesuai digunakan untuk menggoreng keropok lekor
F3-18 F3-19
dalam masa yang singkat?
Which of the given pans is more suitable to be used to fry ‘keropok lekor’ in the shortest period of time?
Kuali tembaga/ Copper pan
[1 markah/ mark]

8 Rajah 8.1 dan Rajah 8.2 menunjukkan aksi aktiviti mendarat oleh dua orang murid.
Diagram 8.1 and Diagram 8.2 show two actions on landing activity by two pupils.

F3-21

Rajah 8.1 Rajah 8.2


Diagram 8.1 Diagram 8.2

(a) Kedua-dua murid itu terjun dari ketinggian yang sama dan mendarat di kawasan yang sama.
Both pupils jumped from the same height and fall on the same ground.
(i) Namakan daya yang menyebabkan kedua-dua murid itu jatuh.
Name the force that causes both pupils to fall down.
Daya graviti/ Gravitational force
[1 markah/ mark]

(ii) Apakah fenomena yang berlaku berdasarkan jawapan di 8(a)(i)?


What is the phenomenon occurs based on the answer in 8(a)(i)?
Gerakan jatuh bebas/ Free fall motion
[1 markah/ mark]

© Penerbit Mahir Sdn. Bhd. (183897-P) 170

F3-23
Fizik Tg 4 (KM)4th 10/2/23.indd 170 10/02/2023 5:02 PM
F3-20

Fizik Tingkatan 4 Penilaian Akhir Tahun


(b) Rajah 8.3 menunjukkan satu set peralatan permainan yang diletakkan di sebuah taman permainan
kanak-kanak.
Diagram 8.3 shows a set of playing equipments placed in a children’s playground.

F3-22

Rajah 8.3
Diagram 8.3

Jadual 2 menunjukkan tiga reka bentuk peralatan permainan yang diletakkan di taman
permainan kanak-kanak.
Table 2 shows three designs of the playing equipments placed in a children’s playground.

Ciri tapak Bahan untuk


Reka bentuk Ketinggian buaian taman permainan peralatan permainan
Design Height of the swing Characteristic of the Material for playing
playground’s base equipments

Rendah Getah Keluli


P
Low Rubber Steel

Tinggi Konkrit Plastik


Q
High Concrete Plastic

Rendah Getah Plastik


R
Low Rubber Plastic

Jadual 2
F3-24
Table 2

171 © Penerbit Mahir Sdn. Bhd. (183897-P)

Fizik Tg 4 (KM)4th 10/2/23.indd 171 10/02/2023 5:02 PM


Fizik Tingkatan 4 Penilaian Akhir Tahun
Berdasarkan tiga reka bentuk peralatan permainan yang ditunjukkan dalam Jadual 2, nyatakan
ciri-ciri yang paling sesuai supaya lebih selamat untuk kegunaan kanak-kanak. Berikan sebab bagi setiap
ciri berikut.
Based on the three designs of the playing equipments shown in Table 2, state the suitable characteristics
so that it is safer for the children. Give reasons for each of the following characteristics.

(i) Ketinggian buaian.


Height of the swing.
Rendah/ Low

Sebab:
Reason:
Menghindari kanak-kanak daripada jatuh dari tempat yang tinggi

To avoid children falls from a high place


[2 markah/ marks]

(ii) Ciri tapak taman permainan.


Characteristic of the playground’s base.
Diperbuat daripada getah/ Made up from rubber

Sebab:
Reason:
Memanjangkan masa kesan hentaman// Mengurangkan daya impuls

To lengthen the time of impact// To reduce the impulsive force


[2 markah/ marks]
(iii) Bahan yang digunakan untuk peralatan permainan.
Material used for the playing equipments.
Diperbuat daripada plastik/ Made up from plastic

Sebab:
Reason:
Memanjangkan masa kesan hentaman// Mengurangkan daya impuls

To lengthen the time of impact// To reduce the impulsive force


[2 markah/ marks]

(c) Berdasarkan jawapan anda di 8(b), pilih reka bentuk peralatan permainan yang paling sesuai
untuk diletakkan di taman permainan kanak-kanak.
Based on your answer in 8(b), choose the most suitable design of the playing equipments to be placed
in the children’s playground.
R
[1 markah/ mark]

© Penerbit Mahir Sdn. Bhd. (183897-P) 172

Fizik Tg 4 (KM)4th 10/2/23.indd 172 10/02/2023 5:02 PM


F3-21 F3-21

Fizik Tingkatan 4 Penilaian Akhir Tahun


Bahagian B

Jawab mana-mana satu soalan daripada bahagian ini.


Answer any one question from this section.

9 Rajah 9.1 dan Rajah 9.2 menunjukkan dua bekas yang diisi dengan air mendidih yang sama isi padu selepas
10 minit. Jisim kedua-dua bekas adalah sama dan diperbuat daripada bahan yang berbeza muatan haba tentu.
Diagram 9.1 and Diagram 9.2 show two containers are filled with same volume of boiling water after 10 minutes.
The mass of both containers are the same and are made of different materials of specific heat capacity.

Termometer
Thermometer

Air
Water

c = 880 J kg-1 °C-1 c = 450 J kg-1 °C-1

Rajah 9.1 Rajah 9.2


Diagram 9.1 F3-23 Diagram 9.2
F3-23
(a) Namakan konsep fizik yang terlibat apabila termometer itu mengukur suhu air.
Name the physics concept involved when the thermometer measures the temperature of water.
[1 markah/ mark]

(b) Menggunakan Rajah 9.1 dan Rajah 9.2, bandingkan jumlah haba awal air, muatan haba
tentu, suhu air selepas 10 minit dan kadar kehilangan haba bagi kedua-dua bekas itu. Kemudian, hubung
kait muatan haba tentu bekas dengan kadar kehilangan haba.
Using Diagram 9.1 and Diagram 9.2, compare the amount of initial heat of water, the specific heat capacity,
the temperature of water after 10 minutes and the rate of heat loss for both containers. Then, relate the specific
heat capacity of containers to the rate of heat loss.
[5 markah/ marks]

(c) Selepas beberapa minit, suhu air dan suhu bekas dalam Rajah 9.1 adalah sama. F1-26 F1-26
Terangkan bagaimana ia boleh berlaku.
After several minutes, the temperature of water and the temperature of the container in Diagram 9.1 are the same.
Explain how does it happen.
[4 markah/ marks]
F1-25 F1-25

173 © Penerbit Mahir Sdn. Bhd. (183897-P)

Fizik Tg 4 (KM)4th 10/2/23.indd 173 10/02/2023 5:02 PM


Fizik Tingkatan 4 Penilaian Akhir Tahun
(d) Rajah 9.3 menunjukkan susunan radas untuk menentukan muatan haba tentu bagi bongkah aluminium.
Diagram 9.3 shows the arrangement of the apparatus to determine the specific heat capacity of an aluminium block.
Termometer
Termometer
Thermometer
Thermometer
Bongkah aluminium
Bongkah aluminium
Aluminium block
Aluminium block
Transformer
Transformer
Transformer Bahan
Bahan Z Z
Material Z
Material Z
Lubang
Lubang
Hole
Hole
Pemanas rendam
Pemanas rendam Cecair Y
Liquid Y Y
Cecair
Immersion heater
Immersion heater Kepingan X Liquid Y
Kepingan
Plate X X
Plate X
Rajah 9.3
Diagram 9.3

Jadual 3 menunjukkan bahan-bahan yang digunakan dalam susunan radas itu.


Table 3 shows the materials used in the arrangement of the apparatus.

Bahan
Material
Set radas
Set of apparatus
Kepingan X Cecair Y Bahan Z Kuasa pemanas rendam
Plate X Liquid Y Material Z Power of immersion heater

Kayu Air Kertas tisu


P 12 W
Wood Water Tissue

Asbestos Alkohol Kertas biasa


Q 48 W
Asbestos Alcohol Plain paper

Kayu Minyak Kertas biasa


R 24 W
Wood Oil Plain paper

Asbestos Minyak Kertas tisu


S 48 W
Asbestos Oil Tissue

Kuprum Air Kertas tisu


T 24 W
Copper Water Tissue

Jadual 3
Table 3

Terangkan kesesuaian setiap bahan dalam Jadual 3 dan seterusnya tentukan set radas yang paling sesuai
digunakan untuk menentukan muatan haba tentu pepejal aluminium itu. Berikan sebab untuk jawapan
anda.
Explain the suitability of each material in Table 3 and determine the most suitable set of apparatus used to
determine the specific heat capacity of aluminium. Give reasons for your answer.
[10 markah/ marks]

© Penerbit Mahir Sdn. Bhd. (183897-P) 174

Fizik Tg 4 (KM)4th 10/2/23.indd 174 10/02/2023 5:02 PM


Fizik Tingkatan 4 Penilaian Akhir Tahun
10 Rajah 10.1 menunjukkan Farid melihat satu kawasan lopak air di atas jalan raya semasa memandu pada waktu
panas terik. Namun, apabila dia sampai ke kawasan itu dia mendapati lopak air itu tidak wujud.
Diagram 10.1 shows Farid saw a puddle of water on the road while driving during a hot day. However, when he reached
that location, he found that the puddle of water did not exist.

Rajah 10.1
Diagram 10.1
F3-24
(a) Berdasarkan situasi dalam Rajah 10.1, F3-23
Based on the situation in Diagram 10.1,
(i) namakan fenomena yang terlibat.
name the phenomenon involved.
[1 markah/ mark]
(ii) terangkan mengapa semasa Farid sampai ke kawasan itu, lopak air itu tidak wujud.
explain why when Farid reached that location, the puddle of water did not exist.
[4 markah/ marks]

(b) Rajah 10.2 menunjukkan kabel gentian optik yang digunakan dalam bidang perubatan dan komunikasi.
Diagram 10.2 shows an optical fibre cable that is used in medical and telecommunications fields.

F1-26

Rajah 10.2
Diagram 10.2
F1-25

175 © Penerbit Mahir Sdn. Bhd. (183897-P)

Fizik Tg 4 (KM)4th 10/2/23.indd 175 10/02/2023 5:02 PM


Fizik Tingkatan 4 Penilaian Akhir Tahun
Anda diminta untuk mengkaji ciri-ciri gentian optik yang digunakan dalam bidang-bidang ini seperti
ditunjukkan dalam Jadual 4.
You are required to study the characteristics of the optical fibre used in these fields as shown in Table 4.

Gentian Perbandingan antara indeks biasan


Sifat gentian optik
optik Kelenturan Ketebalan lapisan dalam, ni dan lapisan luar, no
Characteristic of
Optical Flexibility Thickness Comparison between refractive index of
optical fibre
fibre the inner layer, ni and the outer layer, no

Gentian optik
tunggal yang halus Tinggi Tebal
R ni > no
Single fine High Thick
optical fibres

Berkas gentian optik


halus yang selari Rendah Nipis
S no > ni
Bundle of fine Low Thin
parallel optical fibres

Berkas gentian optik


halus yang selari Tinggi Nipis
T ni > no
Bundle of fine High Thin
parallel optical fibres

Gentian optik
tunggal yang halus Rendah Tebal
U no > ni
Single fine Low Thick
optical fibres

Jadual 4
Table 4

Terangkan kesesuaian setiap ciri gentian optik dalam Jadual 4. Tentukan gentian optik yang paling
sesuai untuk digunakan dalam bidang komunikasi dan perubatan dengan berkesan. Berikan sebab-sebab
untuk pilihan anda.
Explain the suitability of each characteristic of the optical fibre in Table 4. Determine the most suitable optical
fibre used in telecommunications and medicine effectively. Give reasons for your choice.
[10 markah/ marks]

© Penerbit Mahir Sdn. Bhd. (183897-P) 176

Fizik Tg 4 (KM)4th 10/2/23.indd 176 10/02/2023 5:02 PM


Fizik Tingkatan 4 Penilaian Akhir Tahun
(c) Rajah 10.3 dan Rajah 10.4 masing-masing menunjukkan satu sinar cahaya melalui kaca dan berlian.
Diagram 10.3 and Diagram 10.4 show a ray of light passing into a glass and a diamond respectively.
[Indeks biasan kaca = 1.50; Indeks biasan berlian = 2.42]
[Refractive index of glass = 1.50; Refractive index of diamond = 2.42]

Sinar cahaya Sinar cahaya


Light ray Light ray

Kaca Berlian
Glass Diamond
30° 30°

Rajah 10.3 Rajah 10.4


Diagram 10.3 Diagram 10.4

(i) Hitung sudut genting bagi kaca dan berlian.


Calculate the critical angles of the glass and the diamond.
[2 markah/ marks]

(ii) Salin Rajah 10.3 dan Rajah 10.4 dan lengkapkan lintasan sinar cahaya dalam kaca dan dalam berlian
sehingga cahaya itu akhirnya keluar dari setiap objek.
Copy Diagram 10.3 and Diagram 10.4 and complete the path of the light ray in the glass and in the diamond
until it finally emerges from each object.
[3 markah/ marks]

177 © Penerbit Mahir Sdn. Bhd. (183897-P)

Fizik Tg 4 (KM)4th 10/2/23.indd 177 10/02/2023 5:02 PM


F3-23 F3-24

Fizik Tingkatan 4 Penilaian Akhir Tahun


Bahagian C

Jawab semua soalan.


Answer all questions.

11 Rajah 11 menunjukkan satu sinar cahaya merambat melalui suatu gentian optik.
Diagram 11 shows a light ray of light travelling through an optical fibre.

Sinar cahaya
Light ray
Q

F1-26
Rajah 11.1
Diagram 11.1

(a) (i) Namakan fenomena cahaya yang terlibat.


Name the light phenomenon involved.
[1 markah/ mark]

(ii) Terangkan bagaimana fenomena cahaya itu berlaku.


Explain how the light phenomenon occurs.
[4 markah/ marks]

(b) Rajah 11.2 menunjukkan satu sinar cahaya merambat dari kaca ke udara dengan sudut tuju kaca, q1.
Rajah 11.3 menunjukkan satu sinar cahaya merambat dari kaca ke udara dengan sudut tuju kaca, q2.
Diagram 11.2 shows a light ray travels from glass into air with an incident angle, q1. Diagram 11.3 shows
a light ray travels from glass into air with an incident angle, q2.

Udara
Air Udara
Air

Kaca Kaca
q1 Glass q2 Glass

Sinar tuju Sinar tuju


Incident ray Incident ray

Rajah 11.2 Rajah 11.3


Diagram 11.2 Diagram 11.3

© Penerbit Mahir Sdn. Bhd. (183897-P) 178

Fizik Tg 4 (KM)4th 10/2/23.indd 178 10/02/2023 5:02 PM


Fizik Tingkatan 4 Penilaian Akhir Tahun
Dengan menggunakan Rajah 11.2 dan Rajah 11.3;
Using Diagram 11.2 and Diagram 11.3;
(i) Bandingkan ketumpatan kaca dan udara.
Compare the density of glass and air.

(ii) Bandingkan sudut tuju, q1 dan q2.


Compare the incident angle, q1 and q2.
(iii) Bandingkan sudut tuju, q1 dan q2 dengan sudut genting, c bagi kaca.
Compare the incident angle, q1 and q2 with the critical angle, c for glass.

(iv) Nyatakan kesimpulan yang boleh dibuat.


State the conclusion that can be made.
[5 markah/ marks]

(c) Anda diberi dua keping prisma kaca serupa yang berukuran 45°, 90°, 45° dan dua kanta penumpu untuk
membina sebuah binokular ringkas. Dengan menggunakan radas yang dinyatakan, terangkan bagaimana
anda membina sebuah binokular ringkas. Cadangkan pengubahsuaian yang perlu dilakukan terhadap
binokular ringkas itu untuk menghasilkan imej yang lebih terang dan lebih besar.
You are given two pieces of identical prisms with internal angle 45°, 90°, 45° and two converging lenses to make
a binocular. Using the materials stated, explain how do you build a simple binocular. Suggest modifications to be
made on the simple binocular to produce a brighter and bigger image.
[10 markah/ marks]

179 © Penerbit Mahir Sdn. Bhd. (183897-P)

Fizik Tg 4 (KM)4th 10/2/23.indd 179 10/02/2023 5:02 PM


Fizik Tingkatan 4 Penilaian Akhir Tahun

KERTAS 3
SENARAI SEMAK CALON
CANDIDATES’ CHECK LIST

ARAHAN
Anda tidak dibenarkan bekerja dengan radas bagi lima minit pertama. Tempoh ini hendaklah digunakan untuk
menyemak senarai radas, membaca soalan dan merancang eksperimen yang akan dijalankan. Tandakan (✓) pada
ruangan kotak yang disediakan untuk menyemak bahan dan radas yang disediakan dan dibekalkan.

INSTRUCTION
You are not allowed to work with apparatus in first five minutes. This period is used to check the apparatus list, read the
question and plan the experiment which will carry out. Mark (✓) in the box provided to check the materials and apparatus
prepared and supplied.

Soalan 1
Question 1

Bil Radas / Bahan Kuantiti Ya (✓) / Tidak (7)


Number Apparatus / Material Quantity Yes (✓) / No (7)

Bilah gergaji
1 1
Hacksaw blade

Pengapit-G
2 1
G-clamp

Bola plastisin berjisim 10 g, 20 g, 30 g, 40 g dan


3 50 g 1
Plasticine ball 10 g, 20 g, 30 g, 40 g and 50 g

Jam randik (analog / digital)


4 1
Stopwatch (analog / digital)

© Penerbit Mahir Sdn. Bhd. (183897-P) 180

Fizik Tg 4 (KM)4th 10/2/23.indd 180 10/02/2023 5:02 PM


Fizik Tingkatan 4 Penilaian Akhir Tahun
1 Anda dikehendaki menjalankan satu eksperimen untuk menyiasat hubungan antara jisim, m dan tempoh
ayunan, T bagi satu neraca inersia.
You are required to carry out an experiment to investigate the relationship between mass, m and the period of oscillation,
T, of an inertia balance.

Pengapit-G
G-clamp
Bilah gergaji
Hacksaw blade
20 cm
Bola plastisin
Plasticine ball

Rajah 1
Diagram 1

Jalankan eksperimen dengan menggunakan langkah-langkah di bawah:


Carry out the experiment by using the steps below:

(i) Susun radas dengan mengapitkan sebilah gergaji pada satu hujung menggunakan pengapit-G.
Letakkan satu bola plastisin berjisim 10 g pada satu hujung. Jarak di antara bola plastisin
dengan pengapit ialah 20.0 cm. Susunan radas untuk eksperimen ditunjukkan dalam Rajah 1.
Arrange the apparatus by clamping a piece of hacksaw blade at one end using a G-clamp. Fixed a 10 g
plasticine ball at the other end. The arrangement of the apparatus for the experiment is shown in Diagram 1.

(ii) Sesarkan bilah gergaji secara mengufuk ke satu bahagian dan lepaskan supaya ia berayun. Catat
masa diambil untuk 10 ayunan, t menggunakan jam randik.
Displace the hacksaw blade horizontally to one side and release so that it oscillates. Record the time
taken for 10 oscillations, t using a stopwatch.

(iii) Ulang langkah (i) dan (ii) dengan menggunakan jisim bola plastisin, m = 20 g, 30 g, 40 g dan 50 g.
Repeat steps (i) and (ii) by using masses of plasticine, m = 20 g, 30 g, 40 g and 50 g.

(iv) Berdasarkan eksperimen yang dijalankan, anda dikehendaki untuk;


Based on the experiment conducted, you are required to;

(a) Nyatakan,
State,

(i) pemboleh ubah dimanipulasikan.


manipulated variable.
Jisim plastisin/ Mass of plasticine
[1 markah/ mark]

181 © Penerbit Mahir Sdn. Bhd. (183897-P)

Fizik Tg 4 (KM)4th 10/2/23.indd 181 10/02/2023 5:02 PM


Fizik Tingkatan 4 Penilaian Akhir Tahun
(ii) pemboleh ubah bergerak balas.
responding variable.
Tempoh ayunan/ Period of oscillation
[1 markah/ mark]

(b) Hitung tempoh ayunan bilah gergaji, T, dengan menggunakan persamaan di bawah bagi setiap
jisim bola plastisin, m. Kemudian hitung T 2.
Calculate the period of oscillation of hacksaw blade, T, by using the equation below for each mass of the
plasticine ball, m. Then, calculate T 2.

T = t
10

Jadualkan nilai t, T dan T2 bagi setiap m dalam ruang yang disediakan.


Tabulate the value of t, T and T2 for each value of m in the space provided.

m (g) t (s) T (s) T2 (s2)

20 3.7 0.37 0.14

30 4.6 0.46 0.21

40 5.0 0.50 0.25

50 5.8 0.58 0.33

60 6.4 0.64 0.41

[6 markah/ marks]

(c) Pada kertas graf yang disediakan, plot graf T2 melawan m.


On the graph paper provided, plot a graph of T2 against m.
[3 markah/ marks]

(d) Berdasarkan graf anda di (c), nyatakan hubungan antara T 2 dengan m.


Based on your graph in (c), state the relationship between T 2 and m.

T berkadar terus dengan m


2

T 2 is directly proportional to m
[1 markah/ mark]

© Penerbit Mahir Sdn. Bhd. (183897-P) 182

Fizik Tg 4 (KM)4th 10/2/23.indd 182 10/02/2023 5:02 PM


Fizik Tingkatan 4 Penilaian Akhir Tahun
(e) Hitung kecerunan, k bagi graf T 2 melawan m.
Calculate the gradient, k for the graph of T 2 against m.
4.1 – 0
k =
60 – 0
= 0.068 s2 g–1

k = 0.068 s2 g–1
[2 markah/ marks]
(f) Apakah yang akan berlaku kepada inersia bola plastisin apabila jisim bola plastisin semakin bertambah?
What will happen to the inertia of the plasticine ball when the mass of the plasticine ball increases?

Bertambah/ Increases
[1 markah/ mark]

183 © Penerbit Mahir Sdn. Bhd. (183897-P)

Fizik Tg 4 (KM)4th 10/2/23.indd 183 10/02/2023 5:02 PM


Fizik Tingkatan 4 Penilaian Akhir Tahun

Graf T 2 melawan m
Graph of T 2 against m

Tempoh ayunan, T 2 (s2)


Period of oscillation

0.4

0.3

0.2

0.1

Jisim, m (g)
0 10 20 30 40 50 60
Mass

© Penerbit Mahir Sdn. Bhd. (183897-P) 184

Fizik Tg 4 (KM)4th 10/2/23.indd 184 10/02/2023 5:02 PM


Fizik Tingkatan 4 Penilaian Akhir Tahun

Catatan

Fizik Tg 4 (KM)4th 10/2/23.indd 185 10/02/2023 5:02 PM


Fizik Tingkatan 4 Penilaian Akhir Tahun

Catatan

Fizik Tg 4 (KM)4th 10/2/23.indd 186 10/02/2023 5:02 PM

Anda mungkin juga menyukai